You are on page 1of 125

Start

När du inte förstår en förklaring eller ett exempel, eller kör fast på en uppgift - tveka inte att mejla
fysikhjalp@hermods.se

Centralt innehåll och kunskapskrav


Det är Skolverket som bestämmer de nationella kunskapskraven för ämnet. I ditt Kursbibliotek (finns sist
i varje Studieguide) finner du en snabblänk till Skolverkets webbsida med kriterier och centralt innehåll.

Lägg ordentligt med tid på att inte bara läsa utan också förstå centrala innehållet och kunskapskraven.
Gör detta successivt efterhand som du jobbar med kursen. Så här i början kommer centrala innehållet
och kunskapskraven att innehålla en hel del ord som är nya och obekanta för dig - men allt eftersom du
arbetar dig igenom kursen kommer du att förstå vad som avses!

Det är alltså centrala innehållet och kunskapskraven som styr innehållet i kursen.

Klicka på krysset och häng med till Kursens upplägg.

Kursens upplägg
Kursen Fysik 1a består av följande delar:

• Din lärobok

Kursen i Novo:

• Studieguider

• Testa dig själv (som förkortas TDS)

• Uppdrag

• Webblektioner

• Animationer

• Kursbibliotek

• Fysikhjälp

Fusk och plagiat


När man jobbar med uppdragen finns det givetvis olika sätt att göra detta. En del gör hela uppdraget på
en gång, medan andra föredrar att lösa uppgifterna succesivt. En del lär sig bäst genom att sitta i
ensamhet och läsa och räkna, medan andra hellre jobbar i grupp. Lärstilarna är nog lika många som det
finns elever! Viktigt är dock att du "äger" dina kunskaper - dvs det som du lämnar in måste du också
kunna svara för vid den avslutande Muntliga examinationen.
Det är alltså inte tillåtet att låta någon annan lösa uppgifterna i uppdragen åt dig - även om det är du
själv som håller i pennan. Detta betraktas som fusk. Inte heller får du skriva av en lösning som du hittar
på nätet - detta är ju en form av plagiat även om du skriver själv - och det inte tillåtet.

Studietips
Använd hela tiden studieguiderna som utgångspunkt för dina studier. Inte minst bör du ta fasta på
inlärningsmålen som du finner i översikten till varje studieguide. Varva med andra ord läsningen av
studieguiderna med studiet av läroboken så maximerar du din inlärning.

Är du redo att titta lite närmare på kursen och hur du navigerar till de olika delarna?

Tryck först på det röda krysset och klicka sedan på Kursen i Novo.

Fler tips!

• Läs översikten i varje studieguide noga.

• Anteckna det du läser eller gör egna sammanfattningar.

• Repetera, repetera och repetera. Repetition är inlärningens moder!

• Om du har ett betygsmål så bör du öva på att förstå kraven.

• Ha en positiv inställning, det finns forskningsstöd för att det faktiskt påverkar resultatet!

Kursen i Novo
Navet i kursen är Novo - Hermods pedagogiska plattform.

Det är i Novo som du finner de studieaktiviteter som ingår i kursen.

• En översikt

• Kursbibliotek

• Interaktiv bok

• Testa dig själv

• Avslutande examination

• Fysikhjälp

En kursöversikt
Vilka ämnesområden ingår i din kurs i Fysik 1a? Varje äpple representerar ett avsnitt i kursen. Du kan dra
pekaren över äpplet för att se vilket/vilka avsnitt som ingår i de olika modulerna.

I inledningen till respektive studieguide finner du se sedan en djupare innehållsdeklaration med


tillhörande kunskapskrav. Här vill vi enbart ge dig en snabb översikt.

Studieguide 1: Mätnoggrannhet, rörelse

Studieguide 2: Krafter, energi och rörelsemängd

Studieguide 3: Termofysik, klimat och ellära

Studieguide 4: Atom- och kärnfysik, relativitetsteori

Nästa steg!
Börjar du känna dig redo för att sätta igång kursen?

Återigen välkommen och lycka till med din kurs!

Före du startar: Repetera genom att at en snabb titt på bokhyllan igen som du hittar på nästa sida. Det
finns en sådan hylla i varje studieguide.

Genom dina studieguider kommer det att finnas symboler som du ser här i bokhyllan. Exempelvis så ser
du här en röd boll med ett i. När du ser en sådan symbol betyder det att det finns mer information att
läsa om du klickar på den knappen.

Studieguide 1

En översikt
I din första modul kommer du att först få veta lite bakgrundsfakta om fysikens utveckling. Därefter får du
lära dig grundläggande begrepp som storhet, mätetal och enhet samt även hur man ska hantera
mätnoggrannhet och avrundning. Sedan får du läsa om massa och densitet. Till sist kommer ett lite
större kapitel som behandlar rörelse - alltså sträcka, hastighet och acceleration.

Klicka nu på äpplet för att få en konkret målbeskrivning!

Jag börjar med en målbeskrivning, så att du kan få en uppfattning om vad ditt studium av den första
modulen ska leda fram till.

När du har gått igenom de tre första kapitlen, Fysikens värld, Fysikerns sätt att se och Rörelse, ska du:

• känna till viktiga milstolpar i fysikens utveckling och dess betydelse för samhället

• kunna skilja på begreppen storhet, mätetal och enhet

• förstå tankegången bakom mätnoggrannhet och veta hur du ska avrunda dina svar
• förstå och utföra beräkningar på begreppen massa, tyngd och densitet

• förstå och utföra beräkningar på begreppen sträcka, hastighet och acceleration

• förstå vad fritt fall innebär

Studietips
Arbeta med studieguiderna här i Novo parallellt med att du läser och löser uppgifter i din lärobok.

Var aktiv, titta på webblektioner och gör interaktiva övningar här i Novo, gör Testa-dig-själv och andra
övningar.

Arbeta aktivt med ditt betygsmål. Uppgifterna i uppdragen här i Novo är nivåuppdelade (E-, C- och A-
nivå). Koncentrera dig på den nivå som är ditt mål!

Utnyttja fysikhjalp@hermods.se

Att läsa i din lärobok


Fysik, s 7-18

Densitet, s 43-46 + 61

Rörelse, s 65-96

Inledning
Denna studieguide omfattar kapitel 1, 3 och 4 i läroboken.

I detta studieavsnitt ska du efter en del allmän introduktion i det första kapitlet, arbeta vidare med
kapitel 2 och 3 och lära dig grundläggande saker om:

• Tid och rum

• Massa och densitet

• Storhet, mätetal, enhet

• Rimlighet

• Hastighet

• Acceleration

• Fritt fall

• Rörelse som en graf

• Rörelselagarna
När avsnittet är klart ska du sända in ditt första uppdrag. Inga laborationer ska bifogas Uppdrag 1.

Fysikens värld
Det här kapitlet utgör en introduktion till hela denna fysikkurs. Kapitlet beskriver fysikens
problemområden, litet om den historiska utvecklingen och hur den naturvetenskapliga världsbilden har
vuxit fram. Läs igenom s 7-11, och tänk igenom vad där står. Detta kan du ha i bakhuvudet under dina
fortsatta fysikstudier.

Några punkter att fundera på:

*) Inom naturvetenskapen spelar experiment och observationer en viktig roll. De lagar och regler som vi
idag tar för givna har prövats om och om igen i olika typer av experiment och undersökningar. Varje ny
idé prövas experimentellt. På så sätt har teorierna utvecklats och ändrats under historiens gång. Nya
tekniska möjligheter gör det möjligt att göra allt mer detaljerade experiment, vilket leder till att vi kan
förstå naturens uppbyggnad allt bättre.

*) Ofta använder matematik för att enkelt kunna beskriva den verklighet vi observerar. Med
matematikens hjälp blir vår beskrivning av fysikaliska förlopp oftast mer exakt och effektiv.

*) Inom naturvetenskapen används modeller för att beskriva olika saker. Du undrar kanske vad som
menas med en modell i det här sammanhanget. De modeller som används inom fysiken är ofta av
matematisk typ, det vill säga en formel. I regel görs modellerna enklare än verkligheten då vissa mindre
betydelsefulla faktorer bortses från. Det är viktigt att känna till de bakomliggande förutsättningarna
också, så att du förstår hur du ska använda olika formler.

Vad är fysik?
Vad ska vi ha fysiken till?

I dag har utvecklingen medfört att gränserna mellan fysik och övriga naturvetenskaper åter suddats ut.
Gränsdragningen har därmed blivit mer en fråga om praktisk arbetsfördelning än om djupgående
skiljaktighet. Fysiken intar dock alltjämt en särställning, eftersom den genomtränger alla de andra
naturvetenskaperna och även de medicinska och tekniska vetenskaperna.

Man kan säga att fysiken är den materiella världens grundvetenskap. Bakom det mesta av den teknik som
vi utnyttjar dagligen ligger också fenomen och processer av fysikalisk natur. Detta gäller t.ex. när vi ringer
ett telefonsamtal, tänder en lampa, öppnar kylskåpsdörren, trycker på knappen i hissen, sätter oss i bilen
eller slår oss ner framför TV-n. Samtidigt som den fysikaliska forskningen fördjupar vår kunskap om
naturen, bereder den ständigt ny mark för den tekniska utvecklingen.

Varför fysik?

Fysiken ger oss möjlighet att förstå världen runt oss. Fysiken handlar om att beskriva naturen genom att
göra mätningar och konstruera modeller. Till exempel beskriver fysiken hur en atom är uppbyggd men
inte varför det är så. På gymnasiet handlar fysiken mycket om att lära sig ställa upp matematiska
modeller för enkla processer, att öva sig i att lösa problem samt att öva upp sin laborativa färdighet.
Dessa för att öka förståelsen för fysikaliska processer. Dessa färdigheter använder man i många andra
sammanhang.

Matematikrepetition
Fysik 1a är en kurs där du använder matematik - ja, väldigt mycket matematik. Det är en fördel om du har
läst Matematik 3, även om du klarar dig med Matematik 2. Helt nödvändigt är att du behärskar
grunderna i Matematik 1 och 2. Genom att klicka på symbolen nedan, kommer du till en repetitionsfilm
där grunderna i matte repeteras. Filmen är lång - ca 60 min. Tanken är att du ska använda dig av de bitar
du behöver, och att du ska återvända till den vid behov.

https://www.youtube.com/watch?v=r9otbY04hFU HYPERLINK "https://www.youtube.com/watch?


v=r9otbY04hFU&feature=youtu.be"& HYPERLINK "https://www.youtube.com/watch?
v=r9otbY04hFU&feature=youtu.be"feature=youtu.be

Fysikerns sätt att se


I det här kapitlet ska du inleda dina fysikstudier med att lära dig något om de grundläggande modeller
och begrepp som används i fysiken. Vi börjar med att definiera viktiga begrepp i fysiken som tid och rum
samt massa och densitet. Därefter ska vi klargöra vad som means med begreppen storhet, mätetal och
enhet. Till sist ska vi diskutera mätnoggrannhet och hur det påverkar våra resultat. Läs texten i kursboken
s 11-17 om storheter, enheter, värdesiffror och potenser/prefix.

Tid och rum

Tid och längd är ganska självklara begrepp som vi inte funderar så mycket på vad de egentligen betyder.
Vi behöver t.ex. inte kunna den exakta definitionen av 1 sekund för att mäta hur lång tid det tar att åka
till affären. När den tekniska utvecklingen ger möjlighet att göra noggranna mätningar, kan också
definitionen av grundläggande begrepp, som tid och längd, göras allt noggrannare. För muspekaren över

knapparna nedan för att läsa om definitionen av 1 sekund resp 1 meter!

Definitionen av 1 sekund utgår ifrån att elektronerna finns i ”skal” runt atomkärnan. Det finns huvudskal
och olika underskal. När en elektron hoppar mellan två skal sänds det ut ljus (strålning) med mycket
väldefinierade egenskaper, som är specifika för varje grundämne. Definitionen av 1 sekund utgår ifrån
speciell strålning som kan sändas ut av en cesiumatom.

Definitionen av 1 meter utgår ifrån att ljushastigheten i vakuum alltid är lika stor, nämligen 299 792 458
m/s. En meter är då den sträcka som ljuset rör sig på 1/299792458 sekund.

Observera vad som menas med storhet, mätetal och enhet. Du kommer snart att märka hur viktigt det är
att vara noggrann med enheterna när du löser fysikproblem. En god regel är att alltid använda de
enheter som ingår i SI-systemet. Då får du också automatiskt svaret i en SI-enhet. De sju grundenheterna
visas i tabellen på nästa sida. Längre fram får du lära dig fler SI-enheter, som alla utgår ifrån de sju
grundenheterna.
SI-enheter
Förr angavs ofta t ex längdmått med olika enheter (fot, tum, aln, meter) i olika länder. Numera har man
åtminstone inom naturvetenskaperna internationellt enats om gemensamma enheter, de s.k. SI-
enheterna. SI är en förkortning av franskans Standard International.

Meter (m), sekund (s) och kilogram (kg) är exempel på SI- enheter för storheterna längd, tid respektive
massa. Fysikaliska storheter anges med ett mätetal och en enhet.

Klicka på symbolen här intill, så kommer du till en tabell över de sju grundenheterna inom SI.

Storhet, mätetal och enhet


I fysiken använder vi olika storheter för att beteckna olika begrepp eller fenomen vi observerar, t ex
längd, tid, massa, tyngd, kraft, hastighet, acceleration, ström, spänning etc. Alla storheter uttrycks med
dels ett mätetal dels en enhet.

storhet = mätetal + enhet

Vi kan alltså aldrig säga t ex att en kropps massa är 75 utan måste alltid lägga till en enhet. Vi säger att
kroppens massa är 75 kg.

Egentligen finns två olika slag av storheter:

Skalära storheter är de vi kanske oftast möter och tänker på t ex massa, tid, area, volym… Sådana som
uppfyller det som sagts ovan. De uttrycks med ett mätetal och en enhet.

Men vi möter också begrepp där riktningen har stor betydelse. T ex hastighet, kraft, elektriska fält, etc.
Sådana storheter kallas vektoriella storheter. Det är viktigt att förutom mätetal och enhet också ange
riktningen.

Jobba igenom frågorna i Kontroll 1 sid 12. Titta på genomgången genom att klicka på symbolen här intill.

https://www.youtube.com/watch?v=MVgvAT_9j7w HYPERLINK "https://www.youtube.com/watch?


v=MVgvAT_9j7w&feature=share&list=PLvQzFdt9swYqcuWv--nhxnTOTOobnzIiV"& HYPERLINK
"https://www.youtube.com/watch?v=MVgvAT_9j7w&feature=share&list=PLvQzFdt9swYqcuWv--
nhxnTOTOobnzIiV"feature=share HYPERLINK "https://www.youtube.com/watch?
v=MVgvAT_9j7w&feature=share&list=PLvQzFdt9swYqcuWv--nhxnTOTOobnzIiV"& HYPERLINK
"https://www.youtube.com/watch?v=MVgvAT_9j7w&feature=share&list=PLvQzFdt9swYqcuWv--
nhxnTOTOobnzIiV"list=PLvQzFdt9swYqcuWv--nhxnTOTOobnzIiV

Att uppskatta värden


I nästa alla fysikaliska tillämpningar använder man olika mätvärden. En fysiker undersöker som sagt
omvärlden genom att göra mätningar.

Som fysiker och naturvetare måste du hela tiden tänka på att de mätningar du gör, aldrig ger ett exakt
svar. Ett mätvärde kan göras med olika stor noggrannhet beroende på hur du mäter, dvs. vilken
mätmetod som används, men även om du använder den bästa och modernast tänkbara
mätutrustningen, finns det alltid en viss osäkerhet i resultatet.

Två begrepp är viktiga för dig att behärska i dessa sammanhang, nämligen standardavvikelse och
värdesiffror. Vad innebär då dessa begrepp? Klicka på ikonerna nedan!

Standardavvikelse

Det går att uppskatta hur stor osäkerheten är genom att göra fler mätningar av samma sak och
bestämma medelvärdet. För att ange hur stor osäkerheten är kan man använda något statistiskt mått,
t.ex. standardavvikelsen. Även om det inte är den mest korrekta feluppskattningsmetoden så får man i
alla fall en uppfattning om osäkerhet i ett resultat med den metoden. I Matematik 2 har du fått lära dig
standardavvikelse. Repetera gärna i din mattebok, eller läs förklaringen genom att klicka här.
http://www.matteboken.se/lektioner/matte-2/statistik/standardavvikelse

Standardavvikelse går ganska lätt att snabbt beräkna med de flesta funktionsräknarna. Studera din
räknares handbok för att se hur man skall göra.

Värdesiffror

Det är också viktigt att antalet värdesiffror (ibland kallas det säkra siffror eller gällande siffror) i svaret är
anpassat efter mätnoggrannheten i de värden som ligger till grund för en viss beräkning.

Osäkerheten i ett mätvärde där inget annat anges anses vara maximalt en halv enhet i den sista
värdesiffran. Dvs om ett värde anges som t ex 3,4 m anser man att det verkliga värdet ligger i intervallet
3,35-3,45 m. (Tänk på hur man avrundar ett tal till, som i detta fall, en decimal). När man använder detta
värde i en uträkning kan inte resultatet bli bättre än det intervall som uppstår då man använder dessa
yttervärden var för sig. Givetvis ökar felet i ett resultat om flera mätvärden används. Om mätvärdet
multipliceras med något tal multipliceras också felet.

Grundregel: En enkel och bra grundregel när det gäller svaret till en uppgift är att alltid svara med högst
samma noggrannhet som det i uppgiften sämst noggranna mätvärdet har.

Övningar på värdesiffror finns på kommande sidor!

Värdesiffror
Som regel svarar man inte med fler värdesiffror än det finns i det minsta mätetalet.

Om följande tal multipliceras:

24 · 67: →2 värdesiffror i svaret.

24 · 2451: →2 värdesiffror i svaret.

4 · 267: →1 värdesiffra i svaret.

Titta på genomgången genom att klicka på filmsymbolen. https://www.youtube.com/watch?


v=fMbmHNRtEG0 HYPERLINK "https://www.youtube.com/watch?
v=fMbmHNRtEG0&index=37&list=PLvQzFdt9swYqcuWv--nhxnTOTOobnzIiV"& HYPERLINK
"https://www.youtube.com/watch?v=fMbmHNRtEG0&index=37&list=PLvQzFdt9swYqcuWv--
nhxnTOTOobnzIiV"index=37 HYPERLINK "https://www.youtube.com/watch?
v=fMbmHNRtEG0&index=37&list=PLvQzFdt9swYqcuWv--nhxnTOTOobnzIiV"& HYPERLINK
"https://www.youtube.com/watch?v=fMbmHNRtEG0&index=37&list=PLvQzFdt9swYqcuWv--
nhxnTOTOobnzIiV"list=PLvQzFdt9swYqcuWv--nhxnTOTOobnzIiV

Exempel:

Ett rum uppskattas till 4,28 m x 6,85 m.

a) Uppskatta felen i måtten (0,5 cm).

b) Bestäm en övre (Amax) resp en undre (Amin) gräns för rummets area: 29,373675 m2 resp 29,262375 m2

c) Bestäm felet (ΔA) i areaberäkningen: ΔA = (A max - Amin)/2 = (29,373675 - 29,262375)/2 = 0,05565 m 2 ≈


0,1 m2. Fjärde siffran (andra decimalen) är osäker. Vi nöjer oss därför med en decimal, och avrundar felet
uppåt.

d) Med hur många siffror är det rimligt att ange arean? Felet ligger i tredje siffran. Det finns alltså ingen
anledning att ange arean med mer än tre siffror (29,3 m 2).

e) Ange arean med felgränser: A = (29,3 ± 0,1) m 2

Värdesiffror, forts
Exempel:

Ange antalet värdesiffror i följande tal: a) 257 b) 25,7 c) 25,70 d) 0,25 e) 20

Svar: a) 3 st b) 3 st c) 4 st d) 2 st e) 1 st

Exempel:

Världsrekordet i simning på 100 m i en 50-metersbassäng är omkring en minut. Hur stort får felet högst
vara i bassängens längd för att det skall vara meningsfullt att mäta tiden i hundradels sekunder?

Försök lösa uppgiften själv. Kontrollera sedan din lösning genom att klicka på simbassängen!

Lösning:

v = s/t = 100/60 = 1,6667 m/s

På tiden Δt = 0,01 s kommer simmaren alltså sträckan

Δs = v · Δt = 1,6667 · 0,01 m = 0,016667 m

Mätfelet i bassängens längd får alltså vara högst halva den sträckan (bassängen är 50 m), dvs högst 8
mm. Är det rimligt att bygga så exakta bassänger? Är det meningsfullt att mäta tiden i tusendelar?
Stora och små tal
Inom fysiken används ofta väldigt stora respektive små tal. Exempelvis är ljushastigheten 300000000 m/s
och radien hos en atom i storleksordningen 0,0000000005 m.

Dessa tal skrivs enklare på grundpotensform: 3 · 108 m/s respektive 5 · 10-10 m.

Exempel: Skriv följande tal på grundpotensform:

a) 8450 m b) 0,0032 g c) 0,000000450 m

Svar:

a) 8,45 · 103 m b) 3,2 · 10-3 g c) 0,000000450 m = 4,50 · 10-7 m

Exempel: Hur långt är ett ljusår uttryckt i mil?

Lösning: Ljushastigheten v = 3 · 108 m/s. Tiden t är antalet sekunder på ett år. Sträckan ljuset färdas på ett
år:

s = v · t = 3 ∙ 108 ∙ 365 ∙ 24 ∙ 3600 m = 9,4608 ∙ 1015�𝑚m = 9,5 ∙ 1011 mil

Tiopotenser
Naturvetenskapen, och astronomin i synnerhet, rör sig med väldigt stora och små tal. För att förenkla
skrivandet av dessa använder vi tiopotenser. Exempelvis: Jorden har en massa av: 5 980 000 000 000 000
000 000 000 kg, ganska svårhanterligt… Med tiopotenser skriver istället 5,98 · 10 24 kg (talet 24 i
exponenten talar om hur många signifikanta decimaler talet innehåller).

På samma sätt gäller att

5,98 = 5,98 · 100

59,8 = 5,98 · 101

598 = 5,98 · 102

osv.

En elektron har en massa av: 0,000 000 000 000 000 000 000 000 000 00091 kg. Med tiopotenser skriver
vi istället: 9,1 · 10-31 kg (antalet decimaler fram t.o.m. 9:an). På samma sätt gäller att:

0,91 = 9,1 · 10-1

0,091 = 9,1 · 10-2

0,0091 = 9,1 · 10-3


! Antalet atomer i, för oss, observerbara universum: ca 10 78 st.

Tabell med exempel på stora tal i potensform:

Tabell med exempel på små tal i potensform:

Exempel på potensräkning

Prefix
Skall man uttrycka t ex 180 000 W är det ibland enklare att använda ett prefix och säga 180 kW eller 0,18
MW. En tabell över prefixen hittar du i din formelsamling, eller genom att klicka här.
http://www.su.se/polopoly_fs/1.129052.1386338585!/menu/standard/file/Formelblad%20kurs
%201.pdf

Exempel:

Skriv följande tal med prefix: a) 8450 m b) 0,0032 g c) 0,000000450 m d) 6,96 · 10 -4 g

Svar:

a) 8,45 km b) 3,2 mg c) 0,450 μm eller 450 ng d) 0,696 mg eller 696 μg

Av tradition används i fysiken - liksom i matematiken - väldigt ofta bokstäver från det grekiska alfabetet.
Du är ju redan bekant med π och ρ. Det fullständiga grekiska alfabetet hittar du genom att klicka här.
C:\Users\sdur1\Documents\Grekiska alfabetet.pdf

En filmad genomgång av potenser, prefix och storleksordning ser du här:


https://www.youtube.com/watch?v=f-qQsaK9124 HYPERLINK "https://www.youtube.com/watch?v=f-
qQsaK9124&list=PLvQzFdt9swYqcuWv--nhxnTOTOobnzIiV&feature=share&index=1"& HYPERLINK
"https://www.youtube.com/watch?v=f-qQsaK9124&list=PLvQzFdt9swYqcuWv--
nhxnTOTOobnzIiV&feature=share&index=1"list=PLvQzFdt9swYqcuWv--nhxnTOTOobnzIiV HYPERLINK
"https://www.youtube.com/watch?v=f-qQsaK9124&list=PLvQzFdt9swYqcuWv--
nhxnTOTOobnzIiV&feature=share&index=1"& HYPERLINK "https://www.youtube.com/watch?v=f-
qQsaK9124&list=PLvQzFdt9swYqcuWv--nhxnTOTOobnzIiV&feature=share&index=1"feature=share
HYPERLINK "https://www.youtube.com/watch?v=f-qQsaK9124&list=PLvQzFdt9swYqcuWv--
nhxnTOTOobnzIiV&feature=share&index=1"& HYPERLINK "https://www.youtube.com/watch?v=f-
qQsaK9124&list=PLvQzFdt9swYqcuWv--nhxnTOTOobnzIiV&feature=share&index=1"index=1
Massa och densitet
Läs texten i kursboken s 43 - 46.

Ofta förväxlas i dagligt tal begrepp som massa - vikt - tyngd.

Massa och vikt är samma sak och anges i ofta i SI-enheten kg. Andra tänkbara enheter är ton, g (gram),
mg (milligram) etc. Tyngd däremot är detsamma som kraft vilket vi behandlar i nästa modul. En enhet för
kraft är, som du säkert känner igen sedan grundskolan, N (Newton).

Lite populärt brukar man säga att en massa eller vikt är densamma var vi än befinner oss, men tyngden
kan ändras. Om vi t ex flyttar oss till månen kommer tyngden inte längre vara densamma utan bli mindre
eftersom månens dragningskraft på ett visst föremål är mindre än jordens.

Vi definierar begreppet densitet som massa delat med volym. Som symbol för storheten densitet brukar
vi använda en grekisk bokstav som uttalas ”rå” (se marginalen). Enheten kan vara kg/m 3, g/dm3 eller
någon annan kombination av mass- och volymenhet enligt sambandet ovan. Kom ihåg det som sagts om
enheter i förra avsnittet.

= ”rå”

Materia tar plats


Hur stor plats något tar kallas volym. Massan är däremot alltid samma. Det som skiljer är att i olika
ämnen är atomerna olika tät packade, dvs. det får plats fler atomer i samma volym. Det beskriver man i
fysiken med begreppet densitet, eller vardagligt "täthet".

Densiteten definieras som massan delat med volymen, och betecknas med

Eller som en formel:

Densiteten anger hur mycket ett ämne väger per volymsenhet (specifika vikten). Densiteten mäts i SI-
enheten kg/m3, och dess värde för olika ämnen finns i formelsamlingen. Vatten har densiteten 1000
kg/m3. Ämnen med lägre densitet än vatten (t ex olja och trä) flyter på vatten, medan ämnen med högre
densitet (t ex metaller) sjunker.

Exempel: En metallklump väger 745 g och har volymen 83 cm 3. Vilket ämne är det?

Lösning: Massan m = 745 g, volymen V = 83 cm3. Densiteten ρ = m/V = 745/83 g/cm3 = 8,98 g/cm3.

Enligt tabellen troligtvis koppar (8,96 g/cm3).

Lös uppgifterna i din lärobok.

På följande två sidor följer också fler uppgifter på densitet!


Uppgifter på densitet
1.) Ange densiteten hos koppar i uppgiften på föregående sida i SI-enheten kg/m 3.

2.) Vad är massan av luften i ett klassrum med måtten 10 m x 6,0 m x 3,5 m? Luftens densitet är 1,3
kg/m3.

3.) Bestäm massan för en 8,0 mm tjock glasruta till ett skyltfönster med måtten 2,8 m x 7,9 m. Densiteten
för glas är 2500 kg/m3.

4.) Vilken radie (r) har ett järnklot som väger 25 kg? Densiteten för järn är 7300 kg/m 3.

Lösningar

1.) 8,96 g/cm3 = 8,96 · 0,0001 kg/0,000001 m3 = 8,96 ·103 kg/m3

2.) m = ρ · V = 1,3 · 10 · 6,0 · 3,5 kg = 273 kg = 270 kg

3.) Eftersom densiteten är angiven i kg/m 3 omvandlar vi alla mått till m.

4.)

Uppgifter på densitet, forts


5.) Vilka fasta ämnen (4 st) har störst densitet? Använd din tabell.

6.) Vilket har störst volym: 1 kg guld eller 1 kg aluminium? Varför?

7.) När vatten fryser till is utvidgar det sig (en flaska kan spricka). Vad säger det oss om vattnets resp
isens densitet? (Kontrollera i din tabell.)

Lösningar

5.) Svar: Platina, volfram, guld, uran

6.) Svar: Aluminium har störst volym! Aluminium har lägre densitet än guld, så om de ska ha samma
massa måste aluminium ha störst volym.

7.) Svar: Eftersom det fortfarande är samma massa och isens volym ökar, så måste densiteten för is vara
mindre än för vatten. ρ vatten = 998 kg/m3 och ρ is = 920 kg/m3
Laboration
*) Du skall nu genomföra en liten laboration via datorn. Här skall du undersöka massa, volym och
densitet och söka svar på frågorna nedan:

- Varför flyter föremål som t.ex. trä i vatten? Beror det på storleken?

Skapa ett anpassat objekt för att undersöka effekterna av massa och volym på täthet.

- Kan du upptäcka förhållandet? Använd vågen för att mäta massan av ett objekt, och sedan håll
föremålet under vatten för att mäta dess volym.

- Kan du identifiera alla mystiska objekt?

- Klicka på knappen och kör laborationen! http://phet.colorado.edu/sims/density-and-


buoyancy/density_en.html

*) Titta också på lektionen om densitet: https://www.youtube.com/watch?v=07SMdcDCmK8

*) Till sist en lite svårare uppgift - om du vill ha lite extra utmaning:

En tom aluminiumburk har massan 15 g. Vi betraktar burken som en cylinder med höjden 11,7 cm och
diametern 6,0 cm. Hur tjock är aluminiumplåten? Densiteten för aluminium är 2,7 g/cm 3 . Lösning får du
genom att klicka på burken här bredvid.

Lösning

Sammanfattning
Du ska nu kunna följande grundläggande begrepp och motsvarande samband:

• Massa och densitet

• Storhet, mätetal och enhet

• SI-systemets grundenheter

• Mätnoggrannhet och svarsnoggrannhet

• Beräkna standardavvikelse med räknare

*) Gå igenom exempel 1 och 2 på s 16-17.

*) Lös Kontroll 2, 3 och 4, s 14 och 17.

*) Lös Kontroll 1, s 46.

*) Lös övningar 1.1 - 1.10 i läroboken.


*) Lös övningar 3.1 - 3.11 i läroboken.

Rörelse
I det här kapitlet ska vi studera några olika typer av rörelse. Först tittar vi på rörelse med konstant
hastighet. Med hjälp av vardagliga exempel - t.ex. en cykeltur till affären eller en bil som accelereras - ska
vi sedan gå vidare med rörelse där hastigheten inte är konstant, utan förändras på något sätt, och
definiera begrepp som medelhastighet, momentanhastighet och acceleration. Du får också lära dig att
tolka grafer som beskriver rörelser av olika slag.

Nedan tar vi upp de centrala kursbegreppen kring likformig accelererad rörelse. Du läser texten i
kursboken s 65 - 90 för att täcka in hela området. Lär dig hitta i texten, notera speciellt figurer och de grå
faktarutorna i med formler och definitioner.

Först förklaras med en mängd exempel begreppet medelhastighet och rörelse med konstant hastighet.
Dessa begrepp är säkert väl kända från grundskolans fysikkurs.

Här används det kanske redan kända sambandet:

Vägsträckan betecknas med s, hastigheten med v och tiden med t.

Linjebunden rörelse
En rörelse utefter en linje kallas linjebunden. Exempel på detta är ett tåg som rör sig på järnvägen. En bil
på en parkeringsplats eller ett flygplan i luften är ju däremot fria att röra sig i två respektive tre riktningar.
Gymnasiefysiken handlar om att beskriva läget, hastigheten och accelerationen hos föremål som rör sig
linjebundet. Under en bilfärd kan man inte hålla samma fart hela tiden. Man får ofta stanna vid
övergångsställen eller för rött ljus. Därefter ökar åter farten tills man kanske måste bromsa in på nytt.
Trots att farten har ändrats, kan vi beräkna hur stor den har varit i genomsnitt, alltså vi kan beräkna
medelfarten:

Medelfarten = sträckan / tiden

Till vardags anger man nästan alltid hastigheter i km/h. Inom fysiken använder man oftast enheten m/s
(meter per sekund) för hastigheter. I vissa sammanhang skiljer man inom fysiken på "fart" och
"hastighet". Om vi säger fart, så menar vi hur fort. Om vi säger hastighet så menar vi hur fort och i vilken
riktning.

Enligt fysikens lagar måste det till en kraft för att sätta ett föremål i rörelse. Detta verkar ju ganska
självklart. Det är kanske svårare att förstå att föremålet sedan fortsätter i samma riktning och med
samma hastighet utan hjälp av någon kraft. I praktiken avstannar rörelsen efter en stund, men inte av sig
själv! Föremålet stannar därför att det utsätts för bromsande krafter som orsakas av luftmotstånd och
friktion.
Konstant hastighet
Den enklaste rörelsen är när ett föremål färdas med konstant hastighet, d v s som har samma fart hela
tiden. Ett fritt fallande föremål ökar däremot farten hela tiden (upp till c:a 240 km/h för en fritt fallande
fallskärmshoppare) och har alltså inte konstant hastighet. Ett exempel på ett föremål som har konstant
fart är en liten kula som sjunker i vatten. Vattnets motstånd gör att hastigheten snabbt blivit konstant.
Om vi ritar en graf som beskriver hur sträckan beror på tiden får grafen följande utseende. Det kallar vi
en s-t- graf. Observera att s-t-grafens lutning (k-värdet) anger hastigheten.

Du kan nu göra Testa-dig-själv 1:1!

Momentanhastighet
Vi vill också kunna bestämma hastigheten i ett ögonblick när rörelsen inte sker med konstant hastighet.
Då används begreppet momentanhastighet.

Momentanhastighet definieras som hastigheten vid en viss tidpunkt (i ett visst ögonblick = momentant). I
praktiken bestämmer man ofta momentanhastigheten genom att beräkna medelhastigheten under ett
mycket litet tidsintervall.

För att bestämma momentanhastigheten är det enklast att dra en tangent till kurvan. En tangent har
precis samma lutning som kurvan själv i den aktuella punkten, så vi kan bestämma hastigheten med
hjälp av två punkter på tangenten.

Studera noggrant bokens exempel 1 och 2, s 71-73. Tänk igenom hur s-t grafer kan se ut. Lös och räkna
fysikuppgifter i kursboken 4.1-4.11 på s 91-93.

Exempel:

Vi studerar en modell för ett hundrameterslopp under de två första sekunderna och får följande
samband: s(t) = 5t2 - 2,5t3

Nedan visas s-t-grafen för loppet. Ofta ger ju ett diagram en bättre bild än siffror av ett förlopp. Med
hjälp av grafen kan vi t ex beräkna medel- och momentanhastighet. På nästa sida kommer du att se hur
en sekant och en tangent har ritats in i grafen.

Medelhastighet
Medelhastigheten beräknas som förflyttad sträcka delat med den tid förflyttningen tar. Detta uttryck
gäller oavsett om hastigheten varierar.

SI-enheten för hastighet är alltså m/s. I vardagslag anges hastigheten ofta i enheten km/h. Du bör i
allmänhet omvandla till m/s om du ska göra beräkningar. Det gör du genom att dividera med faktorn 3,6.
Faktorn 3,6 är enkelt att hitta. Om vi utgår från 1 km/h får vi att 1 km är 1000 m och att 1 h är 3600 s.
Då får vi att: 1 m/s = 3,6 km/h. Om du vill omvandla km/h till m/s så dividerar du med 3,6. Om du
istället ska omvandla från m/s till km/h multiplicerar du med 3,6.

Rörelse med konstant hastighet kallas också likformig rörelse.

SI-enheten för hastighet är m/s. Medelhastigheten ger en uppfattning om hur stor hastigheten är i
genomsnitt under ett visst tidsintervall. Den säger ingenting om hur stor hastigheten är i ett visst
ögonblick. Observera speciellt att medelhastigheten inte beräknas som medelvärdet av olika hastigheter
under rörelsen, utan som totala sträckan genom totala tiden.

Medel- och momentanhastighet

Här har en sekant ritats in.

Medelhastigheten mellan t1 = 0 s och t2 = 0,75 s är sekantens lutning: vmedel = Δs / Δt

Medelhastighet: Den genomsnittliga hastigheten under en tidsperiod. Se film!

https://www.youtube.com/watch?v=mklY86FZtm0 HYPERLINK "https://www.youtube.com/watch?


v=mklY86FZtm0&feature=youtu.be"& HYPERLINK "https://www.youtube.com/watch?
v=mklY86FZtm0&feature=youtu.be"feature=youtu.be

Momentanhastighet: Den hastighet som gäller vid en bestämd tidpunkt. Dra en tangent till s-t-grafen
och bestäm dess lutning. Se film!

https://www.youtube.com/watch?v=LkOgm6N0Vi0 HYPERLINK "https://www.youtube.com/watch?


v=LkOgm6N0Vi0&feature=youtu.be"& HYPERLINK "https://www.youtube.com/watch?
v=LkOgm6N0Vi0&feature=youtu.be"feature=youtu.be

Här har en tangent ritats in.

Momentanhastigheten i t = 0,75 s ges av tangentens riktning i denna tidpunkt.

Acceleration
När bilen står stilla är farten noll. När bilen startar ökar farten mer och mer. Man säger att bilen
accelererar. När bilen bromsar in och minskar farten, är rörelsen retarderad.

Inledningsvis studerar vi en vagn som rullar på ett plant bord respektive på ett lutande bord. Hur skiljer
sig rörelserna åt? Jo, på det plana bordet är hastigheten konstant, d v s rörelsen är likformig. På det
lutande planet ökar tydligt vagnens hastighet, d v s vagnen accelererar. Storheten acceleration (m/s2) är
alltså hastighetsändring per tidsenhet. Begreppet acceleration illustreras med ett par exempel.

Exempel:

En Mercedes ökar farten likformigt, dvs. med konstant acceleration, från stillastående till 35 m/s på 5 s.

a. Skissa v-t-grafen. b. Vad anger grafens lutning? c. Bestäm accelerationen.

d. Rita a-t-grafen till v-t-grafen.

Lösning:

Acceleration, forts
b. Lutningen a anger hur mycket farten ökar på en viss tid, d v s accelerationen. Storheten acceleration
har alltså enheten m/s2. Enheten på en grafs lutning blir alltid enheten på motsvarande y-axeln delat
med enheten på x-axeln.

c. Accelerationen a = Δv / Δt = (35 m/s) / (5 s) = 7 m/s 2

d. v-t-grafens lutning är konstant, d v s accelerationen är konstant.

Här ser vi ett mycket vanligt skrivsätt för en ändring i en storhet, nämligen den grekiska bokstaven Δ
(uttalas ”delta”). Δv betyder hastighetsändringen, dvs. sluthastigheten minus begynnelsehastigheten. I
klartext kan vi skriva och tänka:

SI-enheten för acceleration är alltså m/s2.

Fritt fall
Experimentellt kan accelerationen hos en fritt fallande kula beräknas. Resultatet presenteras i en v-t-graf.

Accelerationens karaktär studeras enklast i en v-t-graf (se ovan). Vi prickar alltså in tid och hastighet i
diagrammet, anpassar en linje till punkterna och konstaterar att lutningen, dvs accelerationen, är
konstant. Vi bestämmer accelerationen a:
Accelerationen hos ett fritt fallande föremål tycks alltså vara 9,7 m/s 2. I själva verket är
tyngdaccelerationen 9,82 m/s2 på våra breddgrader.

Fritt fall, forts


I det här avsnittet beskrivs vad som menas med tyngdacceleration och tyngdkraftens inverkan. Du får
också se exempel på luftmotståndets inverkan. Luftmotståndets inverkan är stor hos föremål som har
stor area i förhållande till massan, t.ex. löv och fjädrar. Luftmotståndet utnyttjas också av
fallskärmshoppare. Det går inte att beskriva luftmotståndets inverkan med enkla matematiska formler.
Därför kommer du i praktiken mest att studera situationer där luftmotståndet är så litet att det kan
försummas.

Alla föremål på jorden påverkas av tyngdkraften. Tyngdaccelerationen på jorden varierar lite på grund av
att jorden inte är riktigt klotformad men i Sverige är det 9,82 m/s 2.

Det är lätt att tro att olika föremål faller fortare ju tyngre de var. Experiment gjorda av bland andra
Galileo Galilei visar att om inte luftmotstånd finns, t.ex i vakuum, så faller olika kroppar samtidigt. Detta
beror på att de påverkas av tyngdkraften, större massa, större tyngdkraft, så att accelerationen blir
detsamma. Eftersom accelerationen som kropparna får beror på tyngdkraften, har den fått namnet
tyngdacceleration.

Lös uppgifterna 4.30 och 4.31 i läroboken.

Likformigt accelererad rörelse


Rörelse med konstant acceleration kallas också likformigt accelererad rörelse.

• Hastigheten ökar generellt när accelerationen är positiv.

• Hastigheten minskar generellt när accelerationen är negativ.

Med sluthastigheten först i uttrycket ser vi till att få ett positivt värde om hastigheten ökar resp. ett
negativt värde om den minskar. Glöm inte att hastighet är en fysikalisk storhet som har riktning, en s.k.
vektoriell storhet. Det är ju skillnad om hastigheten är riktad åt ena eller andra hållet även om själva
siffervärdet är detsamma.

På samma sätt betecknar Δt tidsändringen. Alltså om vi tänker på klockan så ändras ju tiden från ett
klockslag till ett annat under rörelsen. Vi har alltså en tidsändring.

I detta fall om vi delar en hastighetsändring med en tidsändring och har tiden uttryckt i SI-enheten s (dvs.
sekunder) blir ju bråkvärdet hastighetsändringen per sekund, dvs. per tidsenhet som definitionen säger.

Här precis som i många andra samband får man beräkningsmetoden nästan ”gratis” bara man håller reda
på enheten.
Formler i stället för graf
Fördelen med att använda v-t-grafer är att man får en grafisk bild av rörelsen, samt att det ökar
förståelsen för sambanden mellan sträcka, hastighet och acceleration. Ofta är det dock enklare att
enbart använda sig av formler. Vi skall därför utifrån en v-t-graf härleda några användbara formler, som
även finns med i formelsamlingen. Anta att en bil färdas med hastigheten v0. Vid tiden t=0 ökar farten
med den konstanta accelerationen a fram till tiden t, då bilens fart är v. Sedan t=0 har bilen då färdats
sträckan s. Vi ritar in förloppet i en v-t-graf.

Bilens fart v uttryckt i v0, a och t blir då: v(t) = v0 + at

Sträckan s, uttryckt i v, v0 och t blir på motsvarande sätt: s = t(v0 + v)/2

Det sista sambandet kan även skrivas s = (v0 + v0 + at)/2 vilket ger s = v0t + at2/2

De blåmarkerade formlerna finns i din formelsamling. Tänk igenom så att du förstår härledningarna till
dem!

Exempel på rörelseproblem
Med hjälp av rörelseformlerna kan du lösa många olika typer av problem som handlar om rörelse. Ofta
behöver du kombinera flera olika formler eller lösa ut någon storhet ur en formel.

Arbeta på följande sätt när du löser rörelseproblem:

• Lär dig rörelseformlerna i grundutförandet, så som de står i formelsamlingen.

• Lös ut den storhet som ska beräknas.

• Var noggrann när du definierar rörelseriktningen.

• Definiera en positiv rörelseriktning utifrån vad som är mest praktiskt. Ofta väljer man riktningen så att
starthastigheten blir positiv.

• Avgör sedan om accelerationen är positiv eller negativ.

Exempel:

Ett äpple faller utan begynnelsehastighet fritt i 3 s.

a. Bestäm äpplets sluthastighet. b. Hur långt faller äpplet?

Lösning:

Begynnelsehastigheten v0 = 0 m/s och accelerationen a = 9,82 m/s2 och tiden t = 3 s (i nästa avsnitt
kommer du att få läsa mer om "fritt fall", och då förklaras värdet 9,82 m/s 2).
a. v = v0 + at = 0 + 9,82 · 3 m/s = 29,46 m/s ≈ 29 m/s

b.

Fler exempel på rörelseproblem


Exempel:

En bil färdas med hastigheten 90 km/h. Bilen bromsar hårt så att accelerationen (retardationen) är 7,5
m/s2.

a) Hur lång tid tar det innan bilen stannar?

b) Beräkna bromssträckan.

Lösning:

Härledningar av rörelseformler
Vi använder alltså följande formler:

Se också en film om härledningarna av dessa formler:

https://www.youtube.com/watch?v=Xbf_ETED2os HYPERLINK "https://www.youtube.com/watch?


v=Xbf_ETED2os&feature=youtu.be"& HYPERLINK "https://www.youtube.com/watch?
v=Xbf_ETED2os&feature=youtu.be"feature=youtu.be

Lös uppgifterna 4.12 - 4.29 i din lärobok.

Gör sedan Testa-dig-själv 1:2!

Lektioner, avsnitt 3
Videolektioner där läraren behandlar begreppet rörelse, som är en del av fysikkursen Fysik 1 i gymnasiet.
Han har först genomgång av grundläggande begrepp sedan visar han rikligt med exempeluppgifter.
Läraren går igenom begreppen hastighet, acceleration, momentanhastighet, momentanacceleration,
medelhastighet, medelacceleration, tyngdacceleration och visar hur man kan illustrera detta grafiskt.
Han går även igenom hastighet och sträckaformlerna samt gör grafiska härledningar av vissa och gör
exempeluppgifter med rörelser i både en och två dimensioner.

OBS! Dessa filmer är långa, och meningen är att du skall titta på de som ett led i din repetition - dels nu
och dels vid slutet av kursen. Du kan mycket väl titta på några delar åt gången - du behöver alltså inte se
hela vid ett och samma tillfälle.

https://www.youtube.com/watch?v=Dzcs-adFwR0 HYPERLINK "https://www.youtube.com/watch?


v=Dzcs-adFwR0&feature=youtu.be"& HYPERLINK "https://www.youtube.com/watch?v=Dzcs-
adFwR0&feature=youtu.be"feature=youtu.be

https://www.youtube.com/watch?v=7w08_tI-vrs HYPERLINK "https://www.youtube.com/watch?


v=7w08_tI-vrs&feature=youtu.be"& HYPERLINK "https://www.youtube.com/watch?v=7w08_tI-
vrs&feature=youtu.be"feature=youtu.be

Lös och reflektera över kontrolluppgifterna 1 - 6 i kap 4 i din lärobok!

Summering, avsnitt 3

Repetera lärobokens lösta Exempel 1 - 7 så att du känner dig trygg med lösningsprocedur och begrepp.
Genomför också uppgiften Experimentella övningar 4.32, sid 96.

Sedan tar du itu med Uppdrag 1. Uppdragen bedöms av din webblärare. Tänk på att nivån på hela
lösningen bedöms - inte enbart svaret.

Sammanfattning
I den här modulen har du jobbat med:

*) Inledande reflektioner över vad ämnet fysik är och dess historia, samt vad det kan användas till

*) Mätning och mätnoggrannhet. Massa, volym och densitet

*) Rörelse - begreppen sträcka, hastighet och acceleration. Fritt fall.

Det du nu skall göra är att repetera hela kapitlet (lös gärna fler övningar i din lärobok). Därefter gör du
TDS 1:3, som innehåller uppgifter på hela denna modul. Eventuellt kan det sedan behövas ytterligare lite
repetition. När du tycker att du behärskar samtliga moment i denna modul gör du Uppdrag 1. Detta
skickar du in till din webblärare via datorn, och denne rättar och kommenterar det.

Därefter väntar modul 2, som kommer att behandla

begreppen krafter, energi och rörelsemängd. Du kan alltid gå tillbaka till den här studieguiden vid behov.

Uppdrag, bedömning och kunskapskrav


Uppdragen är en viktig del i kunskapsinhämtningen. Det är därför viktigt att du är noggrann när du
arbetar med uppdragen, och att du är säker på att du förstår och behärskar det du skickar in. Vid den
Avslutande examinationen kommer du att få visa just detta.

Hur mycket arbete du ska lägga ner på svaren styrs bl a av din betygs- ambition, dina förkunskaper, hur
snabbt du läser kursen etc.

Jobba för att hålla studieplanen och de inlämningsdatum som finns i Novo. Risken är annars att du får för
mycket att göra i slutet av kursen. Kom också ihåg att betyget sätts efter den Avslutande examinationen.
Det är meningen att du skall utvecklas och lära dig under kursens gång.

Uppgifterna i uppdragen är uppdelade i olika betygsnivåer (E, C och A). Efter varje fråga anges vilken nivå
den ligger på.

Läs om bedömning och betyg i det svarta fältet. Vi som är lärare i kursen vill att du skall nå ditt mål. Vi
skall också säkerställa att du når Skolverkets uppsatta kunskapskrav. Vill du repetera kunskapskraven för
kursen? Tryck då på ikonen så kommer du direkt till Skolverkets hemsida.
https://www.skolverket.se/undervisning/vuxenutbildningen/komvux-gymnasial/laroplan-for-vux-och-
amnesplaner-for-komvux-gymnasial/amne?url=1530314731%2Fsyllabuscw%2Fjsp%2Fsubject.htm
%3FsubjectCode%3DFYS%26courseCode%3DFYSFYS01a%26lang%3Dsv%26tos%3Dgy%26webtos
%3Dvuxgy%26p%3Dp HYPERLINK "https://www.skolverket.se/undervisning/vuxenutbildningen/komvux-
gymnasial/laroplan-for-vux-och-amnesplaner-for-komvux-gymnasial/amne?
url=1530314731%2Fsyllabuscw%2Fjsp%2Fsubject.htm%3FsubjectCode%3DFYS%26courseCode
%3DFYSFYS01a%26lang%3Dsv%26tos%3Dgy%26webtos%3Dvuxgy%26p
%3Dp&sv.url=12.b173ee8160557dd0b8100d"& HYPERLINK
"https://www.skolverket.se/undervisning/vuxenutbildningen/komvux-gymnasial/laroplan-for-vux-och-
amnesplaner-for-komvux-gymnasial/amne?url=1530314731%2Fsyllabuscw%2Fjsp%2Fsubject.htm
%3FsubjectCode%3DFYS%26courseCode%3DFYSFYS01a%26lang%3Dsv%26tos%3Dgy%26webtos
%3Dvuxgy%26p
%3Dp&sv.url=12.b173ee8160557dd0b8100d"sv.url=12.b173ee8160557dd0b8100d#anchor_FYSFYS01a

Dina resultat under kursens gång samlas i en matris. Matrisen visar i vilken utsträckning du uppnått de
olika kunskapskraven i kursen. Matrisen, tillsammans med lärarens kommentarer, vägleder dig i vad du
behöver träna mer på för att nå ditt betygsmål. Efter den avslutande examinationen kommer matrisen
justeras en sista gång och därmed utgöra en del av betygsmotiveringen. Kom ihåg att nivån du visat
under kursens gång också behöver visas under den avslutande examinationen.

Studieguide 2

En översikt
I din andra modul kommer du att få lära dig mycket om krafter och energi. Du kommer också att få stifta
bekantskap med ett begrepp som troligen är nytt för dig - rörelsemängd.

Klicka nu på äpplet för att få en konkret målbeskrivning!

Här får du en målbeskrivning, så att du kan få en uppfattning om vad ditt studium av den andra modulen
ska leda fram till.
När du har gått igenom de tre första kapitlen, Newtons lagar, Energi och Rörelsemängd, ska du:

• kunna använda Newtons tre lagar

• vara bekant med energibegreppet

• kunna skilja på begreppen arbete, potentiell energi och rörelseenergi, samt kunna använda dessa
vid problemlösning

• förstå begreppet effekt och kunna beräkna denna

• förstå vad friktion är och kunna använda detta begrepp

• kunna använda dig av begreppen rörelsemängd och impuls

Studietips
Arbeta med studieguiderna här i Novo parallellt med att du läser och löser uppgifter i din lärobok.

Var aktiv, titta på webblektioner och gör interaktiva övningar här i Novo, gör Testa-dig-själv och andra
övningar.

Arbeta aktivt med ditt betygsmål. Både uppgifterna i boken och i uppdragen här i Novo är nivåuppdelade
(E-, C- och A-nivå). Koncentrera dig på den nivå som är ditt mål!

Utnyttja fysikhjalp@hermods.se

Att läsa i din lärobok


Krafter i vardagen, s 19 - 42

Energi och arbete, s 97 - 126

Kraft och rörelse, s 245 - 280

Inledning
Denna studieguide omfattar kapitel 2, 5 och 11 i läroboken.

I denna modul kommer du att få stifta bekantskap med följande begrepp:

Newtons lagar

Energibegreppet

Arbete

Potentiell energi

Rörelseenergi
Acceleration

Friktion

Rörelsemängd

Impuls

När modulen är klar skall du sända in ditt andra uppdrag. Laboration nr 1 och nr 2 skall bifogas Uppdrag
2. Instruktioner till dessa kommer längre fram i denna modul.

Krafter
Läs sidorna 19 - 39 som beskriver de vanligaste krafterna i vardagen innan du fortsätter med
instuderingen här på Novo.

Tidigare fick du lära dig litet om hur längdenheten 1 meter och tidsenheten 1 sekund tidigare
definierades. De moderna definitioner som används idag är egentligen svåra att förstå för "vanliga"
människor, även för den som läser fysik på gymnasienivå.

I kapitel 2 har du också fått lära dig att 1 kilogram fortfarande definieras med hjälp av den
kilogramprototyp som tillverkades 1889. Vad är det som gör att just definitionen av massa inte behöver
förbättras?

Det mesta i vårt dagliga liv påverkas av krafter; vi kan gå på jorden utan att ramla eller flyga av tack vare
tyngdkraften. Atomkärnan hålls ihop av starka kärnkraften och slits isär av den elektriska kraften. När
neutronen i atomkärnan byter skepnad och förvandlas till en proton och en elektron, då är den svaga
kärnkraften som verkar. Dessa fyra olika krafter kallas för de fundamentala krafterna i naturen. Vad dessa
krafter egentligen är vet vi inte, vi vet bara att de finns.

Gravitation
Ofta säger man vikt och menar ibland massa men ibland tyngd.

I fysiken undviker vi benämningen vikt just för att det inte är riktigt klart vad som menas. I stället
använder vi begreppen och storheterna massa resp. kraft. Tyngd är ett exempel på kraft.

Det är framöver mycket viktigt att du lär dig skilja på dessa begrepp. Massa är en egenskap som är
knuten till mängden materia, en inre egenskap hos ett föremål. Man kan mäta massor med hjälp av en
balansvåg. Två föremål har samma massa när de balanserar varandra på en balansvåg. Balansvågen ger
samma utslag även om den flyttas till månen. En fjädervåg, t.ex. en badrumsvåg mäter inte massan, utan
tyngden. Den kommer att ge olika utslag på jorden och på månen beroende på att tyngden varierar. Du
får lära dig mer om detta längre fram i kapitlet.

Massan är en egenskap hos ett föremål. Volymen är en annan egenskap. Varken massan eller volymen
kan var för sig, beskriva varför en sten sjunker i vatten, medan en träbit flyter. För att kunna beskriva
denna skillnad hos ämnena inför man, som du redan sett, begreppet densitet tidigt i fysikstudierna.
Vad är en kraft?
En kraft beskriver hur en kropp påverkas utifrån. En bok som ligger på bordet påverkar bordet med en
kraft. Bordet påverkar också kroppen med en kraft. När du ligger i sängen påverkar du sängen med en
kraft som gör att den mjuka madrassen trycks ihop. Andra exempel på kraftpåverkan är när bilen
accelererar med hjälp av motorns dragkraft, eller månens bana runt jorden på grund av gravitationen.

OBS! En kraft har alltid både storlek och riktning. En storhet som har både storlek och riktning kallas en
vektor.

Krafter brukar betecknas med F, men även andra beteckningar förekommer för speciella typer av krafter.
Krafter mäts i SI-enheten Newton, 1 N.

Vi ska studera några ofta förekommande kraftsituationer och speciellt titta på hur krafterna betecknas
och ritas.

Exempel 1: En låda står på golvet. Det verkar två krafter på lådan, en normalkraft, FN, och tyngdkraften,
Fg.

Tyngdkraften från jorden verkar på alla kroppar med massa. Den utgår ifrån kroppens tyngdpunkt och är
alltid riktad rakt neråt.

En normalkraft verkar från underlaget, i det här fallet från golvet.

Normalkraften är alltid vinkelrät mot underlaget. Normalkraften brukar ibland betecknas med N, vilket
inte ska blandas ihop med enheten Newton.

Parallella krafter
Exempel 2: En lampa är upphängd i två vertikala linor. Det verkar tre krafter på lampan, två spännkrafter,
Fs, från linorna och tyngdkraften, Fg.

En spännkraft verkar i t.ex. en lina. Spännkraften utgår ifrån linans fästpunkt och verkar i linans riktning.
När flera krafter verkar på ett föremål kan man bestämma den resulterande kraften, kraftresultanten.
Exempel 3 nedan visar hur du kan resonera när du ska bestämma kraftresultanten.

Exempel 3: Två krafter, F1 = 18 N och F2 = 35 N, verkar på en låda, se figur. Bestäm resultantens storlek och
riktning.

Resultantens storlek blir: FR = F2 - F1


Resultanten är alltså här differensen av de två krafterna.

Om krafterna varit parallella och riktade åt samma håll skulle resultanten vara summan av de två
krafterna. Titta på filmen om krafter och resultanter!

https://www.youtube.com/watch?v=jqOKnQQbCRQ HYPERLINK "https://www.youtube.com/watch?


v=jqOKnQQbCRQ&list=PLvQzFdt9swYqcuWv--nhxnTOTOobnzIiV&feature=share&index=6"& HYPERLINK
"https://www.youtube.com/watch?v=jqOKnQQbCRQ&list=PLvQzFdt9swYqcuWv--
nhxnTOTOobnzIiV&feature=share&index=6"list=PLvQzFdt9swYqcuWv--nhxnTOTOobnzIiV HYPERLINK
"https://www.youtube.com/watch?v=jqOKnQQbCRQ&list=PLvQzFdt9swYqcuWv--
nhxnTOTOobnzIiV&feature=share&index=6"& HYPERLINK "https://www.youtube.com/watch?
v=jqOKnQQbCRQ&list=PLvQzFdt9swYqcuWv--nhxnTOTOobnzIiV&feature=share&index=6"feature=share
HYPERLINK "https://www.youtube.com/watch?v=jqOKnQQbCRQ&list=PLvQzFdt9swYqcuWv--
nhxnTOTOobnzIiV&feature=share&index=6"& HYPERLINK "https://www.youtube.com/watch?
v=jqOKnQQbCRQ&list=PLvQzFdt9swYqcuWv--nhxnTOTOobnzIiV&feature=share&index=6"index=6

Mer om parallella krafter


• Rita alltid ut resultanten streckad i figuren, så det tydligt framgår att det är enkraftresultant och
inte ytterligare en kraft.

• Rita också alltid en figur som visar krafternas och kraftresultantens inbördes storlek och riktning.
Då framgår det tydligt vad som gäller, och risken för fel och missförstånd är inte lika stor.

Man kan, i beräkningssammanhang, tänka sig att resultanten kan ersätta de krafter den motsvarar.
Resultanten har alltså precis samma verkan på föremålet i fråga som de verkliga krafterna tillsammans.
Om resultanten är noll råder oftast s.k. jämvikt som vi ska studera lite längre fram.

På en massa på jorden verkar alltid en tyngdkraft, med angreppspunkt i massans tyngdpunkt, riktad mot
jordens centrum, samt en normalkraft riktad vinkelrätt mot bordet/underlaget och med angreppspunkt i
gränsytan mellan massan och underlaget.

Klicka på symbolerna nedan för att se en genomgångar av olika krafter!

https://www.youtube.com/watch?v=6aUxjTbWPJg HYPERLINK "https://www.youtube.com/watch?


v=6aUxjTbWPJg&feature=share&list=PLvQzFdt9swYqcuWv--nhxnTOTOobnzIiV&index=7"& HYPERLINK
"https://www.youtube.com/watch?v=6aUxjTbWPJg&feature=share&list=PLvQzFdt9swYqcuWv--
nhxnTOTOobnzIiV&index=7"feature=share HYPERLINK "https://www.youtube.com/watch?
v=6aUxjTbWPJg&feature=share&list=PLvQzFdt9swYqcuWv--nhxnTOTOobnzIiV&index=7"& HYPERLINK
"https://www.youtube.com/watch?v=6aUxjTbWPJg&feature=share&list=PLvQzFdt9swYqcuWv--
nhxnTOTOobnzIiV&index=7"list=PLvQzFdt9swYqcuWv--nhxnTOTOobnzIiV HYPERLINK
"https://www.youtube.com/watch?v=6aUxjTbWPJg&feature=share&list=PLvQzFdt9swYqcuWv--
nhxnTOTOobnzIiV&index=7"& HYPERLINK "https://www.youtube.com/watch?
v=6aUxjTbWPJg&feature=share&list=PLvQzFdt9swYqcuWv--nhxnTOTOobnzIiV&index=7"index=7

https://www.youtube.com/watch?v=UyVylH1caoM HYPERLINK "https://www.youtube.com/watch?


v=UyVylH1caoM&list=PLvQzFdt9swYqcuWv--nhxnTOTOobnzIiV&feature=share&index=5"& HYPERLINK
"https://www.youtube.com/watch?v=UyVylH1caoM&list=PLvQzFdt9swYqcuWv--
nhxnTOTOobnzIiV&feature=share&index=5"list=PLvQzFdt9swYqcuWv--nhxnTOTOobnzIiV HYPERLINK
"https://www.youtube.com/watch?v=UyVylH1caoM&list=PLvQzFdt9swYqcuWv--
nhxnTOTOobnzIiV&feature=share&index=5"& HYPERLINK "https://www.youtube.com/watch?
v=UyVylH1caoM&list=PLvQzFdt9swYqcuWv--nhxnTOTOobnzIiV&feature=share&index=5"feature=share
HYPERLINK "https://www.youtube.com/watch?v=UyVylH1caoM&list=PLvQzFdt9swYqcuWv--
nhxnTOTOobnzIiV&feature=share&index=5"& HYPERLINK "https://www.youtube.com/watch?
v=UyVylH1caoM&list=PLvQzFdt9swYqcuWv--nhxnTOTOobnzIiV&feature=share&index=5"index=5

Simulering
Man använder en dynamometer för att mäta krafter.

Vi hänger några olika vikter i en dynamometer och antecknar massa och kraft i en värdetabell och
kommer med hjälp av ett diagram fram till följande samband mellan massa och tyngd

F = m · g , där g= 9,82 N/kg (tyngdfaktorn).

Upptäck de krafter som verkar när du försöker skjuta på ett arkivskåp. Skapa en pålagd kraft och se den
resulterande friktionskraften och totalkraften som verkar på skåpet. Diagrammet visar krafter, läge,
hastighet och acceleration kontra tid. Visa en fri kropps diagram av alla krafter (inklusive gravitations- och
normalkrafter). Titta också på klippet om gravitationskraft och tyngd.

Klicka på bilden nedan för att köra simuleringen! Titta också på filmen.

http://phet.colorado.edu/en/simulation/forces-1d

https://www.youtube.com/watch?v=BeeZ5Ao1IEk HYPERLINK "https://www.youtube.com/watch?


v=BeeZ5Ao1IEk&feature=share&list=PLvQzFdt9swYqcuWv--nhxnTOTOobnzIiV&index=3"& HYPERLINK
"https://www.youtube.com/watch?v=BeeZ5Ao1IEk&feature=share&list=PLvQzFdt9swYqcuWv--
nhxnTOTOobnzIiV&index=3"feature=share HYPERLINK "https://www.youtube.com/watch?
v=BeeZ5Ao1IEk&feature=share&list=PLvQzFdt9swYqcuWv--nhxnTOTOobnzIiV&index=3"& HYPERLINK
"https://www.youtube.com/watch?v=BeeZ5Ao1IEk&feature=share&list=PLvQzFdt9swYqcuWv--
nhxnTOTOobnzIiV&index=3"list=PLvQzFdt9swYqcuWv--nhxnTOTOobnzIiV HYPERLINK
"https://www.youtube.com/watch?v=BeeZ5Ao1IEk&feature=share&list=PLvQzFdt9swYqcuWv--
nhxnTOTOobnzIiV&index=3"& HYPERLINK "https://www.youtube.com/watch?
v=BeeZ5Ao1IEk&feature=share&list=PLvQzFdt9swYqcuWv--nhxnTOTOobnzIiV&index=3"index=3

Icke-parallella krafter
Läs sidorna 245 - 252 i läroboken.

Vi ska också studera några exempel med krafter som inte är parallella. Då kan vi inte bara addera eller
subtrahera krafterna, utan vi måste också at hänsyn till riktningen när vi bestämmer resultantkraften.
Exempel 4: Bestäm resultantkraften till F1 och F2 i figuren här intill.

Resultantkraften ska ha samma verkan som F1 och F2 tillsammans.

När du bestämmer resultantkraften till F1 och F2 måste du som sagt at hänsyn till krafternas riktningar.
Gör på följande sätt:

Rita två hjälplinjer som är parallella med F 1 respektive F2 och som går genom spetsen på krafterna. Dessa
hjälplinjer är streckade i figuren. Tillsammans med F 1 och F2 bildar de en parallellogram. Nu kan du rita
resultantkraften, FR. Den utgår från krafternas startpunkt och slutar där hjälplinjerna skär varandra, som
en diagonal i parallellogrammen.

Komposantuppdelning
I praktisk problemlösning kan det vara enklare att använda komposantuppdelning för att bestämma
krafter och resultantkrafter. Exempel 5 nedan visar hur du komposantuppdelar en enkel kraft i två mot
varandra vinkelräta riktningar:

Exempel 5: Kraften F är riktad snett uppåt som figuren visar. Tänk dig att du placerar kraften i ett
koordinatsystem med startpunkten i origo. Då kan du bestämma x- och y-koordinaterna för kraften.
Dessa kallas för kraftens komposanter i x-led och y-led, och betecknas Fx respektive Fy, se figuren här
intill. Fx, Fy och F kan tillsammans bilda en rätvinklig triangel. Om vinkeln mellan F och x-axeln är α, kan
du bestämma komposanternas storlek med hjälp av trigonometri.

Sambandet mellan Fx och Fy bestäms av Pythagoras sats:

Rita alltid komposanterna streckade, som i figuren ovan, så att det tydligt framgår att det inte är fler
krafter som verkar. Du kan också betrakta F som en resultant till Fx och Fy. Komposanterna har ju
tillsammans samma verkan som den ursprungliga kraften, F. Studera noga bokens exempel om krafter.
Läs igenom Exempel 1 och 2, s 247 och 249, i läroboken. (Analytisk metod med cosinussatsen förutsätter
Matematik 3c.)

Addition av vektorer

Polygonmetoden är att föredra när vi ska addera fler än två vektorer.


Klicka på bilden nedan för att göra en interaktiv övning!

http://tube.geogebra.org/student/m16948

Övningar på komposantuppdelning
I många fall stöter vi på problem då vi måste dela upp krafter i deras komposanter istället för att sätta
ihop komposanterna till en resultant.

Titta på två videolektioner om komposantuppdelning genom att klicka på bilderna nedan!

https://www.youtube.com/watch?v=1L5OFgsp7lE

https://www.youtube.com/watch?v=JMg7u8Al8lE HYPERLINK "https://www.youtube.com/watch?


v=JMg7u8Al8lE&feature=youtu.be"& HYPERLINK "https://www.youtube.com/watch?
v=JMg7u8Al8lE&feature=youtu.be"feature=youtu.be

Utforska komposantuppdelning då du kan ändra parametrarna själv! Klicka på symbolerna, så får du


två interaktiva övningar att jobba med!

http://tube.geogebra.org/student/m4861

https://www.geogebra.org/m/VYDSx3UU

Newtons lagar
Newtons lagar handlar om samband mellan kraft, massa och rörelse. De kan tillämpas i många olika
situationer, t.ex. när du väger frukt i affären eller när en bil accelererar. Ibland kan det kännas som om
Newtons lagar strider mot dina erfarenheter i verkliga situationer. Vi ska försöka reda ut begreppen i det
här kapitlet, och du kommer att lära dig att beskriva verkliga situationer och lösa problem med hjälp av
Newtons lagar.

Läs mer om Isak Newton i läroboken, s 270. Givetvis kan du hitta mycket mer om hans bakgrund och
historia, där https://sv.wikipedia.org/wiki/Isaac_Newton kan vara en utgångspunkt.
Newtons första lag
Läs s 254-256 i läroboken.

Kraften som verkar mellan två kroppar beskrivs av Newtons gravitationslag (1687): I ett system som
exempelvis en galax med miljarder individuella stjärnor är kraften på en kropp lika med summan av
krafterna från alla ingående stjärnor (och annat). Newton fann också tre andra lagar som vi påverkas av
dagligen:

Ett föremål som befinner sig i vila eller likformig rörelse behåller detta tillstånd så länge kraft-
resultanten är noll eller inga yttre krafter verkar på den. Med likformig rörelse menas att föremålet rör
sig med konstant fart och riktning. För att ändra farten eller ändra riktning krävs en kraft.

När åker bil måste bilen använda en kraft för att åka fram. Åker bilen med jämn fart på en rak väg, räcker
det med en kraft som är lika stor som kraften från luftmotstånd och all friktion. Om du vill öka farten,
måste du gasa på. Ju mer du gasar, desto större blir accelerationen.

Det har också betydelse hur stor massa som ska accelereras. Det krävs en stor kraft för att accelerera en
lastbil, jämfört med den som behövs för en personbil.

Även för att minska farten behövs en kraft, t. ex friktionskraften mellan bromsskivor och klossar. Kraft
behövs också för att ändra rörelseriktning, t. ex när man sparkar på en boll.

Detta brukar sammanfattas i den så kallade tröghetslagen (= Newtons första lag):

• Det behövs alltid en kraft för att ändra ett föremåls fart eller rörelseriktning.

• Trögheten gör att ett föremål som är stillastående förblir stillastående, om det inte påverkas av någon
kraft.

Newtons första lag, tröghetslagen, säger att ett föremåls hastighet förblir oförändrad om kraftresultanten
på föremålet är noll. Skälet är att massan är trög och inte vill förändra sitt rörelsetillstånd.

Newtons första lag, exempel


Vi tillämpar tröghetslagen:

Exempel 5:

Rita ut kraftresultanten F på bollen med den momentana farten v i nedanstående situationer:

a) En boll som rör sig uppåt från marken.

b) En boll faller fritt nedåt mot marken.

c) En boll som rullar friktionsfritt på marken.


Fundera en stund innan du tittar på lösningen (blå ikonen)!

Lösning:

Tyngdkraften är den enda kraft som påverkar bollen. Kraftresultanten är alltså riktad nedåt. Det spelar
alltså ingen roll åt vilket håll bollen rör sig.

Princip: Kraftresultanten på ett föremål är riktad åt det håll som farten ökar åt, eller åt motsatt håll som
farten minskar åt.

Lös uppgift 11.11 - 11.13 på s 276 i läroboken.

Jämvikt
Vi ska börja med att studera den enklaste tillämpningen av Newtons första lag, nämligen när en kropp
ligger stilla. Då säger vi att kroppen är i s.k. jämvikt. Vid jämvikt är den resulterande kraften lika med noll.
Det innebär att ett föremål befinner sig i vila eller i likformig rörelse.

Exempel 6: Vi studerar lådan från exempel 1. Två krafter, tyngdkraften F g och normalkraften FN, verkar på
lådan. Eftersom lådan ligger stilla är den i jämvikt. Krafterna F g och FN är motriktade och lika stora.

När du ska analysera vilka krafter som verkar i en viss situation, är det ofta lämpligt att frilägga det
intressanta föremålet. Vad som menas med friläggning får du lära dig i nästa exempel.

Exempel 7: Två likadana böcker ligger ovanpå varandra på ett bord, se figuren ovan. Vilka krafter verkar
på den understa boken? Varje bok har tyngden Fg.

Vi frilägger den understa boken genom att bara rita den boken och de krafter som verkar på den. På så
sätt renodlar vi situationen och riskerar inte att blanda ihop krafter som verkar på olika föremål. Boken
har tyngden Fg, som verkar från tyngdpunkten rakt neråt. Boken påverkas också av en kraft från den
översta boken. Den kraften är lika med den översta bokens tyngd, Fg. Vi ritar den kraften utgående ifrån
bokens ovansida. Från bordet verkar en uppåtriktad normalkraft, F N. Eftersom vi har jämvikt är FN lika
med summan av de två andra krafterna: F N = 2Fg.

Jämvikt, forts
Exempel 8: Här ska vi studera kraftjämvikt med hjälp av komposantuppdelning. En lampa med massan
7,5 kg är upphängd mitt på en lina, som figuren visar. Hur stora är spännkrafterna i linan?

Vi har kraftjämvikt. Det betyder att den resulterande kraften är noll. Eftersom lampan hänger i mitten av
linan och linans är symmetriskt vinklad åt båda håll, kommer spännkrafterna att vara lika stora åt båda
hållen. Vi måste at hänsyn till riktningarna när vi studerar kraftjämvikten. Det gör vi enklast genom att
komposantuppdela F i horisontell och vertikal riktning på samma sätt som beskrivs i exempel 5. Vi får då:

Detta ger:

Nu kan vi studera x- och y-riktningarna var för sig. Vi ska ha jämvikt i både x- och y-led. Vi har ritat
kraftpilar utgående från punkten på linan där lampan hänger. Komposanterna i x-led är lika stora men
motriktade. Därför räcker det att studera jämvikten i y-led. Vi får följande jämviktsvillkor:

2 · Fy = m · g vilket ger

Eftersom Fy = F · sin 18° kan vi bestämma F:

Spännkrafterna är 120 N i linan. Värt att notera är att om vi bara ser på linan så är ju även den i jämvikt,
vilket innebär att i linan finns då alltid två krafter, en åt vardera hållet som håller varandra i jämvikt.
Enklare uttryckt är att om man drar i en lina med en viss kraft kommer linans fästpunkt i andra änden att
hålla emot med en lika stor kraft. Om inte fästet brister förstås.

Newtons andra lag


Ett föremål som utsätts för en kraft accelererar. Formeln definierar kraftenheten 1 N, d.v.s. kraften som
ger massan 1 kg accelerationen 1 m/s2 kallas 1 N. Accelerationen som verkar på en kropp är
proportionell mot kraften som verkar på den.

Läs s 258 - 261 i läroboken.

Newtons andra lag, ”Kraftlagen”, ger alltså ett samband mellan kraft, massa och acceleration. Den kan
tillämpas i alla situationer där vi studerar accelererad rörelse.

En bil som accelereras påverkas av en resulterande kraft. Hur stor accelerationen blir beror av kraftens
storlek och bilens massa enligt Newtons andra lag: F = m · a. Detta samband brukar kallas kraftlagen.
Observera att det är den resulterande kraften som avses.

Enheten 1 Newton får sin definition med hjälp av kraftlagen: 1 N = 1 kg · m/s2

I förra kapitlet fick du lära dig att fritt fallande föremål påverkas av tyngdaccelerationen, g = 9,82 m/s2.
Med kraftlagen kan du nu beräkna tyngdkraften: F = m · g (Ofta betecknas tyngdkraften bara med mg.)

Exempel 9: En låda har massan 3,0 kg. Beräkna tyngden.

Lösning: F = m · g = 3,0 · 9,82 N ≈ 29 N

Exempel 10: En raket med massan 18 ton skjuts iväg rakt uppåt. Motorns dragkraft är 250 kN.

a) Bestäm raketens acceleration. Du kan anta att accelerationen är konstant (likformigt

accelererad rörelse).

b) Vilken hastighet kommer raketen att ha uppnått 10 s efter start?

Lösning: Försök lösa uppgiften själv. Kontrollera sedan din lösning här:

a) De krafter som verkar på raketen är motorns dragkraft och tyngdkraften.

Raketens tyngd är mg = 18 · 103 · 9,82 N = 176760 N

Den resulterande kraften är F = Fdrag - mg = 250 · 103 - 176760 = 73240 N

Accelerationen bestäms med hjälp av kraftlagen:

b) Hastigheten kan bestämmas med hjälp av rörelselagarna. Vid likformigt accelererad rörelse gäller:

v = v0 + a · t Med v0 = 0 blir v = a · t = 4,1 · 10 = 41 m/s

Titta på Exempel 6 och 7 sid 259 i läroboken, samt lös Kontroll 6, sid 261.

Sammanfattning
*) En kraft ger upphov till en accelererad rörelse enligt Newtons andra lag: F = m · a

*) Om kraften beror på jordens tyngdkraft på föremålet blir formeln: F = m · g

*) Ett föremål är i vila om de resulterande krafterna på föremålet är noll. Föremålet kan då vara stilla
eller röra sig med konstant hastighet.

*) R = F1 + F2 + F3 + F4 + ... i respektive riktning

Är du tyngdlös i en hiss? Titta på filmen!

https://www.youtube.com/watch?v=2k-2IlJcaHg HYPERLINK "https://www.youtube.com/watch?v=2k-


2IlJcaHg&feature=youtu.be"& HYPERLINK "https://www.youtube.com/watch?v=2k-
2IlJcaHg&feature=youtu.be"feature=youtu.be

Newtons tredje lag


När man avfyrar en kanon kastas kanonen bakåt medan kulan flyger framåt. Det kallas rekyl. Man säger
att reaktionskraften är lika stor som kraften och riktad åt motsatt håll. När man slår huvudet i väggen för
att uppdragen i fysik är för svåra, får man ont för att väggen slår tillbaka med lika stor kraft.

Hittills har vi studerat krafterna på ett föremål. Vi har beräknat den resulterande kraften på ett föremål
som accelereras och vi har studerat föremål i jämvikt. I det här avsnittet ska vi studera hur två olika
föremål påverkar varandra med krafter. Det beskrivs i Newtons tredje lag.

Läs s 37 - 38 i läroboken. Enligt Newtons tredje lag påverkar två föremål varandra med lika stora,
motriktade krafter. Till varje kraft finns det alltså en motkraft. Kraft och motkraft är lika stora, men
motsatt riktade. En lampa är upphängd i en kabel. Lampan påverkas av en uppåtriktad kraft från kabeln.
Samtidigt påverkas kabeln av en lika stor neråtriktad motkraft från lampan.

Ett speciellt intressant exempel på Newtons tredje lag beskrivs med hjälp av Newtons gravitationslag.
Enligt denna lag påverkar alla kroppar varandra med lika stora, attraherande krafter som beror av
kropparnas massor och avståndet mellan dem. Vi kan uppskatta hur stor attraktionskraften är mellan två
personer på 25 cm avstånd från varandra. Anta att varje person har massan 100 kg. Newtons
gravitationslag ger:

Denna kraft är - som du ser - mycket liten.

På nästa sida ska vi beräkna kraften mellan jorden och en människa som står på marken.

Newtons tredje lag, forts


Vi beräknar kraften mellan jorden och en människa som står på marken och får följande resultat:

Vi antar att människan väger 100 kg.

Data om jorden hittar du i din formelsamling. Jordens massa är 5,977 · 10 24 kg.

Hur stort är avståndet mellan kropparna? För att förenkla beräkningarna antar vi att hela Jordens massa
är koncentrerad till dess tyngdpunkt. Avståndet mellan kropparna är då lika med jordens radie. Jorden
har större radie vid ekvatorn än vid polerna, men vi kan beräkna "medelradien" med hjälp av data som
finns i formelsamlingen och får 6367,75 km. Kraftens storlek blir då:

Som du ser är kraften lika med tyngden av en 100 kg kropp.


Observera att konstanten G i Newtons gravitationslag är en "fysikalisk konstant", gravitationskonstanten.
Den betecknar inte tyngdkraften. Du hittar en sammanställning av de viktigaste fysikaliska konstanterna i
formelsamlingen.

Lös uppgift 11.15 och 11.17 på s 276 i läroboken.

Laboration
Visualisera gravitationskraften som två föremål utövar på varandra. Ändra egenskaper av föremålen för
att se vad som händer. Klicka på bilden till vänster! Titta även på filmen väga jorden.

http://phet.colorado.edu/en/simulation/gravity-force-lab

https://www.youtube.com/watch?v=9yMQM5AN8mc HYPERLINK "https://www.youtube.com/watch?


v=9yMQM5AN8mc&feature=youtu.be"& HYPERLINK "https://www.youtube.com/watch?
v=9yMQM5AN8mc&feature=youtu.be"feature=youtu.be

Sammanfattning, krafter
Läs sammanfattningarna på s 39 och s 273 i kursboken. Du skall behärska följande begrepp:

*) Kraft som vektoriell storhet, kraftenheter

*) Kraft och resultant

*) Addering av krafter som har olika riktningar

*) Studera kraftjämvikt med hjälp av komposantuppdelning

*) Kraftjämvikt

Du ska kunna följande lagar:

*) Newtons första lag, tröghetslagen

*) Newtons andra lag, kraftlagen

*) Newtons tredje lag, lagen om aktion och reaktion

*) Newtons gravitationslag

Gör övning 2.1 - 2.22 på s 39 samt övningarna 11.1 - 11.6, 11.9, 11.18, 11.24, 11.25 och 11.29

på s 274 i kursboken. Titta också på filmerna från Khan Academy ovan. I den högra finner du dessutom
automaträttade tester.

https://www.youtube.com/watch?v=NYVMlmL0BPQ HYPERLINK "https://www.youtube.com/watch?


v=NYVMlmL0BPQ&feature=youtu.be"& HYPERLINK "https://www.youtube.com/watch?
v=NYVMlmL0BPQ&feature=youtu.be"feature=youtu.be

https://www.khanacademy.org/science/physics/forces-newtons-laws

Vill du ha fler filmade genomgångar? Klicka i så fall här!

https://www.youtube.com/watch?v=45GQcyEBMhk HYPERLINK "https://www.youtube.com/watch?


v=45GQcyEBMhk&feature=youtu.be"& HYPERLINK "https://www.youtube.com/watch?
v=45GQcyEBMhk&feature=youtu.be"feature=youtu.be

https://www.youtube.com/watch?v=gF5_NFwVOt0 HYPERLINK "https://www.youtube.com/watch?


v=gF5_NFwVOt0&feature=youtu.be"& HYPERLINK "https://www.youtube.com/watch?
v=gF5_NFwVOt0&feature=youtu.be"feature=youtu.be

Detta är två videolektioner där läraren Börje Sundvall pratar om olika typer av krafter samt Newtons tre
lagar. Han gör rikligt med exempel av olika svårighetsgrad. Observera att detta

är långa filmer - det kan vara lämpligt att titta på delar av dem vid olika tillfällen. Eller som repetition vid
kursens slut!

Energi
Energi är ett centralt begrepp inom fysiken. I det här kapitlet ska vi studera mekanisk energi. Vi inleder
med ett litet avsnitt om olika typer av energi och en presentation av energilagen, som är en av
"grundpelarna" inom fysiken.

Orientera dig i läroboken genom att läsa sid 97 - 120 för att kunna följa resonemanget som följer.

Enligt energilagen kan energi inte förstöras eller försvinna, bara omvandlas mellan olika former.
Energilagen är en av de viktigaste principerna i fysiken. Den gäller i både makroskopiska och
mikroskopiska system. Du kommer att använda denna lag i en stor del av dina fortsatta fysikstudier. I det
här kapitlet ska vi i första hand studera mekanisk energi i form av rörelseenergi och lägesenergi. I nästa
kapitel ska vi titta närmare på värmeenergi.

Gör övning 5.1 - 5.2 på s 121 i läroboken.

Laborationsdags!

I samband med det här kapitlet ska du genomföra en hemlaboration. Det gör du med hjälp av en
instruktion. Labbinstruktion och rapportmall finns att hämta på näst sista sidan i denna studieguide
(Sammanfattning och nästa steg). Du bifogar din laborationsrapport i Uppdrag 2.
Arbete
Arbete I vardagligt språk kan vi uträtta ett arbete på många olika sätt. Arbete i fysikalisk mening uträttas
när en resulterande kraft verkar i rörelseriktningen. Det betyder att ett föremål måste röra sig för att ett
arbete ska uträttas. Om kraften i rörelseriktningen är F, uträttas arbetet W = F · s där s är sträckan. Arbete
är en form av energi. I din lärobok betecknas arbete med W.

SI-enheten för alla former av energi är joule, J. I detta kapitel ska vi definiera vad som i fysikalisk mening
menas med energi, arbete och effekt. Hur många hästkrafter måste t.ex. en Porsche ha för att accelerera
från 0-100 på 4,8 s?

Arbete, forts
Formeln W = F · s ger dig att 1 J = 1 Nm. Du multiplicerar ju kraften F (enhet N) med sträckan s (enhet m).

Om kraften inte verkar i rörelseriktningen, bidrar bara en del av kraften till det uträttade arbetet.
Komposantuppdela kraften så som du fick lära dig i kapitel 4.

Exemplet nedan visar hur du kan gå tillväga.

Kraften F verkar på en låda, som figuren visar. F bildar vinkeln α med rörelseriktningen. Hur stort arbete
uträttas när lådan flyttas sträckan s? Klicka även på bilden till höger.

https://archive.geogebra.org/en/upload/files/Sweden_vhojd/mechanic_work.html

Dela upp F i komposanter i x- och y-led, som figuren visar. Du ser då att x-komposanten är i
rörelseriktningen och y-komposanten är vinkelrät mot rörelseriktningen.

Om du tänker dig att det är kraftkomposanterna som verkar istället för F, så kommer endast F x att bidra
till det uträttade arbetet. Fx och Fy kan då beräknas enligt:

Fx = F · cos α och Fy = F · sin α Det uträttade arbetet blir då: W = F x · s = F · cos α · s

Ett mekaniskt arbete uträttar du endast, om du använder en kraft som gör att föremålet förflyttas i
kraftens riktning. Både energi och arbete anges i joule (J) eller Nm.

Kinetisk och potentiell energi


Kinetisk energi

Ett föremål som rör sig har rörelseenergi. Rörelseenergi, eller kinetisk energi, definieras på följande sätt:
Precis som annan energi mäts den kinetiska energin i enheten joule, J.

Kinetisk energi är proportionell mot hastigheten i kvadrat. Det betyder att den kinetiska energin blir fyra
gånger så stor om hastigheten fördubblas. Vi återkommer till detta längre fram.

Potentiell energi

Den potentiella energin, eller lägesenergin, bestäms alltid relativt en viss referensnivå, nollnivå. Den
potentiella energin definieras på följande sätt:

där h är höjden över nollnivån. Den potentiella energin blir negativ om föremålet befinner sig under
nollnivån. Enheten är även här joule, J.

Studera Exempel 2 - 7 på s 102 - 106 i läroboken. Lös övningarna 5.6 - 5.9 samt 5.16 - 5.17 på

s 121 - 122.

Mekanisk energi
Den mekaniska energin är summan av den kinetiska och den potentiella energin:

Nedan finns tre korta filmer om energi, som du uppmanas att titta på!

https://www.youtube.com/watch?v=dXurKNVhatA HYPERLINK "https://www.youtube.com/watch?


v=dXurKNVhatA&feature=youtu.be"& HYPERLINK "https://www.youtube.com/watch?
v=dXurKNVhatA&feature=youtu.be"feature=youtu.be

https://www.youtube.com/watch?v=o9Yg9s7uhgA HYPERLINK "https://www.youtube.com/watch?


v=o9Yg9s7uhgA&feature=youtu.be"& HYPERLINK "https://www.youtube.com/watch?
v=o9Yg9s7uhgA&feature=youtu.be"feature=youtu.be

https://www.youtube.com/watch?v=AnYgFv_iurI HYPERLINK "https://www.youtube.com/watch?


v=AnYgFv_iurI&feature=youtu.be"& HYPERLINK "https://www.youtube.com/watch?
v=AnYgFv_iurI&feature=youtu.be"feature=youtu.be

Mekanisk energi, forts


I de flesta verkliga situationer bevaras inte den mekaniska energin. Vi har förluster på grund av friktion,
värme, luftmotstånd m.m. många situationer kan vi dock få en god uppfattning om förloppet genom att
försumma förlusterna. Vi studerar en idealiserad problemsituation där vi bortser ifrån alla förluster, på
samma sätt som när vi studerade fritt fall. Fördelen är att i så fall bevaras den mekaniska energin. Då blir
det relativt enkelt att analysera olika typer av problem med hjälp av energilagen. Kom ihåg att den
mekaniska energin bevaras i varje ögonblick.
En person som hoppar simhopp har från börja bara potentiell energi. När personen faller neråt ökar
hennes kinetiska energi samtidigt som den potentiella energin minskar. När hon till sist landar i vattnet
har all potentiell energi omvandlats till kinetisk energi. Den totala mekaniska energin kan tecknas som:

Höjden, h, och hastigheten, v, ändras hela tiden under hoppet, men summan av de två termerna i
uttrycket ovan är hela tiden lika stor. Här ser vi ett exempel på den s k energilagen, som innebär att
energin är konstant i ett slutet system. I ett slutet system kan energi bara omvandlas mellan olika former.

Energiprincipen
Energi kan varken skapas eller förstöras. Den kan bara omvandlas i olika former.

Det är det samma sak som att säga att ingen energi försvinner vid en energiomsättning, eller att energin
är lika stor före som efter en energiomsättning. Detta kallar man energiprincipen. Man har ofta nytta av
denna när man löser problem i fysiken.

Det finns en bestämd mängd energi i universum och denna är alltid konstant. Om du äter ett äpple så
innehåller det en mängd solenergi som solen har skickat ut genom att omvandla massa till energi. Äpplet
blir energi i din kropp som gör att du kan utföra ett arbete, t. ex lyfta 10 kg potatis på en hylla, 2 m från
marken. Energin finns nu i potatisen som kan ramla och slå sönder mormors fina vas. Under processen
omvandlas en del av energin till värme genom friktion, men den värmen försvinner inte utan värmer
Jorden och slutligen Universum där den kom ifrån.

Studera energiprincipen genom att klicka på bilderna nedan!

http://phet.colorado.edu/en/simulation/energy-skate-park-basics

http://phet.colorado.edu/en/simulation/energy-forms-and-changes

Laboration nr 1
I samband med denna modul skall du genomföra två laborationer, och det är nu dags för den första som
är en hemlaboration.

Laborationsinstruktion och rapportmall hittar du genom att klicka på knapparna nedan. (Du kan också
komma åt dessa om du klickar på

"Laborationer" i Introduktionsmodulen eller i Kursbiblioteket sist i denna studieguide.)


Du skall nu göra Laboration nr 1!

Laboration nr 1: Energiomvandlingar
http://novo.hermods.se/pluginfile.php/270167/mod_page/content/31/Energiomvandlingar.pdf

Rapport: Bifogas med Uppdrag 2


http://novo.hermods.se/pluginfile.php/270167/mod_page/content/67/1.%20Rapportmall%20med
%20checklista%2C%20rev.pdf

Effekt
Effekt definieras som arbete per tidsenhet.

Effekt mäts i enheten watt (W). Observera att både enheten watt och storheten arbete (energi)
betecknas med W. Det brukar framgå av sammanhanget vilket som avses, men du ska vara medveten om
risken för missförstånd.

Av definitionen på 1 W ser du också att 1 W = 1 J/s.

På elräkningen brukar energiförbrukningen anges i antal kilowattimmar, kWh. Du kan lätt räkna ut antal
joule per kWh om du vet att det går 1000 W på 1 kW och 3600 s på 1 h:

1 kWh = 1000 W ∙ 3600 s = 3,6 ∙ 106 J.

Studera också gärna din formelsamling. Där finns tabeller som visar samband mellan olika enheter för
energi, effekt m.m.

Titta på Exempel 11 på s 118 och gör övningarna 5.36 - 5-39 på s 125 i din lärobok.

Exempel på effekt
Vi tänker oss att du och en kamrat har tävlat om vem som kan dra en tungt lastad kärra uppför en backe
på kortast möjliga tid. Vi antar att det tog 6 min för dig att dra upp kärran, medan din kamrat behövde 7
min. Visserligen utförde ni samma arbete, men du var effektivast, eftersom du gjorde arbetet på kortare
tid!

När man ska köpa en motor för att driva en pump, en hissanordning eller liknande, är man givetvis
intresserad av hur stark motorn är. Men man vill också veta hur lång tid det tar för motorn att uträtta ett
visst arbete. Man vill veta hur effektiv motorn är. Ju kortare tid det tar för motorn att utföra ett visst
arbete, desto större effekt säger man att motorn har.

Med effekt menar man hur stort arbete i newtonmeter eller i joule som uträttas under 1 sekund.
Exempel: En lyftkran lyfter en last som väger 400 kg, d. v.s. tyngden är 4000 N. Lasten lyfts till en höjd av
3 m. Hur stor blir effekten om det tar 6 s att lyfta upp lasten?

Lösning: Arbetet = 4000 · 3 Nm = 12000 Nm = 12000 J

Effekten = 12000 J / 6 s = 2000 J/s = 2000 W (eller 2 kW)

Ibland träffar du på en äldre enhet för effekt, nämligen hästkraft som inte är en kraft, utan en
effektenhet. Begreppet infördes av James Watt när han arbetade med att förbättra ångmaskinen.
Hästkraft används fortfarande i bilindustrin. Om man vill omvandla hästkrafter till watt:

1 hästkraft = 736 W

Motorns hästkrafter och glödlampans watt är båda enheter för effekt.

Verkningsgrad
Hittills har vi bortsett från eventuella förluster på grund av friktion m.m. Det kan vi inte göra när vi ska
studera de energiomvandlingar som sker i t.ex. ett kraftverk. Ofta anges hur stor andel av del tillförda
energin som kan utnyttjas till elenergi. Denna andel brukar kallas verkningsgrad, och definieras på
följande sätt: Verkningsgraden är:

Verkningsgraden anges i procent eller som ett tal i intervallet noll till 1:

Verkningsgraden är en dimensionslös storhet som ofta anges i procentform.

I exempel 9 på s 126 i läroboken anges att 92 % av den potentiella energin omvandlas till elektrisk energi.
Det betyder att verkningsgraden är 92 %. Lös Kontroll 5 på i läroboken.

Verkningsgraden anger hur mycket av den energi som tillförs som är möjlig att utnyttja. I ett vanligt
kraftverk med verkningsgraden 40 % kan bara ca 40 % av energiinnehållet i bränslet omvandlas till
elenergin. En bensinmotor har verkningsgraden ca 30 %

medan en elmotor har ca 80 %. Det är bland annat därför forskning pågår för att konstruera bättre
elbilar.

Titta på en annorlunda förklaring av energibegreppet!

https://www.youtube.com/watch?v=vl4g7T5gw1M HYPERLINK "https://www.youtube.com/watch?


v=vl4g7T5gw1M&feature=youtu.be"& HYPERLINK "https://www.youtube.com/watch?
v=vl4g7T5gw1M&feature=youtu.be"feature=youtu.be

Friktion
Läs s 34-35 i läroboken. Friktionskraften är proportionell mot normalkraften:
där μ är friktionskoefficienten. I problem där vi har kraftjämvikt, kan friktionskraften oftast bestämmas
med hjälp av jämviktsvillkoren. Exemplet nedan visas en kraftsituation där friktionskraften och
friktionskoefficienten ska bestämmas.

Exempel: En låda med massan 15 kg dras längs golvet med en dragkraft på 65 N. Lådan rör sig med
konstant hastighet. Bestäm friktionskraften och friktionskoefficienten.

Vi har kraftjämvikt eftersom lådan rör sig med konstant hastighet. Vi kan ställa upp följande
jämviktsvillkor: FN = m · g och F = Fμ

Friktionskraften är lika stor som dragkraften F, det vill säga 65 N.

Vi kan nu bestämma normalkraften: FN = m · g = 15 · 9,82 N = 147,3 N

Till sist beräknar vi friktionskoefficienten:

(Obs! enhetslös)

Utforska krafter, energi och arbete medan du trycker föremål upp och ner på en ramp! Sänk och höj
rampen för att se hur lutningsvinkeln påverkar de parallella krafterna som verkar. Diagrammen visar
krafter, energi och arbete.

http://phet.colorado.edu/en/simulation/the-ramp

http://phet.colorado.edu/en/simulation/ramp-forces-and-motion

Friktionsarbete
När en bil bromsar in, verkar en friktionskraft i motsatt riktning mot bilens rörelseriktning.
Friktionskraften uträttar ett arbete, bromsarbetet, som leder till att bilens hastighet minskar.
Bromsarbetet leder till att bilens rörelseenergi minskar. Vi kan teckna energiomvandlingen:

Bromsarbete = ändring av rörelseenergi. Med formler:

Exempel: En bil kör med hastigheten 45 km/h. Den bromsar in till stillastående. Friktionskraften är 5,0 kN
och bilens massa är 850 kg.

a) Hur lång är bromssträckan? b) Hur lång blir bromssträckan om bilen istället kör med hastigheten 90
km/h?
a) Enligt resonemanget ovan får vi:

Obs! Hastigheten i m/s.

b) Lös själv! Lösning finns här:

Sammanfattning
Repetera de övningsuppgifter som hänvisats till i föregående text. Gör gärna

fler övningar, t.ex. 5.20, 5.24 och 5.25.

Repetera lärobokens lösta exempel, samt studera och lös kontrollfrågorna under de sidor som du läst i
läroboken för detta avsnitt.

Läs sammanfattningen på s 120 i läroboken. Gör också Testa-dig-själv 2:1 (den orange knappen nedan).

Du ska förstå följande begrepp och kunna tillämpa dem i praktisk problemlösning:

*) Energiprincipen

*) Arbete och effekt

*) Kinetisk energi

*) Potentiell energi

*) Mekanisk energi

*) Verkningsgrad

*) Friktion

Rörelsemängd
Läs s 266-272 i din lärobok. Vi kan beräkna rörelsemängden för ett föremål om vi vet massan och
hastigheten. Rörelsemängden definieras som massan gånger hastigheten, p = mv. Rörelsemängden är en
vektorstorhet och har samma riktning som hastigheten. Enheten för rörelsemängd är 1 kgm/s. Lös
uppgifterna 11.37-11.40 på s 280 i läroboken.

Inledningsvis låter vi två vagnar med lika massor skjutas isär av en fjäder. Kan man säga något om
vagnarnas fart efter kollisionen? Självklart blir vagnarnas farter lika, men med motsatt riktning. Två
vagnar med olika massor får sedan skjutas isär på samma sätt. Nu är det dock svårare att säga något om
farterna efter kollisionen. I detta avsnitt skall vi dock komma fram till hur dessa farter kan beräknas!

Vi studerar inledningsvis ett explosionsförlopp, d v s där två vagnar med massorna m 1 och m2
inledningsvis är stillastående och sammankopplade med en spänd fjäder. När fjädern utlöses åker
vagnarna iväg åt var sitt håll med olika hastighet. Vi ska nu använda kunskaper från Fysik och försöka att
komma fram till ett samband mellan vagnarnas hastiget och massor. Först står alltså vagnarna stilla och
påverkar varandra med fjäderkraften F:

Forts nästa sida!

Rörelsemängd, forts
Under själva explosionsförloppet åker vagnarna isär. Påverkade av kraften F, som inte är konstant, ökar
vagnarnas hastigheter. Vi kan under den korta tid fjädern sprätter isär ställa upp kraftekvationen för
vagnarna, där a1 och a2 är deras acceleration:

m1a1 = −F (1)

m2a2 = F (2)

Eftersom vi valt positiv referensriktning åt höger, blir kraften F på m 1 negativ. Vi är trots allt inte
intresserade av krafterna. Adderar vi (1) och (2) eliminerar vi kraften:

m1a1 + m2a2 = 0 (3)

Vi är heller inte intresserade av accelerationen. Om vi därför multiplicerar höger och vänster ledet i (3)
med tiden t får vi hastigheten. Kom ihåg att t · a = v !

Tydligen är i detta fall summan av produkten av vagnarnas hastighet och massa noll.

Ett föremål som har massan m och hastigheten v sägs ha rörelsemängden p: p = m · v

Vektorstorheten rörelsemängd mäts i enheten [kgm/s].

Exempel: Vilken rörelsemängd har en bil med massan 1200 kg och farten 25 m/s?

Lösning: p = m ∙ v = 1200 ∙ 25 kgm/s = 30 000 kgm/s

Rörelsemängdens konstans
Man är sällan intresserad av rörelsemängdens storlek. Begreppet rörelsemängd används nästan
uteslutande som ett sätt att tänka när man löser stöt- och kollisionsproblem. I uttrycket (4) ovan kom vi
fram till att summan av rörelsemängderna var noll.

Uttrycket visar alltså att rörelsemängden är konstant under ett rörelseförlopp, under förutsättning att det
saknas påverkan från yttre krafter (friktion t ex). Om t ex två vagnar kolliderar är deras totala
rörelsemängd lika stor före som efter kollisionen. Rörelsemängden är bevarad. I en kollision där
resultanten av de yttre krafterna är noll bevaras rörelsemängden.

Lagen om rörelsemängdens bevarande:

https://www.youtube.com/watch?v=4IYDb6K5UF8 HYPERLINK "https://www.youtube.com/watch?


v=4IYDb6K5UF8&feature=youtu.be"& HYPERLINK "https://www.youtube.com/watch?
v=4IYDb6K5UF8&feature=youtu.be"feature=youtu.be

Klicka på bilden för att köra laborationen:

http://tube.geogebra.org/student/m54561?mobile=true

Exempel:

En kanon med massan 850 kg avlossar en kanonkula med massan 20 kg. Kulan får hastigheten 35 m/s.
Vilken rekylhastighet får kanonen?

För lösning - klicka här:

Impulslagen
En boll som studsar i golvet ändrar rörelseriktning vid studsen. Den påverkas av en uppåtriktad kraft som
verkar under den tid bollen är i kontakt med golvet. Begreppet impuls ger ett mått på kraftens verkan
under en viss tid. Impulsen definieras som medelkraften multiplicerad med tiden, I = Ft. Enheten för
impuls är 1 Ns.

Medelkraften är konstant under hela förloppet. I Ft-grafen nedan till höger blir impulsen, I = Ft, lika med
arean under grafen. I verkligheten varierar kraften med tiden, t ex som i den vänstra grafen nedan. Även
då är impulsen lika med arean under grafen.

Enligt impulslagen är den impuls som ett föremål påverkas av lika med ändringen av rörelsemängden:

Av sambandet ovan kan vi dra slutsatsen att enheten för impuls, Ns, är samma sak som enheten för
rörelsemängd, kgm/s. Det kan vi visa med Newtons andra lag, kraftlagen:
Vi sätter in i impulslagen:

Metoden kallas dimensionsanalys.

Lös uppgifterna 11.41-11.43 på s 280 i läroboken.

Impulslagen, forts
När man missar spikhuvudet med hammaren och istället träffar pekfingernageln så gör det ju ganska ont.
Tydligen påverkas nageln av en rätt så stor kraft. Frågan är hur stor? Likaså när en boll träffar en vägg så
ändrar ju bollens hastighet riktning nästan omedelbart. Det måste innebära att bollen (och väggen)
påverkas av en stor kraft. För att hitta en metod för att beräkna krafternas storlek vid korta stötförlopp
gör vi en liten härledning. Vi börjar med två välkända formler:

Vänsterledet är inget annat än ändringen av rörelsemängden, dvs. om kraften F verkar under tiden t på
ett föremål så ändras dess rörelsemängd Δp. Ändring av rörelsemängd kallas även impuls, som mäts i
enheten Ns.

Exempel: Anta att hammaren träffar pekfingernageln med farten v = 2,5 m/s och lämnar nageln med
samma motriktade fart. Kontakttiden mellan hammare och nagel uppskattas till 1 ms. Vilken kraft
påverkas nageln av, om hammaren väger m=0,75 kg?

För lösning - se här:

Se även filmen -------->

https://www.youtube.com/watch?v=bA6wtKFjXMM HYPERLINK "https://www.youtube.com/watch?


v=bA6wtKFjXMM&feature=youtu.be"& HYPERLINK "https://www.youtube.com/watch?
v=bA6wtKFjXMM&feature=youtu.be"feature=youtu.be

Videolektion, rörelsemängd och impuls


Se videolektionen om rörelsemängd och impuls genom att klicka på bilden nedan:

https://www.youtube.com/watch?v=UmGU6citC74 HYPERLINK "https://www.youtube.com/watch?


v=UmGU6citC74&feature=youtu.be"& HYPERLINK "https://www.youtube.com/watch?
v=UmGU6citC74&feature=youtu.be"feature=youtu.be
Bevarande av rörelsemängden
I det här avsnittet ska vi studera vad som händer med rörelsemängden när två föremål kolliderar med
varandra. Vi kan beskriva det som ett sammansatt system som består av två föremål. Systemet påverkas
inte av krafter utifrån. Det kan kallas för ett slutet system. Då bortser vi t ex från luftmotstånd, friktion
och annan yttre kraftpåverkan. Resultanten av de yttre krafterna är noll.

Gå igenom exemplet om rörelsemängdens bevarande på s 268 i läroboken.

Resonemang ger lösning! Ibland är det betydligt enklare att resonera sig fram till en lösning än att räkna
sig fram via t ex rörelsemängd och impuls. Vi skall se tre exempel på detta. Vi antar att stötarna är
fullständigt elastiska, dvs. både rörelsemängd och rörelseenergi bevaras.

Exempel: En boll som kastas med farten 18 m/s rakt in i en vägg studsar tillbaks med ungefär samma
hastighet. Hur stor är bollens hastighetsändring efter jämfört med före studsen i väggen?

Lösning: Först minskar bollen farten från 18 till 0 m/s. Sen ökar den farten från 0 till 18 m/s åt andra
hållet. Alltså är bollens hastighetsändring (18 + 18) m/s = 36 m/s.

Exempel: En golfklubba träffar bollen med farten 25 m/s. Klubbans fart ändras obetydligt efter träffen.
Bestäm golfbollens hastighet.

Lösning: Se här:

Anta istället att bollen träffar den stillastående klubban med farten 25 m/s. Det är i princip samma sak så
länge bollens och klubbans relativa hastigheter är oförändrade. Bollen studsar då tillbaks med
hastigheten 25 m/s och har alltså ändrat sin hastighet med 50 m/s. Den stillastående bollen ändrar sin
hastighet lika mycket, dvs. den får farten 50 m/s.

Tänk till! En golfspelare slår iväg en golfboll med hastigheten 120 km/h. Med vilken

kraft påverkar klubban bollen om bollens massa är 45 g och kontakttiden är 0,2 ms?

Stöt
I det här avsnittet ska vi tillämpa lagen om rörelsemängdens bevarande på olika typer av stötar.
Rörelsemängden bevaras vid alla typer av stötar. I det här avsnittet ska vi diskutera följande typer av
stötar:

*) Rak stöt: Föremålen rör sig längs en rät linje både före och efter kollisionen. Hastighetsvektorerna är
parallella.

*) Oelastisk stöt: Den kinetiska energin i systemet är mindre efter kollisionen än före. I praktiken är alla
stötar oelastiska.

*) Fullständigt oelastisk stöt: Föremålen hänger samman efter kollisionen. De fortsätter tillsammans
med samma hastighet.

*) Elastisk stöt: Den kinetiska energin för systemet är lika stor efter kollisionen som före.
Exemplet nedan visar en fullständigt oelastisk stöt:

Exempel: En bil med massan 1100 kg och hastigheten 15 m/s kör in i en stillastående bil med massan 950
kg. Båda fortsätter framåt tillsammans efter kollisionen. Vilken hastighet har de sammankopplade
fordonen direkt efter kollisionen?

Lösning: Se här:

Mer om stötar
Exempel 1: En tågvagn med farten 2,5 m/s och vikten 16 ton kolliderar med en stillastående vagn som
väger 28 ton. Efter kollisionen har 16 tons vagn farten 0,55 m/s åt motsatt håll. Bestäm farten hos 28
tons vagn.

Lösning:

Exempel 2: Är kollisionen mellan vagnarna i föregående exempel fullständigt elastisk?

Lösning:

Här hittar du en genomgång av rörelsemängd och impuls från KhanAcademy. Klicka på bilden och välj
område: https://www.khanacademy.org/science/physics/linear-momentum

Sammanfattning
Studera Tänk till! 10 samt gör Kontroll 9 på s 269 i läroboken.

Du har också här bredvid två längre filmer om rörelsemängd och impuls.

https://www.youtube.com/watch?v=eyATHdTb0Z8 HYPERLINK "https://www.youtube.com/watch?


v=eyATHdTb0Z8&feature=youtu.be"& HYPERLINK "https://www.youtube.com/watch?
v=eyATHdTb0Z8&feature=youtu.be"feature=youtu.be

https://www.youtube.com/watch?v=11yuAqR19bs HYPERLINK "https://www.youtube.com/watch?


v=11yuAqR19bs&feature=youtu.be"& HYPERLINK "https://www.youtube.com/watch?
v=11yuAqR19bs&feature=youtu.be"feature=youtu.be

Sammanfattning rörelsemängd:

Läs sammanfattningen, de sista punkterna i läroboken s 273.


Du ska kunna följande om rörelsemängd och impuls:

Laboration nr 2
I samband med denna modul skall du genomföra två laborationer, och det är nu dags för den andra som
är en webblaboration.

Laborationsinstruktion, laborationsfilm och rapportmall hittar du genom att klicka på knapparna nedan.
(Du kan också komma åt dessa om du klickar på "Laborationer" i Introduktionsmodulen eller i
Kursbiblioteket sist i denna studieguide.)

Du skall nu göra Laboration nr 2!

Laboration nr 2: Rörelsemängdens bevarande


http://novo.hermods.se/pluginfile.php/270167/mod_page/content/65/R%C3%B6relsem
%C3%A4ngdens%20bevarande.pdf

Film till Lab nr 2 https://vimeo.com/207279555

Rapport: Bifogas med Uppdrag 2


http://novo.hermods.se/pluginfile.php/270167/mod_page/content/67/1.%20Rapportmall%20med
%20checklista%2C%20rev.pdf

Sammanfattning
I den här modulen har du jobbat med:

*) Krafter

*) Energi, effekt och friktion

*) Rörelsemängd och impuls

Det du nu skall göra är att repetera hela kapitlet (lös gärna fler övningar i din lärobok). Därefter gör du
TDS 2:2, som innehåller uppgifter på hela denna modul. Eventuellt kan det sedan behövas ytterligare lite
repetition.

När du tycker att du behärskar samtliga moment i denna modul gör du Uppdrag 2. Detta skickar du in till
din webblärare via datorn, och denne rättar och kommenterar det. Glöm inte att bifoga de två
laborationsrapporterna.

Därefter väntar modul 3, som kommer att behandla begreppen termofysik, klimat och ellära.
Uppdrag, bedömning och kunskapskrav
Uppdragen är en viktig del i kunskapsinhämtningen. Det är därför viktigt att du är noggrann när du
arbetar med uppdragen, och att du är säker på att du förstår och behärskar det du skickar in. Vid den
avslutande examinationen kommer du att få visa just detta.

Hur mycket arbete du ska lägga ner på svaren styrs bl a av din betygsambi-tion, dina förkunskaper, hur
snabbt du läser kursen etc.

Jobba för att hålla studieplanen och de inlämningsdatum som finns i Novo. Risken är annars att du får för
mycket att göra i slutet av kursen.

Kom också ihåg att betyget sätts efter den Avslutande examinationen. Det är meningen att du skall
utvecklas och lära dig under kursens gång.

Frågorna i uppdragen är uppdelade i olika betygsnivåer: E, C, och A.

Efter varje uppdrag står det vilken nivå uppgiften ligger på.

Läs om bedömning och betyg i det svarta fältet. Vi som är lärare i kursen vill att du skall nå ditt mål. Vi
skall också säkerställa att du når Skolverkets uppsatta kunskaps-krav. Vill du repetera kunskapskraven för
kursen? Tryck då på ikonen så kommer du direkt till Skolverkets hemsida.
https://www.skolverket.se/undervisning/vuxenutbildningen/komvux-gymnasial/laroplan-for-vux-och-
amnesplaner-for-komvux-gymnasial/amne?url=1530314731%2Fsyllabuscw%2Fjsp%2Fsubject.htm
%3FsubjectCode%3DFYS%26courseCode%3DFYSFYS01a%26lang%3Dsv%26tos%3Dgy%26webtos
%3Dvuxgy%26p%3Dp HYPERLINK "https://www.skolverket.se/undervisning/vuxenutbildningen/komvux-
gymnasial/laroplan-for-vux-och-amnesplaner-for-komvux-gymnasial/amne?
url=1530314731%2Fsyllabuscw%2Fjsp%2Fsubject.htm%3FsubjectCode%3DFYS%26courseCode
%3DFYSFYS01a%26lang%3Dsv%26tos%3Dgy%26webtos%3Dvuxgy%26p
%3Dp&sv.url=12.b173ee8160557dd0b8100d#anchor_FYSFYS01a"& HYPERLINK
"https://www.skolverket.se/undervisning/vuxenutbildningen/komvux-gymnasial/laroplan-for-vux-och-
amnesplaner-for-komvux-gymnasial/amne?url=1530314731%2Fsyllabuscw%2Fjsp%2Fsubject.htm
%3FsubjectCode%3DFYS%26courseCode%3DFYSFYS01a%26lang%3Dsv%26tos%3Dgy%26webtos
%3Dvuxgy%26p
%3Dp&sv.url=12.b173ee8160557dd0b8100d#anchor_FYSFYS01a"sv.url=12.b173ee8160557dd0b8100d#
anchor_FYSFYS01a

Dina resultat under kursens gång samlas i en matris. Matrisen visar i vilken utsträckning du uppnått de
olika kunskapskraven i kursen. Matrisen, tillsammans med lärarens kommentarer, vägleder dig i vad du
behöver träna mer på för att nå ditt betygsmål. Efter den avslutande examinationen kommer matrisen
justeras en sista gång och därmed utgöra en del av betygsmotiveringen. Kom ihåg att nivån du visat
under kursens gång också behöver visas under den avslutande examinationen.

Studieguide 3
En översikt
I din tredje modul kommer du att få lära dig mycket om termofysik - här studerar du bl. a tryck och
temperatur.

Detta följs av ett avsnitt om klimat och väder, och till sist kommer ett stort avsnitt om grundläggande
ellära.

Klicka nu på äpplet för att få en konkret målbeskrivning!

Målbeskrivning
Här får du en målbeskrivning, så att du kan få en uppfattning om vad ditt studium av den tredje modulen
ska leda fram till. När du har gått igenom de tre kapitlen Termofysik, Klimat och väder och Elektricitet
skall du:

• förstå vad tryck är och kunna göra beräkningar på detta

• vara bekant med begreppen temperatur och absolut temperatur

• kunna använda tillståndslagen för ideala gaser

• förstå begreppet värme och kunna göra beräkningar på värmeenergi, samt härvid använda dig av
begreppen specifik värmekapacitet samt smält- resp ångbildningsvärme

• förstå vad som menas med energikvalitet

• ha kunskaper om atmosfär, luft och lufttryck

• förstå skillnaden mellan klimat och väder, samt känna till olika väderfenomen och kunna tolka
väderprognoser

• förstå bakgrunden till växthuseffekten samt kunna tolka olika klimatmodeller

• ha kunskaper om och kunna göra beräkningar på elektrisk laddning och elektriska fält

• behärska och kunna räkna på olika begrepp inom ellära, t ex spänning, ström, resistans och
effekt

• förstå begreppet elektrisk potential

Studietips
Arbeta med studieguiderna här i Novo parallellt med att du läser och löser uppgifter i din lärobok.

Var aktiv, titta på webblektioner och gör interaktiva övningar här i Novo, gör Testa-dig-själv och andra
övningar.

Arbeta aktivt med ditt betygsmål. Uppgifterna i upp-dragen här i Novo är nivå-uppdelade (E-, C- och A-
nivå). Koncentrera dig på den nivå som är ditt mål!
Utnyttja fysikhjalp@hermods.se

Att läsa i din lärobok


Laddningar och fält, s 127 - 154

Elektrisk energi, spänning och ström, s 155 - 174

Elektriska kretsar, s 175 - 202

Tryck, s 46 - 61

Värme, s 203 - 220

Energi, miljö och klimat, 221 - 244

Inledning
Denna studieguide omfattar delar av kapitel 3 samt kapitel 6 - 10 i läroboken.

I denna modul kommer du bl a att få stifta bekantskap med följande begrepp:

Tryck och temperatur

Värmeenergi

Energikvalitet

Klimat och väder

Elektrisk laddning

Elektriska fält

Spänning, ström, resistans

Elektrisk potential

Effekt

När modulen är klar skall du sända in ditt tredje uppdrag. Laboration nr 3 och nr 4 skall bifogas Uppdrag
3. Instruktioner till dessa kommer längre fram i denna modul.

Avsnitt 7: Ellära
De begrepp som tas upp i följande avsnitt om elektricitet hittar du i läroboken under kapitel 6, 7 och 8.
Läs i första hand sidorna 127 - 150, 155 - 166, 168 - 170 samt 175 - 196. Studera bilder, lösta exempel
samt memorera de grå faktarutorna.
Kunskapen om elektriciteten började växa fram redan under 1700-talet. Det var dock inte förrän på
1900-talet som elektriciteten blev allmänt tillgänglig i vårt samhälle. Idag är det få som klarar sig utan
tillgång till elektricitet.

Vad är elektricitet?

I det här kapitlet ska vi försöka svara på den frågan. Vi ska gå igenom vad som avses med elektrisk
laddning, spänning och ström. Vi ska också studera olika typer av elektriska kretsar och gå igenom vilka
samband som gäller vid olika typer av kopplingar. Till sist tar vi upp hur stor energiomsättningen är i en
elektrisk krets och hur den elektriska effekten kan beräknas.

Lite historia om el:

1800 Volta uppfinner batteriet men ingen visste vad det var som vi idag kallar för el. Många började
experimentera med el.

1890 Det svenska ledningsnätet började byggas ut och ASEA startas. Först används el bara i industrin
men snart börjar vanliga hushåll använda glödlampor.

1910 Under första världskriget blir glödlampan en ersättning för fotogen.

1930 Nu börjar man sätta in en elektrisk vattenpump i husen och man slipper bära vatten i hinkar. Radion
gör sitt intåg.

1950 Nu får vanliga människor elektriska apparater som spis, kylskåp, dammsugare och tvättmaskin. Tv:n
gör sin debut.

1970 Husen värms med el och alla skaffar nya elektriska apparater.

1990 Apparater som är mindre energikrävande blir tillgängliga. Nu introduceras t ex mikrovågsugnen och
lågenergilampan.

Elektrisk laddning
I ett och samma grundämne är alla atomer lika, det har vi pratat om i samband med materia, men alla
atomer består av samma "byggstenar".

Alla atomer har en kärna. Kärnan består av protoner och neutroner. Kring atomkärnan kretsar elektroner
på olika avstånd från kärnan. Mellan kärnan och elektronmolnen finns absolut ingenting.

Det finns två slags elektriska laddningar, nämligen positiva och negativa. Protonerna är positivt laddade
och elektronerna är negativt laddade. Neutronerna har däremot ingen elektrisk laddning.

Antalet protoner, neutroner och elektroner i en atom bestämmer vilket ämne det är. I en atom finns det
lika många elektroner som protoner. Atomen är neutral. Om en eller flera elektroner lämnar atomen,
kommer protonernas positiva laddningar bli fler och atomen blir positivt laddad. Om atomen får extra
elektroner blir atomen negativt laddad. Positiva eller negativa atomer kallas för joner.

Statisk elektricitet uppstår genom att vi tar bort eller tillför elektroner genom friktion. Om elektronerna
slits bort blir föremålet positivt laddat och om elektronerna blir fler så blir föremålet negativt laddat.
Elektriska laddningar påverkar varandra med krafter. Lika laddningar repellerar (stöter bort) varandra.
Olika laddningar attraherar (dras till) varandra. Med lika menar vi att de har samma tecken, antingen
båda positiva eller båda negativa.

Den fysikaliska storheten laddning betecknas med Q. Ibland används q för att markera att det är en liten
laddning. SI-enheten för laddning är coulomb, som skrivs C.

Titta nu på filmen!

https://www.youtube.com/watch?v=rpyHFMwaNDM HYPERLINK "https://www.youtube.com/watch?


v=rpyHFMwaNDM&feature=youtu.be"& HYPERLINK "https://www.youtube.com/watch?
v=rpyHFMwaNDM&feature=youtu.be"feature=youtu.be

Repulsion och attraktion


Två ledande lätta aluminiumfolieklädda kulor hänger i koppartrådar (figur 1). Koppartrådarna ansluts
sedan till den stora kulan på en bandgenerator. Bandgeneratorn sätts på. Kulorna åker då isär, d v s de
repellerar varandra (figur 2). Därefter flyttas den ena anslutningen till bandgeneratorns nedre del.
Kulorna vill då närma sig varandra, dvs de attraherar varandra (figur 3). Hur kan fenomenen förklaras?

Bandgeneratorn ’pumpar upp’ laddning till den stora metallkulan. Ansluts båda koppartrådarna till
bandgeneratorns metallkula får de små kulorna samma laddning. Tydligen repellerar lika laddning
varandra. Om vi ansluter istället ena koppartråden till metallkulan och den andra till bandgeneratorns
nedre del, som har ”brist” på laddning, får kulorna olika laddning. Tydligen attraherar olika laddningar
varandra. Vilken laddning kulorna har, positiv eller negativ, ger försöket inget besked om.

Varför håller en ballong till din tröja? Gnid en ballong på en tröja, sedan släpp ballongen och den flyger
över och fastnar på tröjan. Se laddningarna i tröjan, ballonger, och väggen.

http://phet.colorado.edu/en/simulation/balloons

Elementarladdningen
Elementarladdningen e, är en naturkonstant som definieras som det storleken av protonens eller
elektronens laddning. Det går inte att skapa en mindre laddning än den elektronen eller en enskild
proton har. e = 1,60 ∙ 10-19 C

Protonens och elektronen har lika stora laddningar, men protonens laddning är positiv och elektronens
negativ.

Man säger att laddning är en s.k. kvantiserad storhet, vilket betyder att den bara förekommer i vissa
bestämda värden. Man kan säga att alla laddningar, oavsett tecken, består av ett helt antal
elementarladdningar Q = n ∙ e , där n är ett positivt eller negativt heltal.

Exempel:
Hur många elektroner finns på den stora bandgeneratorkulan, vars laddning är Q=87 nC?

Lösning:

Exempel:

En kula med laddningen 5,0 nC nuddar kort en lika stor kula med laddningen 10nC. Bestäm kulornas
laddning efter kontakten. Se figuren nedan.

Lösning:

Se filmerna här till höger - om att laddning bevaras och kvantiseras!

https://youtu.be/qWizhPaqNAU

https://youtu.be/I HYPERLINK "https://youtu.be/IiOvHMUO5M0?


list=PL36DUELaLN1of98lYf3MEM12vN0C2bnVq"i HYPERLINK "https://youtu.be/IiOvHMUO5M0?
list=PL36DUELaLN1of98lYf3MEM12vN0C2bnVq"OvHMUO5M0?
list=PL36DUELaLN1of98lYf3MEM12vN0C2bnVq

Coulombs lag
Krafterna mellan två laddningar kan beräknas med hjälp av Coulombs lag:

där k är en konstant, k = 8,99 ∙109 Nm2/C2, Q1 och Q2 är laddningarnas storlek och r är avståndet mellan
laddningarna. Enligt Newtons tredje lag påverkar laddningarna varandra med lika stora, motriktade
krafter.

Exempel: Två laddningar med storleken Q1 = +0,15 μC och Q2 = -0,48 μC befinner sig på

a) 25 cm avstånd från varandra

b) 12,5 cm avstånd från varandra

Hur stor kraft verkar på vardera laddningen i de två fallen? Bestäm storlek och riktning.

Lösning: Laddningarna har olika tecken. Då är krafterna attraherande i båda fallen. Coulombs lag ger:
Coulombs lag, forts
Jämför resultaten i a och b. Om avståndet halveras blir kraften fyra gånger så stor. Det beror på att
avståndet kvadreras i nämnaren när kraften beräknas.

Viktigt när du räknar med Coulombs lag: Använd alltid positiva värden när du beräknar krafter med hjälp
av denna lag. Avgör om kraften är attraherande eller repellerande genom att studera laddningarnas
tecken. Visa riktningen med en figur eller förklarande text. Coulombs lag ger kraftens storlek, inte
riktning!

Exempel: Två laddningar med vardera laddningen 5,6 nC och 2,9 nC befinner sig på avståndet 0,50 mm
från varandra. Bestäm kraftens storlek laddningarna påverkas av.

Lösning:

Exempel: Tre laddningar, q1 = -5nC, q2 = +10 nC och q3= -5nC, är placerade enligt figuren nedan. Bestäm
den resulterande kraften på q3.

Lösning:

Elektrisk influens
De flesta föremål är elektriskt neutrala. De innehåller precis lika många protoner som elektroner, så att
laddningarna tar ut varandra. Om du kommer ihåg minilaborationen med ballong mot tröjan blir
ballongen uppladdad. Du kan då få den att fastna mot väggen. Varför?

Väggen är elektrisk neutral; den är oladdad. Om ballongen är negativt laddad kommer den att putta bort
elektroner i väggen. Väggens utsida blir positivt laddad och insidan blir negativt laddad. Ballongen
kommer då att fastna eftersom positiva och negativa laddningar attraherar varandra. På samma sätt om
håller en positivt laddad plaststav intill en oladdad pingisboll, målad med en ledande färg, dras
pingisbollen mot plaststaven. Hur kan det komma sig att ett laddat föremål attraherar ett oladdat?
Pingisbollen är ju ledande, d v s har gott om fria elektroner. Dessa elektroner i pingisbollen dras mot den
laddade staven. Alltså attraherar staven bollen, enligt figuren nedan. Fenomenet kallas elektrisk influens.
Titta på filmen!

https://www.youtube.com/watch?v=p97T7wUF988 HYPERLINK "https://www.youtube.com/watch?


v=p97T7wUF988&feature=youtu.be"& HYPERLINK "https://www.youtube.com/watch?
v=p97T7wUF988&feature=youtu.be"feature=youtu.be

Ledare och isolatorer


Olika material har olika elektriska egenskaper. Metaller betraktas som ledare eftersom det finns en
massa elektroner som är fritt rörliga, dvs. inte bundna till atomen. I en isolator däremot sitter
elektronerna hårt bundna vid atomerna. Keramik, gummi och de flesta plaster är isolatorer.

Jobba med övningsuppgifterna i läroboken, 6.1 - 6.14, s 151.

Titta på en genomgång av ledare och isolatorer!

https://youtu.be/ert20L-B3pU?list=PLq7xgBW6yRglB0tEGXg7e_-XrZif53bJO>

Elektriska fält
Läs texten i läroboken s 141 - 149.

I de här avsnitten ska vi definiera elektrisk spänning och elektrisk fältstyrka. Vi börjar med att definiera
elektrisk fältstyrka. Därefter går vi tillbaka och definierar spänning. Till sist knyter vi ihop de båda
begreppen!

Innan vi introducerar elektrisk fältstyrka ska vi anknyta till något som du känner till, nämligen jordens
gravitationsfält. Ett föremål som befinner sig i jordens gravitationsfält påverkas av en kraft, tyngdkraften.
Tyngdkraften verkar överallt i gravitationsfältet.

Vi har en liten positiv elektrisk provladdning, q, som befinner sig i närheten av en stor laddad kula.
Laddningen påverkas av en elektrisk kraft från kulan. På samma sätt som vi har ett gravitationsfält runt
jorden, kan vi införa ett elektriskt fält runt kulan. Det elektriska fältet ger upphov till den kraft som verkar
på provladdningen.

Vi definierar den elektriska fältstyrkan, E, som elektrisk kraft per laddningsenhet:

Enheten för elektrisk fältstyrka är N/C.

Det elektriska fältet har samma riktning som kraften på en positiv provladdning, se figur nedan. I den
vänstra figuren är kulan positivt laddad. Den positiva provladdningen repelleras. Fältet är utåtriktat. I den
högra figuren är kulan negativt laddad. Den positiva provladdningen attraheras. Fältet är inåtriktat. Titta
på filmen!
https://www.youtube.com/watch?v=NvNd-0AYAVg HYPERLINK "https://www.youtube.com/watch?
v=NvNd-0AYAVg&feature=youtu.be"& HYPERLINK "https://www.youtube.com/watch?v=NvNd-
0AYAVg&feature=youtu.be"feature=youtu.be

Fältstyrka
Fältstyrkan är ett mått på fältlinjernas täthet. Exempelvis minskar fältstyrkan med avståndet från en
punktladdning. Vektorstorheten elektrisk fältstyrka E definieras som kvoten mellan den kraft F en
laddning q påverkas av i fältet:

Enheten är N/C.

Exempel:

Mellan två laddade parallella plattor är det elektriska fältet homogent, dvs. fältstyrkan är konstant. Vilken
fältstyrka krävs för att en pingisboll som väger 5,3 g skall kunna sväva fritt, om bollen är laddad med Q =
-87 nC ?

Lösning: Vi börjar med att rita en figur. Eftersom jämvikt råder och bollen skall sväva måste bollens
tyngdkraft Ft vara lika stor som kraften Fe från det elektriska fältet, dvs:

Svaret verkar vara stort och säger oss ännu inte så mycket.

Fältstyrka, forts
En laddning, q, påverkas alltså av en kraft i det elektriska fältet, E, så att F = Q · E.

Observera att kraft och fältstyrka är vektoriella storheter. De har alltså båda storlek och riktning. Den
elektriska kraften har samma riktning som det elektriska fältet.

Exempel:

Hur stor är fältstyrkan på avståndet r från den positiva punktladdningen Q?

Lösning:

Fältstyrkan från en punktladdning avtar med avståndet, dvs fältlinjerna glesnar när avståndet ökar. Vi
inför en liten testladdning q på avståndet r från Q och beräknar med hjälp av Coulombs lag kraften den
påverkas av:

Det elektriska fältet på avståndet r från Q är tydligen:

Storheterna spänning och fältstyrka hänger samman, frågan är hur? Anta att vi vill flytta den positiva
laddningen q sträckan d från den positiva till den negativa elektroden i det homogena fältet E. Vi måste
utföra arbetet W:

Om spänningen mellan elektroderna är U kan vi även uttrycka arbetet med tidigare kunskaper. Sätter vi
samman de två uttrycken får vi:

Detta är alltså sambandet mellan spänning och fältstyrka. Fältstyrkan anger alltså hur stor spänningen är
per meter. Enheten för elektrisk fältstyrka kan alltså även uttryckas som V/m.

Elektrisk spänning
Vi börjar med gravitationsfältet. Du uträttar ett arbete när du lyfter upp en låda från golvet och ställer
den på bordet. En uppåtriktad kraft, motriktad gravitationsfältet, verkar för att lyfta lådan. Det uträttade
arbetet ger lådan en ökad potentiell energi, W = mgh.

Vi genomför nu motsvarande resonemang i det elektriska fältet. När en liten provladdning "lyfts" från en
punkt till en annan i det elektriska fältet, uträttas ett arbete, W. Vi definierar den elektriska spänningen
mellan punkterna som arbetet per laddningsenhet:

Elektrisk spänning betecknas med U och mäts i enheten volt (V), vilket uttryckt i SI-systemets
grundenheter blir: V= J/C

OBS! Spänning mäts alltid mellan två punkter.

En elektron (q) som befinner sig i det elektriska fältet mellan två laddade kondensatorplattor (se bilden)
dras ju till den positiva plattan. För att flytta elektronen till den negativa plattan måste man utföra ett
arbete, eller om man så vill, öka dess (elektriska) lägesenergi. Frågan är hur stor elektronens ökning i
lägesenergi är när den når den negativa plattan? För att besvara frågan inför vi begreppet spänning.
Spänningen (U) mellan två punkter i ett elektriskt fält definieras som kvoten mellan den lägesenergi (E)
laddningen förlorar när den rör sig mellan punkterna och dess laddning (q), d v s:
Elektrisk spänning, exempel

Exempel:

Vilket arbete krävs för att föra elektronen (q = 1,6 · 10 -19 C) i kondensatorn ovan från den positiva till den
negativa plattan, om spänningen U mellan plattorna är 12 V?

Lösning:

Elektronens lägesenergi ökar med E, d v s med det krävda arbetet: E = q · U = 1,6 · 10 -19 · 12 =

= 1,92 · 10-18 J

Exempel:

Vilken hastighet får elektronen om den får falla fritt tillbaks till den positiva plattan?

Lösning:

Samband mellan fältstyrka och spänning


I ett homogent fält, t ex mellan två parallella metallplattor, är alltså den elektriska fältstyrkan:

där U är spänningen mellan plattorna och d är avståndet mellan plattorna. Fältet är riktat från plus till
minus, det vill säga samma riktning som kraften på en positiv provladdning verkar.

Vi kan nu ange enheten för fältstyrkan som 1V/m. Om vi utnyttjar tidigare definitioner får vi genom
dimension analys:

Detta är samma enhet som vi har använt tidigare.


Exempel:

Vilken spänning krävs för att hålla pingisbollen svävande i det tidigare exemplet, om avståndet mellan
plattorna är:

a) 1 dm?

b) 1 m?

Lösning:

Flytta punktladdningarna runt på spelplanen och visa sedan det elektriska fältet, spänningar,
ekvipotentiallinjer och mer. http://phet.colorado.edu/en/simulation/charges-and-fields

Spela hockey med elektriska laddningar. Placera laddningar på isen, slå och försök få pucken i mål. Visa
det elektriska fältet. Rita upp puckens rörelse. Gör spelet svårare genom att placera väggar framför mål.
http://phet.colorado.edu/en/simulation/electric-hockey

Millikans försök
Millikans försök som ledde till att elektronens laddning kunde bestämmas, utgör en milstolpe i fysiken.
Millikan kom fram till att elektronens laddning är den minsta laddningsenheten. Läs om Millikans försök
på https://sv.wikipedia.org/wiki/Millikans_oljedroppsf%C3%B6rs%C3%B6k

Du behöver inte kunna några detaljer om Millikans försök, men du bör försöka förstå de fysikaliska
resonemangen och beräkningarna.

Klicka på bilden för att själv utföra Millikans experiment!

http://www.learnerstv.com/animation/animation.php?ani=186 HYPERLINK
"http://www.learnerstv.com/animation/animation.php?ani=186&cat=Chemistry"& HYPERLINK
"http://www.learnerstv.com/animation/animation.php?ani=186&cat=Chemistry"cat=Chemistry

En oljedroppe med laddningen Q befinner sig svävande mellan plattorna. Den påverkas av två krafter:

Den elektriska kraften F = Q · E och tyngdkraften mg.

Eftersom fältet är homogent gäller:

Droppens laddning kan då uttryckas som:


Experiment visar att all laddning är en heltalsmultipel av en minsta laddning, den s k
elementarladdningen e = 1,602 ⋅10−19 C.

Jobba med övningsuppgifterna 6.18 - 6.21, sid 153, i läroboken.

Elektrisk ström
I det här avsnittet ska vi definiera elektrisk ström. Det kan vara svårt att skilja på ström och spänning.
Tänk på strömmen som en ström av elektroner som rör sig i ledaren. Du kan jämföra med vatten som
strömmar i en vattenledning.

Strömmen definieras som den laddning Q som passerar ett tvärsnitt av en ledning per tidsenhet:

Strömmen mäts i enheten ampere, 1 A. Ampere är en av grundenheterna i SI-systemet. Med hjälp av


definitionen av ström kan vi uttrycka enheten för laddning på följande sätt: 1 C = 1 As

Det finns två förutsättningar för att vi ska få en ström i en elektrisk krets:

● Kretsen måste vara sluten. Det får inte finnas något avbrott i kretsen. (Strömmen skall kunna gå runt i
kretsen.)

● En spänningskälla måste vara ansluten.

Strömriktningen definieras så som en tänkt positiv laddning skulle röra sig, det vill säga från plus till
minus. Egentligen är det elektronerna som rör sig i strömkretsen, och eftersom de är negativt laddade
rör de sig motsatt den riktning som en sådan positiv laddning skulle röra sig.

Mätning av ström och spänning


Studera de elektriska symbolerna och kretsarna på s 177-178 noga. Använd rätt symboler när du själv
ritar elektriska kretsar.

Du använder en voltmeter för att mäta spänningen och en amperemeter för att mäta strömmen i en
elektrisk krets. Mätinstrumenten måste anslutas på rätt sätt och ha sådana egenskaper att de inte
påverkar kretsen, eller påverkar så lite som möjligt.

En amperemeter ansluts i serie med komponenten i kretsen. Den mäter strömmen som går genom
komponenten. Amperemetern ska ha liten resistans för att inte påverka strömmen.

En voltmeter ansluts parallellt med komponenten i kretsen. Den mäter spänningen över en komponent.
Voltmetern ska ha stor resistans för att strömmen inte ska gå igenom den.

Se filmen om hur du kopplar in en voltmeter och en amperemeter!

https://www.youtube.com/watch?v=G_mheWS8svc
Likström och växelström
Likström, DC (direct current), är elektrisk ström om alltid har samma riktning. Växelström, AC
(alternating current), är en elektrisk ström vars riktning växlar. Fördelen med växelström är att
överföringen ger små energiförluster.

Vad är det som går i ledningar när vi säger att det går ström? Elektronerna ger sina laddningar till
varandra och den laddningsmängden som fortplantas kallas för ström. De fria, valenselektronerna börjar
röra på sig eftersom de känner av en laddning, en minusladdning i baken och en plusladdning framför.
Dessa laddningar kommer från batteriets plus och minuspol. Hur stora dessa laddningar är är
spänningen. Som jämförelse med en vattenledning , kan man säga att spänningen motsvaras av trycket i
vattenledningen skapat av pumpen eller höjdskillnaden och elektronernas laddning det strömmande
vattnet. Elektronerna kan inte röra på sig obehindrad eftersom de inte är ensamma i ledaren. Detta ger
upphov till ett motstånd, men mer om det lite senare.

Exempel: a) Strömmen i en koppartråd är 2,6 A. Hur stor laddning har passerat tråden på 5 s?

b) Hur många elektroner har passerat laddningen på samma tid?

Lösning:

Titta inuti ett batteri för att se hur det fungerar. Välj batterispänningen, flytta laddningar från ena änden
av batteriet till den andra. En voltmeter berättar den resulterande batterispänningen Klicka på bilden för
att köra simuleringen!

http://phet.colorado.edu/en/simulation/battery-voltage

Några övningar
1.) Spänningen mellan den stora och lilla bandgeneratorkulan är 5 kV. Anta att den stora kulan är negativ
och den lilla kulan positiv.

a) Bestäm den elektriska lägesenergin hos en elektron som befinner sig på den stora kulan.

b) Vilken hastighet har en elektron som lossnar från den stora kulan när den träffar den lilla kulan?

Lösning:

2.) Bestäm energiinnehållet i ett 12 volts bilbatteri, som stått på laddning i 12 h med laddningsströmmen
3,5 A. Svara i kWh.

Lösning:
Resistans
Läs om resistans och resistivitet sid 178 - 180 i läroboken.

Metaller är goda ledare av elektricitet men de har ändå ett visst motstånd mot strömmen. Motståndet
blir större ju längre ledaren är, och mindre ju tjockare ledaren är. Motståndet beror också på metallens
egenskaper som kallas för resistivitet. Motstånd eller resistans mäts i ohm. Enheten har fått sitt namn
efter den tyske fysikern Georg Ohm (1789-1854) och betecknas med bokstaven Ω (omega).

Resistansen i en komponent definieras som kvoten mellan spänningen över komponenten och strömmen
genom komponenten:

Enhet: Ω = V/A

Sambandet kan skrivas om så att vi får U = RI , vilket brukar kallas Ohms lag.

Om resistansen är konstant är spänningen direkt proportionell mot strömmen. Motstånd, resistorer, har
konstant resistans. En glödlampa har inte konstant resistans. Resistansen i glödlampan ökar när
strömmen ökar. Det beror på att resistansen ökar när temperaturen ökar.

Klicka på bilden för att köra simuleringen!

http://phet.colorado.edu/en/simulation/ohms-law

Resistivitet
Resistansen hos en metalltråd bestäms av sambandet:

där l är trådens längd, A är tvärsnittsarean och ρ är resistiviteten. Resistiviteten är en materialegenskap.

I formelsamlingen finns en tabell som visar resistiviteten hos några metaller och metallegeringar. Goda
ledare som koppar och silver har liten resistivitet.

Observera att resistiviteten anges i enheten Ω ∙ mm2/m. Areaenheten är alltså mm2. Det betyder att du
ska ange tvärsnittsarean i samma enhet, alltså mm 2, när du beräknar resistansen!

Exempel:

Beräkna resistansen för en 1,8 m lång konstantantråd med diametern 0,2 mm.

(Resistiviteten för konstantan finns i formelsamlingen.)

Lösning:
Läs om resistans och resistivitet sid 178 - 180 i läroboken.

Mer om resistivitet
Lär om motståndet i en tråd. Ändra dess resistivitet, längd och tvärsnittsarea för att se hur de påverkar
trådens motstånd. Storlekarna på symbolerna i ekvationen förändras tillsammans med diagrammet.

http://phet.colorado.edu/en/simulation/resistance-in-a-wire

Titta inuti en resistor för att se hur den fungerar. Öka batterispänningen för att öka elektronflödet genom
motståndet och öka motståndet för att blockera flödet av elektroner. Titta också på
temperaturändringen i motståndet.

http://phet.colorado.edu/en/simulation/battery-resistor-circuit

Titta också på ett filmklipp om resistans.

https://www.youtube.com/watch?v=HvmkeERl65c HYPERLINK "https://www.youtube.com/watch?


v=HvmkeERl65c&feature=youtu.be"& HYPERLINK "https://www.youtube.com/watch?
v=HvmkeERl65c&feature=youtu.be"feature=youtu.be

Seriekoppling
I det här avsnittet ska vi studera olika typer av elektriska kretsar. Vi skall analysera olika kretsar där flera
komponenter är anslutna i parallellkoppling eller seriekoppling.

Läs texten i läroboken s 182-187.

Seriekoppling:

Vid seriekoppling är strömmen lika stor i hela kretsen. Spänningen över varje resistor kan bestämmas
med hjälp av Ohms lag:

U1 = R1I och U2 = R2I

Spänningen över båda resistorerna tillsammans blir U = U 1 + U2 = R1I + R2I = (R1 + R2)I

Vi kan alltså ersätta de båda resistorerna med en resistor med resistansen R = R 1 + R2. Denna kallas
ersättningsresistans.

Vid seriekoppling av flera resistorer blir ersättningsresistansen lika med summan av resistanserna:
Parallellkoppling

Vid parallellkoppling delas strömmen upp i de parallella grenarna. Summan av grenströmmarna är lika
med huvudströmmen:

I = I1 + I2

Detta kallas Kirchhoffs 1:a lag.

Spänningen mäts alltid över en komponent. Vid parallellkoppling utgår de parallella grenarna från
gemensamma punkter. Då måste spänningen vara lika stor över varje gren.

Spänningen över R1 är: U = R1I1

Spänningen över R2 är: U = R2I2

För respektive grenström gäller:

Vi vill ersätta R1 och R2 med ett motstånd med resistansen R. Vi utnyttjar sambandet mellan huvudström
och grenströmmar:

Vi kan bestämma ersättningsresistansen vid parallellkoppling av flera motstånd enligt sambandet:

Effekt
Läs texten i läroboken sid 180 om effekt i resistorer. Elektrisk energi används till uppvärmning, belysning,
maskiner etc. överallt i samhället.

Den elektriska energin kan beräknas med hjälp av Joules lag:

För den elektriska effekten gäller då:

Generellt gäller att energiomsättningen kan beräknas som effekten gånger tiden: W = P · t

Det gäller oavsett vilken typ av energi det handlar om.


Effekten, P, talar om hur snabbt en elektrisk apparat kan omvandla elektrisk energi till någon annan
energiform. Effekten mäts i watt, W. Den elektriska effekten beror på hur stor strömmen och spänningen
är.

Den elektriska energin, E, bestäms av effekten hos en apparat och hur länge den är inkopplad. Ofta
betecknas energin med W istället för med E.

Energin mäts i joule, J. Inom elläran använder man ofta wattsekund, Ws, istället för joule. En wattsekund
är lika mycket som en joule. För att mäta större elektriska energier använder man kilowattimme, kWh.

Om effekten 1 watt är inkopplad i en sekund, har man förbrukat 1 Ws. Om effekten 1000 watt är
inkopplat under en timme, har man förbrukat 1 kWh.

Om man leder ström genom en ledare blir ledaren varm. Detta fysikaliska fenomen har utnyttjats i en hel
del enkla elektriska apparater. Några exempel är brödrost, strykjärn och elektriska element.

Effekt, några exempel


1) Bestäm energiinnehållet i ett 12 volts bilbatteri, som stått på laddning i 12 h med laddningsströmmen
3,5 A. Svara i kWh.

Lösning:

2.) Spänningen över lampan L1 är 12 V och över L2 8 V. Lamporna förbrukar tillsammans effekten 10 W.
Bestäm strömmen I genom kretsen.

Lösning:

3.) Spänningen över en 1500 W kokplattas nätanslutning är 230 V. Bestäm resistansen i den
värmealstrande kokplattan.

Lösning:

Exempel på kopplingar
1.) Se figur. Ersättningsresistansen är 75 Ω. R 1 = 120 ΩΩoch R2 = 50 Ω. Bestäm R3.
Lösning:

2.) Se figur. R1 = 125 Ω, R2 = 175 Ω och R3 = 225 Ω. Strömmen genom R2 är 150mA. Hur stor är den totala
spänningen i kretsen?

Lösning:

Lös övningarna 8.8 - 8.21 på sid 198-199 i läroboken.

Klicka på bilden till vänster och bygg din egen krets! http://phet.colorado.edu/en/simulation/circuit-construction-
kit-dc

Potential och elektromotorisk spänning


Läs texten i läroboken s 187-194.

Lös uppgifterna 8.23, 8.24, 8.27, 8.30, 8.32 och 8.35 i läroboken.

Som förberedelse för laborationen behöver du följande bakgrund (som inte ges lika detaljerat i
kursboken):

Den spänning som vi maximalt kan få ut från en spänningskälla, t.ex. ett batteri, kallas den
elektromotoriska spänningen, ems. I praktiken får vi inte ut så hög spänning. Det beror på energiförluster
i spänningskällan. Spänningskällan har en inre resistans, Ri.

Polspänningen är den spänning som vi får ut från spänningskällan. Polspänningen minskar när strömmen
ökar. Följande samband gäller mellan ems (E), polspänningen (U), den inre resistansen (R i) och strömmen
i kretsen (I):

Elektromotorisk spänning (ems):

Vi börjar med att mäta spänningen över en belyst obelastad fotocell. Vi låter sedan fotocellen driva en
liten fläkt och mäter samtidigt spänningen. Belysningen är densamma som tidigare. Det visar sig att
spänningen över fotocellen sjunker när fläkten kopplas in. Sedan övergår vi till att mäta spänningen på
ett obelastat batteri, den s.k. tomgångsspänningen. Vi kopplar sedan in en liten glödlampa i serie med
batteriet. Batterispänningen sjunker. Den kallar vi för polspänning. Man kan undra varför spänningen
sjunker över en fotocell eller över ett batteri som belastas? Skälet är att fotocellen och batteriet har
resistans. Går det ström genom dem blir det ett spänningsfall över resistansen. Alltså sjunker spänningen
något. Spänningen som mäts när det inte går någon ström kallas tomgångsspänning, elektromotorisk
spänning (ems) eller elektromotorisk kraft (emk).

Ett batteri kan tänkas bestå av en spänningskälla (E) seriekopplad med batteriets inre resistans (R i)

Elektrisk potential
Begreppet potential och spänning hänger tätt samman. Spänning mäts ju mellan två punkter, t ex över
en glödlampa. I vissa sammanhang är det lämpligt att alltid ange spänningen till en referenspunkt.
Referenspunktens potential sätts då till 0 V.

I vägguttaget är det jorden som utgör referenspunkten 0 V. Storheten potential mäts alltså i enheten
volt. Begreppet potential har även viss koppling till potentiell energi. Potentiell (läges-) energi mäts ju
alltid i förhållande till en nollnivå. I elsammanhang anges den elektriska lägesenergin i förhållande till
jorden. Man brukar dock sällan beräkna den elektriska lägesenergin. I elsammanhang är det istället
potential som gäller. Skillnad i potential är spänning.

Exempel:

Bilden visar två plattor med olika potential V. Den undre plattan är jordad och har alltså potentialen 0 V.
Vilken elektrisk lägesenergi har den positiva laddningen q = 1,6nC om den befinner sig på plattan vid:

a) A?

b) B?

c) Bestäm spänningen mellan plattorna.

Lösning:

Laddningens elektriska lägesenergi E = qV dvs laddningen multiplicerad med potentialen vid laddningen.

a) EA = q · VA = 1,6 · 10-19 · 0 J = 0 J

b) EB = q · VB = 1,6 · 10-19 · 5 J = 8 · 10-9 J

c) Spänningen U är skillnad i potential: UAB = VB - VA = 5 - 0 = 5 V


Potential i elektriska kretsar
Begreppet potential har kanske störst tillämpning i elektriska kretsar. I en bils elsystem t ex ges kaross
och motor (som är anslutna till batteriets minuspol) potentialen 0 V, som alltså är bilens jord. Skall man
sen koppla in en lampa t ex räcker det med en ledare från batteriet till lampan. Återledaren ansluts till
närmaste jord, t ex kaross eller motor. Begreppet potential används även för att förenkla beräkningar i
kretsar.

För att bestämma t.ex. strömmen i en krets kan man göra en ”potentialvandring”. Spänningen i ett
obelastat batteri kallas som vi har sett ”elektromotorisk spänning” (ems). Det är det värdet på
spänningen som står på batteriet. Batteriet har också en inre resistans. Om vi går ett varv i kretsen måste
summan av potentialerna vara noll eftersom vi startar och slutar i samma punkt. Detta kallas även
Kirchoffs andra lag. Går vi mot strömmen i en resistor ökar potentialen, går vi med strömmen, minskar
potentialen.

Titta på en genomgång genom att klicka på bilden nedan!

https://www.youtube.com/watch?v=B4v99dXwQik HYPERLINK "https://www.youtube.com/watch?


v=B4v99dXwQik&feature=youtu.be"& HYPERLINK "https://www.youtube.com/watch?
v=B4v99dXwQik&feature=youtu.be"feature=youtu.be

Laboration nr 3
I samband med denna modul skall du genomföra två laborationer, och det är nu dags för den första som
är en webblaboration.

Laborationsinstruktion och rapportmall hittar du genom att klicka på knapparna nedan. (Du kan också
komma åt dessa om du klickar på "Laborationer" i Introduktionsmodulen eller i Kursbiblioteket sist i
denna studieguide.)

Du skall nu göra Laboration nr 3!

Laboration nr 3: Elektricitet - batteri

http://novo.hermods.se/pluginfile.php/270167/mod_page/content/43/Elektricitet%20-
%20batteri.pdf

Film till Lab nr 3

https://vimeo.com/202366338

Rapport: Bifogas med Uppdrag 3

http://novo.hermods.se/pluginfile.php/270167/mod_page/content/31/1.%20Rapportmall%20med
%20checklista%2C%20rev.pdf
Sammanfattning, avsnitt 7
Sammanfattning ellära:

Du ska kunna följande om elektricitet:

● Elektrisk laddning, kraftverkan mellan laddningar och elektrisk influens

● Coulombs lag

● Definitionen av potential, elektrisk spänning och elektrisk fältstyrka

● Elementarladdningen

● Definitionen av elektrisk ström

● Kunna rita elektriska kopplingsscheman på ett korrekt sätt

● Kunna ansluta mätinstrument för att mäta ström och spänning i en elektrisk krets

● Resistans och resistivitet

● Serie- och parallellkoppling av resistorer

● Ohms lag

● Samband mellan elektromotorisk spänning och polspänning

● Beräkning av energi- och effektutveckling i elektriska kretsar

● Repetera de övningsuppgifter som hänvisats till i föregående text.

● Repetera lärobokens lösta exempel, samt studera och lös kontrollfrågorna under de sidor som du läst i
läroboken för detta avsnitt.

● Läs sammanfattningarna i kursboken på s 196 i läroboken.

● Gör Testa-dig-själv 3:1.

Avsnitt 8: Termofysik
Termofysik handlar om begrepp som tryck, temperatur och värme. I det här kapitlet ska vi gå igenom hur
dessa begrepp definieras. Vi kommer bl. a. att diskutera varför båtar flyter och varmluftsballonger stiger
upp i luften. Vi ska också studera energiflödena vid uppvärmning och avsvalning. Vi börjar med att
behandla tryck, tryckkraft och lyftkraft för fasta material, vätskor och gaser. Läs s 46-61 i din lärobok.
Tryck
Tryck (P) definieras som kraft (F) per areaenhet (A), dvs. kraften dividerad med arean, där kraften verkar
vinkelrät mot arean:

!!! SI-enheten för tryck är Pascal (Pa). 1 Pa = 1 N/m 2

Ett tryck på 1 pascal betyder att kraften 1 N trycker på arean 1 m 2.

I alla formler i fysiken måste vi använda enheten Pa för tryck, annars funkar det inte.

Liten yta - stort tryck:

Ibland vill man ha största möjliga tryck. Att skära bröd med en slö kniv är nästan omöjlig och att få en
spruta med en tjock nål är ganska smärtsamt. En nål har en riktigt tunn spets. Den orsakar ett stort tryck
med relativ liten kraft eftersom ytan är liten.

Trycket 1 Pa är ett relativt litet tryck. Ett enkelt A4-papper som ligger på bordet, utövar ett tryck på
knappt 1 Pa. Det kan vi visa med följande beräkning:

Ett A4-papper har måtten 21,0 cm ∙ 29,6 cm och väger 4,97 g. Trycket blir då:

Klicka på filmsymbolen för att se en genomgång av tryck!

https://www.youtube.com/watch?v=G0wUIeoZukI HYPERLINK "https://www.youtube.com/watch?


v=G0wUIeoZukI&list=PLBRUvF-imZr_fyinJfiLWTUrEPDXwFgHC&feature=share&index=1"& HYPERLINK
"https://www.youtube.com/watch?v=G0wUIeoZukI&list=PLBRUvF-
imZr_fyinJfiLWTUrEPDXwFgHC&feature=share&index=1"list=PLBRUvF-imZr_fyinJfiLWTUrEPDXwFgHC
HYPERLINK "https://www.youtube.com/watch?v=G0wUIeoZukI&list=PLBRUvF-
imZr_fyinJfiLWTUrEPDXwFgHC&feature=share&index=1"& HYPERLINK
"https://www.youtube.com/watch?v=G0wUIeoZukI&list=PLBRUvF-
imZr_fyinJfiLWTUrEPDXwFgHC&feature=share&index=1"feature=share HYPERLINK
"https://www.youtube.com/watch?v=G0wUIeoZukI&list=PLBRUvF-
imZr_fyinJfiLWTUrEPDXwFgHC&feature=share&index=1"& HYPERLINK
"https://www.youtube.com/watch?v=G0wUIeoZukI&list=PLBRUvF-
imZr_fyinJfiLWTUrEPDXwFgHC&feature=share&index=1"index=1

Vätsketryck och lyftkraft


Alla som dykt under vatten vet att trycket ökar med djupet. Men hur ökar det och är trycket olika vid
samma djup om man t ex dyker i en smal brunn eller en sjö?

Exempel: I vilket av nedanstående vattenfyllda kärl är trycket störst vid botten?


Lösning: Om vi mäter trycket vid botten på samtliga kärl med en digital manometer så ser vi att trycket är
samma vid bottnen på samtliga kärl. Slutsatsen är alltså att trycket är oberoende av kärlets form och
endast beroende av avståndet till vattenytan.

Exempel: Hur stort är trycket på ett visst djup i en vätska och vad beror trycket på?

Lösning: Anta ett djup h och ett tvärsnittsarea A som vattenpelaren med densiteten ρρ trycker på. Ställ
sedan upp ett samband (med bokstäver!) för trycket p:

Vätsketryck och lyftkraft, forts


Exempel: Titanic ligger på djupet 2000 m utanför USA:s östkust. Hur stort är trycket på det djupet?

Lösning:

Titta på genomgången genom att klicka på filmsymbolen:

https://www.youtube.com/watch?v=QVf8tGqZN2o HYPERLINK "https://www.youtube.com/watch?


v=QVf8tGqZN2o&list=PLBRUvF-imZr_fyinJfiLWTUrEPDXwFgHC&feature=share"& HYPERLINK
"https://www.youtube.com/watch?v=QVf8tGqZN2o&list=PLBRUvF-
imZr_fyinJfiLWTUrEPDXwFgHC&feature=share"list=PLBRUvF-imZr_fyinJfiLWTUrEPDXwFgHC HYPERLINK
"https://www.youtube.com/watch?v=QVf8tGqZN2o&list=PLBRUvF-
imZr_fyinJfiLWTUrEPDXwFgHC&feature=share"& HYPERLINK "https://www.youtube.com/watch?
v=QVf8tGqZN2o&list=PLBRUvF-imZr_fyinJfiLWTUrEPDXwFgHC&feature=share"feature=share

Trycket i en vätska är proportionellt mot vätskans densitet och vätskedjupet:

Ett föremål som är nedsänkt i en vätska kommer då att utsättas för en större tryckkraft mot undersidan
jämfört med översidan. Den resulterande uppåtriktade kraften brukar kallas lyftkraft. På samma sätt har
alla vätskor och gaser en lyftkraft. Luftens lyftkraft räcker för att en ballong med heliumgas ska stiga
uppåt. Helium har mindre täthet än luft.

Arkimedes princip beskriver hur lyftkraften verkar: "Ett föremål som är nersänkt i en vätska påverkas av
en lyftkraft som är lika stor som tyngden av den undanträngda vätskan."

Om man lägger ett föremål i vatten har vattnet en lyftkraft som man kan räkna ut:

1. Man tar reda på hur mycket vattnet stiger. Då vet man föremålets volym.

2. Sedan räknar man ut hur mycket samma volym vatten väger och vilken tyngd det har.
3. Vattnets lyftkraft är lika stor som tyngden av vattnet som föremålet har trängt undan.

Exempel på lyftkraft
Exempel: En 1,5 m lång träbjälke med tvärsnittsarean 10cm∙10cm flyter i vattnet. En tredjedel av
träbjälken sticker upp ovanför vattnet. Bestäm träbjälkens densitet.

Lösning: De krafter som verkar på bjälken är tyngdkraften, mg, och vätskans lyftkraft, F 0. Eftersom bjälken
flyter har vi kraftjämvikt, det vill säga mg = F0.

Vattnets densitet: ρvatten = 1000 kg/m3

2/3 av träbjälken är nedsänkt i vattnet. Volymen av den undanträngda vattnet blir:

Vvatten = 2/3 · 0,10 · 0,10 · 1,5 m3 = 0,010 m3

Träbjälkens massa kan tecknas:

mträ = ρträ · Vträ , där Vträ = 0,10 · 0,10 · 1,5 m3 = 0,015 m3

Med hjälp av kraftjämvikt får vi följande ekvation:

Fler exempel på lyftkraft


Här följer tre exempel. Försök att lösa uppgifterna själv, innan du tittar på

lösningen!

1.) En låda med massan 62,5 kg och volymen 0,035 m 3 hänger i en stålvajer. Hela lådan hänger nedsänkt i
vatten. Hur stor är spännkraften i stålvajern?

Lösning:

2.) En rektangulär träbit med massan 735 g och basarean 9,3 cm x 30 cm flyter i vatten enligt figuren.
a) Vilka krafter verkar på träbiten?

b) Hur djupt sjunker träbiten ner under vattnet?

c) Vad väger det av träbiten undanträngda vattnet?

Lösning:

3.) En gummibåt med massan 60 kg och volymen 1,2 m 3 flyter i saltvatten

med densiteten 1035 kg/m3.

a) Bestäm lyftkraften på båten när den är tom.

b) Hur många personer med massan 75 kg kan åka i båten?

Lösning:

Gastryck och lufttryck


Lufttrycket är ca 100 kPa.

Lös uppgifterna 3.12-3.25 och 3.28.

Runt Jorden finns atmosfären som inehåller en massa luft. Luften väger och trycker mot Jordens yta med
en kraft av 100 000 N på varje m2, alltså 100 000 Pa. Det motsvarar 10 m vatten ovanför oss.

Vingarna på ett flygplan är formade så att lufttrycket blir mindre på ovansidan än på undersidan när
planet flyger. Detta för att luften behöver gå en längre väg på ovansidan än på undersidan. När det blåser
mycket hårt kan takpannorna blåsa av husen på grund av att trycket ovanpå huset blir mindre är inuti
huset. Dvs. trycket inifrån huset som lyfter pannorna, inte vinden i sig.

Vid havsytan är normalt lufttryck 101,3 kPa. Lufttrycket mäts ofta i enheten millibar (mbar). Normalt
lufttryck är 1013 mbar. Förr mättes lufttrycket i mm Hg (kvicksilver). Denna enhet kallas även torr. Det
normala lufttrycket är 760 torr.
Exempel: Vattennivån i havet påverkas av lufttrycket.

Hur mycket skiljer sig vattennivån mellan lågtryck (960 mbar) och högtryck (1060 mbar)?

Lösning:

Tryckenheter:

1 atmosfär = 1 bar = 100 000 Pa

760 mm Hg = 1 013 mbar = 1 013 hPa = 101 300 Pa

Minilaboration
När kommer objekten flyta och när kommer de att sjunka? Lär dig hur lyftkraft fungerar med block.
Pilarna visar de applicerade krafterna, och du kan ändra egenskaperna hos blocken och vätskan.

Klicka på bilden nedan för att utföra minilaborationerna om lyftkraft och tryck!

http://phet.colorado.edu/en/simulation/buoyancy

Temperatur
Läs texten i läroboken s 204-218, så att du känner igen de begrepp vi tar upp i detta av-

snitt. Du känner säkert redan till hur man mäter temperatur med celsiusskalan. Kelvinskalan har lika
stora skalsteg som celsiusskalan, men temperaturen noll Kelvin, dvs. 0 K (OBS kelvin skrivs utan symbolen
för grader) motsvarar -273,16°C, det vill säga absoluta nollpunkten. Läs mer om temperaturskalor längre
ner.

0 K = -273,16 °C

När du räknar med temperaturdifferenser spelar det ingen roll om du använder celsiusskalan eller
kelvinskalan, men i alla andra sammanhang ska du använda kelvinskalan när du gör beräkningar i fysik
(och kemi). Med andra ord, båda skalorna har lika stora avstånd mellan graderingar.

Det finns ingen gräns, enligt vad vi vet, för hur varmt något kan bli. Det handlar om tillgången på energi.
Den lägsta temperatur däremot som vi kan uppnå är -273 °C. Detta kallas den absoluta nollpunkten.
Lägre temperatur kan inte finnas eftersom då skulle all rörelse stanna och alla atomer skulle kollapsa.
Världen som vi känner till skulle inte finnas. Termodynamik handlar bland annat om sambandet mellan
värme och energi. Typiska termodynamiska frågor kan t ex vara hur mycket energi som krävs för att höja
temperaturen på 2 liter vatten eller 5 kg järn från 20 till 60 ℃ eller hur mycket energi som krävs för att
smälta 3 kg is?
Skalan visar temperaturen:

Anders Celsius gjorde sin termometerskala med hjälp av vatten. Vattnets fryspunkt är noll grader på
skalan och kokpunkten är 100 °C.

I främst USA och Storbritannien används Fahrenheit-termometern. Fahrenheit bestämde nollpunkten


med hjälp av en blandning av is och koksalt. Ren is smälter vid +32 °F och vattnets kokpunkt är +212 °F.
Fahrenheit grader används inte i fysikaliska sammanhang.

Kelvin skalan har noll grader på absoluta nollpunkten. Eftersom detta är den lägsta möjliga
temperaturen, finns inga minusgrader på Kelvin skalan. I alla formler där temperatur ingår ska man
använda temperaturer i grader Kelvin. Vattens fryspunkt är 273 K.

Tillståndslagen för ideala gaser


S 223-224. Tillståndslagen för gaser ger ett samband mellan tryck, volym och temperatur hos en viss
bestämd gasmängd:

Observera att temperaturen måste anges i kelvinskalan. Ofta är en av storheterna p, V eller T konstant.

Om temperaturen är konstant får vi Boyles lag: p · V = konstant

Om volymen är konstant får vi Charles lag: p/T = konstant

Exemplet i läroboken visar hur du använder gaslagarna vid praktisk problemlösning.

Lös uppgift 10.1 på s 243 i läroboken.

Exempel på Boyles lag: En väderballong fylld med helium har radien 1,5 m och har ett övertryck på 400
kPa när den släpps vid jordytan. Vid en viss höjd är övertrycket i ballongen 50 kPa. Hur stor är diametern
då?

Lösning:

Värme
Temperaturen är ett mått på molekylernas kinetiska energi. Denna energi kallas termisk energi. När du
tillför energi utifrån kommer den inre energin hos systemet att öka. Energin kan tillföras som
värmeenergi eller som någon form av arbete. När till exempel en bil bromsar in blir broms klossarna
varma och temperaturen ökar. Bromsarbetet leder till att den inre energin hos broms klossarna ökar.
Värme, eller värmeenergi, överförs från något som har högre temperatur till något som har lägre
temperatur. Värme kan aldrig spontant överföras från ett föremål med lägre temperatur till ett föremål
med högre temperatur.

Värme är rörelse

Värme är rörelse hos molekylerna. Att ett ämne förlorar värme betyder att molekylerna rör sig
långsammare.

Exempel: Tänk dig att du lägger en isbit i en isolerad termos som innehåller vatten. Vad händer? Isbiten
smälter. Efter en stund har hela isbiten smält och vatten i termosen har fått litet lägre temperatur. Detta
sker isolerat i termosen, utan att energi avges eller tas upp från

omgivningen. Från början hade vattnet högre temperatur än isbiten. När isbiten har smält har
blandningen samma temperatur. Värmeenergi har överförts från vattnet till isbiten. Isbiten har
absorberat, tagit emot, värmeenergi från vattnet. Blandningen har fått samma temperatur. Vi har fått
termisk jämvikt.

I exemplet ovan är det inte svårt att inse att det motsatta förloppet är orimligt. Vatten kan inte övergå i is
utan att energiutbyte med omgivningen sker. Värme överförs genom värmeledning, värmestrålning eller
värmeströmning. Värmeöverföringen pågår tills det har blivit termisk jämvikt.

Värme, forts
Värmeenergi betecknas oftast med Q. Precis som annan energi mäts värmeenergi i enheten joule (J).

Vi måste göra en skillnad mellan värme och temperatur!!!

Värme är energi. Temperatur är ett mått på hur mycket energi något innehåller.

Ju fortare de enskilda atomerna rör sig i gaser eller vätskor desto högre är deras kinetisk energi. Hos ett
fast ämne ökar vibrationen hos atomerna när temperaturen stiger. Begreppet värme förklaras inom
fysiken som: Energi som strömmar från en varmare del till en kallare del av ett föremål. Föremålets totala
inre energi är oförändrad.

Värmekapacitet
Du hittar mer om begreppen kring värmekapacitet i läroboken s 207 - 211.

Förmågan att lagra inre energi varierar mellan olika ämnen. Den specifika värmekapaciteten, som är en
materialkonstant, beskriver denna egenskap hos olika ämnen.

Den specifika värmekapaciteten är ett mått på hur mycket energi som måste tillföras per kilogram för att
temperaturen ska öka med 1K (eller 1°C). I din formelsamling kan du studera den specifika
värmekapaciteten för vanliga fasta ämnen och vätskor. Du kan speciellt notera att vatten (vätska) och is
(fast ämne) har olika specifik värmekapacitet. Enheten för värmekapacitet är J/(kg∙K).

Energiomsättning:

Ett ämne med massan m och specifika värmekapaciteten c tillförs värme så att temperaturökningen blir
ΔT. Den energi som går åt är:

Gå igenom exempel 1 och 2 i läroboken, s 208-209.

Jämförs olika ämnens specifika värme visar sig vatten ha högsta värdet. Uppenbarligen är vatten ett bra
medium att lagra energi i.

1 kWh = 3,6 MJ

Titta på filmen om omvandling från Joule till kWh.

https://www.youtube.com/watch?v=LVLDX05vwm0 HYPERLINK "https://www.youtube.com/watch?


v=LVLDX05vwm0&feature=youtu.be"& HYPERLINK "https://www.youtube.com/watch?
v=LVLDX05vwm0&feature=youtu.be"feature=youtu.be

Värmekapacitet, exempel
1.) En tempererad simbassäng har måtten 25 x 17 x 1,6 m. Vattnets temperatur skall efter rengöring av
bassängen höjas från 8° till 22°.

a) Beräkna energiåtgången.

b) Vad kostar uppvärmningen, om 1 kWh kostar 60 öre?

Lösning:

2.) Tre liter 90-gradigt vatten hälls i ett kar med femton liter 12-gradigt vatten. Bestäm vattnets
blandningstemperatur.

Lösning:

Laboration nr 4
I samband med denna modul skall du genomföra två laborationer, och det är nu dags för den andra som
är en hemlaboration.

Laborationsinstruktion och rapportmall hittar du genom att klicka på knapparna nedan. (Du kan också
komma åt dessa om du klickar på

"Laborationer" i Introduktionsmodulen eller i Kursbiblioteket sist i denna studieguide.)

Du skall nu göra Laboration nr 3!

Laboration nr 4: Specifik värmekapacitet

https://novo.hermods.se/pluginfile.php/452543/mod_page/content/70/Specifik%20v
%C3%A4rmekapacitet.pdf

Rapport: Bifogas med Uppdrag 3

http://novo.hermods.se/pluginfile.php/270167/mod_page/content/31/1.%20Rapportmall%20med
%20checklista%2C%20rev.pdf

Faser och fasövergångar


När vatten fryser blir det is och när det kokar bildas ånga. Man säger att vatten, liksom andra ämnen, har
tre faser: fast, flytande och gas. När is smälter sker en övergång från fast fas till flytande fas, en s.k.
fasövergång. Det sker också en fasövergång när vatten kokar till ånga. Att åstadkomma fasövergångar är i
regel betydligt mer energikrävande än att höja temperaturen på ett ämne i samma fas.

Värm, kyl och komprimera atomer och molekyler och titta på när de byter mellan fast, flytande och gas
faser. Klicka på bilden nedan:

http://phet.colorado.edu/en/simulation/states-of-matter-basics

Smältning och förångning


När ett ämne smälter går det åt energi för själva smältningen. Om du har nollgradig is, måste du tillföra
energi för att få nollgradigt vatten. På motsvarande sätt måste det tillföras energi för att överföra
kokande vatten till vattenånga.

Ett ämnes smältvärme anger hur mycket energi som går åt för att överföra 1 kg av ett ämne från fast till
flytande form. Ångbildningsvärmen anger på motsvarande sätt hur mycket energi som går åt för att
överföra 1 kg av ett ämne från flytande form till gasform. Båda dessa storheter finns i din
formelsamlingen Enheten är J/kg.

Studera grafen nedan extra noga. Den visar hur temperaturen ökar vid konstant energitillförsel. I detta
fall är det is som värms upp och smälter. Vattnet värmes sedan upp och förångas. Under smältningen
respektive förångningen ökar inte temperaturen.
Smältning och förångning, forts
Att ett ämne är fast i rumstemperatur betyder att temperaturen måste vara högre för att det ska smälta.
Guld smälter när det värms till +1 063°C. Smält guld stelnar också vid +1 063°C. Den här temperaturen
kallas guldets smältpunkt.

I en blandning av vatten och is har både vattnet och isen temperaturen 0°C. Man kan fråga sig hur
mycket energi som måste tillföras för att den nollgradiga isen skall smälta? Det är rimligt att anta att det
går åt dubbelt så mycket energi för att smälta dubbelt så mycket is. Vid fasövergångar är temperaturen
konstant. Allmänt gäller att för att smälta ett ämne med massan m krävs smältenergin:

Es = m · cs

Konstanten cs (J/kg) är ämnets specifika smältentalpi och finns angivet i formelsamlingen. Omvänt
frigörs samma mängd energi när ämnet stelnar.

Exempel: Hur mycket energi går åt för att smälta 2,5 kg nollgradig is?

Lösning:

Smältning och förångning, exempel


Förångning

Den temperatur ett ämne har när det bildar gas kallar vi ämnets kokpunkt. Kokpunkten behöver inte vara
plusgrader. Syre har en kokpunkt på -183 °C. På samma sätt som vid smältning av is åtgår energi vid
förångning av vatten. Om m är massan på ämnet som skall förångas krävs ångbildningsenegin:

E k = m ∙ ck

Konstanten ck (J/kg) är ämnets specifika ångbildningsentalpi och finns angivet i formelsamlingen. Omvänt
frigörs samma mängd energi när ånga kondenserar, dvs övergår från gas till vätskefas.

Exempel: Hur många liter hundragradigt vatten kan 1 kWh koka bort?

Lösning:

Exempel: Man häller i 0,75 liter vatten i en kastrull för att koka potatis. Kastrullen ställs på snabbplattan
med maximalt effekt. Plattan avger då effekten 1500 W. Man väntar tills vattnet kokat upp och går sedan
till affären, men glömmer att vrida ner effekten på plattan. Hur lång tid tar det minst innan allt vatten
kokat bort?

Lösning:
Det behövs mycket värme för att värma vatten och för att is ska smälta. Detta gör att våren kommer
senare vid kusterna. Det tar helt enkelt längre tid att värma upp havet än markytan.

Jobba med övningarna 9.1 - 9.12, sid 219 - 220, i läroboken.

Energikvalitet
I det här avsnittet kommer du att läsa om praktiska tillämpningar som visar hur energi används och
omvandlas i samhället. Begreppet verkningsgrad, som vi har diskuterat redan i avsnitt 5, definieras också
i avsnittet om värmepumpar.

Verkningsgraden hos en maskin definieras på följande sätt:

Energi i form av elström är ju lättare att utnyttja än t ex energin i 12 gradigt vatten. Man talar därför om
olika energikvalitet. El och mekanisk energi är extra prima energi, som kan användas direkt utan
förluster. Kemisk- och kärnenergi, samt värme vid hög temperatur är prima energi, som kan omvandlas
till extra prima med förluster. Värme vid låg temperatur är sekunda energi (spillvärme) som är svår att dra
nytta av mer än som t ex värme i ett hus.

Det är inte energieffektivt att omvandla extra prima el direkt till sekunda värme i ett hus. Elen kan
dessutom vara framställd i ett oljekondenskraftverk med endast 40 % verkningsgrad. Då är det bättre att
värma huset med spillvärme från en industri eller prima energi från biobränsle. Det bästa är kanske att
låta el driva en värmepump som ”suger ut” lågvärdig energi ur luften, marken eller en sjö. Genom att
tillföra 1 kWh el kan man i bästa fall få ut 3 kWh energi.

Energikvalitet, forts
● Energi har olika kvalitet

● Mekanisk och elektrisk energi har hög kvalitet eftersom de nästan fullständigt kan omvandla till
varandra och till värme

● Värmeenergi har låg kvalitet eftersom bara en liten del kan omvandlas till andra energiformer. T ex
kan vi inte utnyttja värmen som finns i sjöar i någon större utsträckning, men vi kan omvandla vattnets
lägesenergi

Enligt första huvudsatsen kan energi omvandlas mellan olika energiformer. Endast elektrisk energi kan
rent praktiskt omvandlas till mekaniskt arbete (mer än 90 %) och omvänt. Det är dock mindre än hälften
av den kemiska energin, som t.ex. i olja kan omvandlas till mekanisk energi, enligt den 3:e huvudsatsen.
Det beror på temperaturskillnaden vid energiomvandlingen.

Exempel: I kärnkraftverk är ångans temperatur 280℃ och kyltemperaturen i kondensorn 20℃.


Carnots formel för verkningsgrad ger:

Det betyder att högst ca 40 % av bränslets energi omvandlas till elektrisk energi. Om man använder
kylvattnet för fjärrvärme, t.ex. kan man öka verkningsgraden till ca 80 %. Som jämförelse har
bränslecellerna en verkningsgrad på 83-85%.

Entropi
Naturen strävar efter så låg entalpi och så hög entropi som möjligt.

Entalpi är ett mått på mängden energi i ett system. Entropi är ett mått på mängden "oordning" i ett
system.

Titta på hur olika typer av molekyler bildar ett fast, flytande eller gasformigt ämne. Titta också på en
genomgång av entalpi och entropi. Klicka på bilderna nedan:

http://phet.colorado.edu/en/simulation/states-of-matter

https://www.youtube.com/watch?v=qMEmShkl_FE HYPERLINK "https://www.youtube.com/watch?


v=qMEmShkl_FE&feature=youtu.be"& HYPERLINK "https://www.youtube.com/watch?
v=qMEmShkl_FE&feature=youtu.be"feature=youtu.be

Sammanfattning, avsnitt 8
Du ska kunna följande om tryck och temperatur:

*) Definition av tryck

*) Vätsketryck

*) Arkimedes princip

*) Samband mellan kelvinskalan och celsiusskalan

*) Tillståndslagen för gaser

Du ska kunna följande om värme:

*) Specifik värmekapacitet

*) Ångbildningsvärme och smältvärme

*) Kunna tillämpa energilagen vid problemlösning som handlar om värmeenergi

*) Verkningsgrad
● Repetera de övningsuppgifter som hänvisats till i föregående text.

● Repetera lärobokens lösta exempel, samt studera och lös kontrollfrågorna under de sidor som du läst i
läroboken för detta avsnitt.

● Läs sammanfattningarna på s 61 och s 218 i läroboken.

Titta också på filmerna här till höger, samt de fem första genomgångarna under "Thermodynamics".

https://www.youtube.com/watch?v=LQQNw0Prmcw HYPERLINK "https://www.youtube.com/watch?


v=LQQNw0Prmcw&feature=youtu.be"& HYPERLINK "https://www.youtube.com/watch?
v=LQQNw0Prmcw&feature=youtu.be"feature=youtu.be

https://www.youtube.com/watch?v=VtEqn-5XHpU HYPERLINK "https://www.youtube.com/watch?


v=VtEqn-5XHpU&feature=youtu.be"& HYPERLINK "https://www.youtube.com/watch?v=VtEqn-
5XHpU&feature=youtu.be"feature=youtu.be

https://www.khanacademy.org/science/physics/thermodynamics

Avsnitt 9: Klimat och väder


Klimat, miljö och energi handlar om hur energiförsörjningen fungerar, energins roll i samhället och hur
klimatet påverkas av vår energianvändning.

Väder är det som utspelar sig utanför ditt fönster, i mindre områden under några timmar och dagar. När
vi pratar om klimat då pratar vi om större områden och under längre perioder, t.ex decennier.
Människorna håller just nu på att ändra världens klimat genom stora och ökande utsläpp av
växthusgaser.

Titta på filmen "Klimatförändringar"!

https://www.youtube.com/watch?v=r6uMUJfYiM4 HYPERLINK "https://www.youtube.com/watch?


v=r6uMUJfYiM4&feature=youtu.be"& HYPERLINK "https://www.youtube.com/watch?
v=r6uMUJfYiM4&feature=youtu.be"feature=youtu.be

Om vi kunde utnyttja bara en förvinnande liten del av solenergin skulle man lösa hela världens
energiproblem. Det finns stora möjligheter i energin från solen. Det skulle t.ex. räcka med att täcka 5 %
av Saharas yta med solceller för att tillgodogöra hela jordens energibehov.

I solen sker kärnreaktioner, fusion. Energi som frigörs skickar solen ut i form av strålningsenergi. En del av
solens strålningsenergi finns lagrad i fossila bränslen. Från jorden strålar det ut ungefär lika mycket
energi som den tar emot.

Det som vi kallar väder är händelser i atmosfären. Varm luft stiger uppåt och kall luft sjunker.
Luftströmmar bildas i atmosfären och värmen sprids genom strömning. Information om temperatur,
luftfuktighet, vindhastighet, lufttryck och solstrålning samlas med hjälp av sensorer och bearbetas med
hjälp av superdatorer. Dessa beräkningar utgör grunden för väderrapporterna.

https://youtu.be/u4ViiZTpvYc

Klimat och väder, forts


När jorden värmts av solljuset sänder den ut värmestrålning. Största delen av atmosfären består av syre
och kväve som släpper värmestrålningen igenom. I atmosfären finns också en del vattenånga, koldioxid
eller metan som absorberar strålningen och ökar sina rörelser. På så sätt sprids värmestrålningen åt alla
håll. En del studsar tillbaka till marken som blir varmare. Det är detta som är växthuseffekten. Om vi
räknar med den del som rymmer 99 % av atmosfärens massa är atmosfären ca. 30 km tjock. Utan
atmosfär skulle Jordens medeltemperatur ligga långt under vattnets fryspunkt ca -15 °C. Nu ligger den
kring +15 °C.

Problemet idag är inte växthuseffekten i sig utan den skenande och accelererande växthuseffekten. Vi
släpper ut mer och mer växthus gaser i atmosfären och värmen strålar ut i rymden ännu långsammare.
Detta gör att atmosfären värms upp mer och mer. Vi kan nå en "point of no return" inom en snar framtid,
med katastrofala följder.

Hur påverkar växthusgaserna klimatet? Klicka på bilden till vänster. Upptäck atmosfären under istiden
och idag. Vad händer när du lägger moln? Ändra växthusgashalten och se hur temperaturen förändras.
Jämför sedan effekten av glasrutor. Zooma in och se hur ljus interagerar med molekyler. Bidrar alla
atmosfäriska gaser till växthuseffekten? Utför minilaborationen genom att klicka på bilden i mitten. Titta
också på animationen längst till höger.

https://www.youtube.com/watch?v=ROZJmX73FF4 HYPERLINK "https://www.youtube.com/watch?


v=ROZJmX73FF4&feature=youtu.be"& HYPERLINK "https://www.youtube.com/watch?
v=ROZJmX73FF4&feature=youtu.be"feature=youtu.be

http://phet.colorado.edu/en/simulation/greenhouse

https://www.youtube.com/watch?v=Zpu7IZcdzXE HYPERLINK "https://www.youtube.com/watch?


v=Zpu7IZcdzXE&feature=youtu.be"& HYPERLINK "https://www.youtube.com/watch?
v=Zpu7IZcdzXE&feature=youtu.be"feature=youtu.be

Klimat och väder, instuderingsuppgifter


Klimat och väder är i kursplanen för Fysik 1 det avsnitt som innehåller bl. a termofysik. I kursboken har
innehållet organiserats annorlunda. Utan stora delar av termofysikgrunderna är innehållet för klimat och
väder betydligt mindre och detta avsnitt kan du välja att reducera omfånget på till följande
instuderingsfrågor (1-3) samt problemlösning med ideala gas lagen (punkt 4 nedan). Skolverkets
övningsprov i Fy1 ger också ett tillämpat problem värt att studera. Du kan naturligtvis också välja att läsa
kapitlet i kursboken på samma sätt som du läst tidigare kapitel.

Instuderingsuppgifter:
1. Beskriv vad som menas med prognosmodell.

Svarsförslag hämtat från SMHI: ”En prognosmodell ska kunna förutse vädret på en given plats vid en
given tidpunkt. För att göra detta måste vädret på alla platser i alla nivåer i atmosfären beräknas.” och
”Att göra en väderprognos innebär att förutsäga exempelvis temperatur, vind, fuktighet, nederbörd,
molnighet och dimma på alla platser, vid rätt tidpunkt och på alla nivåer i jordens atmosfär. Utmaningen
ligger i att vår planet roterar och har olika egenskaper som påverkar strålningsbalansen, såsom
fördelningen av land och hav, olika jordarter och vegetation. Detta brukar benämnas
väderprognosproblemet.”

2. Beskriv vad som menas med klimatmodell.

Svarsförslag hämtat från SMHI: ”När man studerar klimatet i framtiden är man helt hänvisad till
beräkningar. För ett klimat som förändras räcker det inte med statistik över det som varit. Klimatmodeller
är istället det redskap som finns utvecklat för sådana studier. Dessa modeller är 3-dimensionella
matematiska beskrivningar av atmosfären, landytan, hav, sjöar och is. ” och till skillnad mot
prognosmodell gäller: ”En klimatmodell däremot ska beskriva klimatet, som enkelt uttryckt är vädret i
medeltal under en längre period och över ett större område. En klimatmodell kan inte förutse hur varmt
det kommer att vara i Norrköping den 20 januari 2098, men den kan säga något om hur vintrarna i södra
Sverige kommer att vara i slutet av seklet.”

Klimat och väder instud.uppg, forts


3. Beskriv växthuseffekten.

Svarsförslag hämtat från SMHI : ”Inkommande solstrålning passerar i stort sett genom växthusgaserna
och värmer upp jordytan. Den uppvärmda jordytan sänder i sin tur ut värmestrålning vars passage
effektivt hindras av växthusgaserna i atmosfären. En del av den hindrade värmestrålningen återstrålas
tillbaka mot jorden, vilket gör att temperaturen hos jordytan hålls både varmare och jämnare jämfört
med en planet utan en atmosfär. Det är detta vi kallar växthuseffekten.”

Notera att det är tack vare växthuseffekten som vi kan leva på jorden. Debatten kring växthuseffekt
handlar främst om ökning av växthuseffekten och de negativa konsekvenser som därav följer.

4. När det gäller beräkningar bygger kapitlet i hög grad på Termofysik.

Mer träning på ideala gaslagen är nyttigt. Läs läroboken s 222 - 224 som handlar om hur den ideala
gaslagen kan användas som en enkel modell för atmosfären.

Klimat och väder, instuderingsfrågor


Här följer tolv instuderingsfrågor på kap 8, Klimat och väder. Svar på frågorna finner du efter den sista
frågan på nästa sida.

1.) Hur stort är trycket i en cykelpump innan man börjar komprimera luften?
2.) Hur stort är övertrycket i cykelpumpen innan man börjar komprimera luften?

3.) Förklara kort vad en manometer är?

4.) Vad är namnet på den lag som säger att ”trycket och volymen är omvänt proportionella mot
varandra”?

5.) Vad kallas den lag som med om formel beskrivs så: pV/T = konstant

6.) Nämn minst fyra gaser som finns i atmosfären.

7.) Varför krävs det mycket kraftfulla datorer för att göra en bra väderprognos?

8.) Beskriv hur en databeräknad väderprognos görs. Dela gärna upp det i olika steg.

9.) Hur kan en meterolog veta om prognosen är tillförlitlig?

Forts nästa sida!

Klimat och väder, instud.frågor, forts


10.) Vilka kända fysikaliska lagar används för att förutsäga vädret. Om du har tillgång till Internet kan du
även leta där.

11.) Varför är lutningen på jordaxeln (ca 23°) viktig?

12.) Vad har vattnets värmkapacitet med väder och klimat att göra?

13.) Hur kan det vara sant att solstrålarna inte värmer upp atmosfären speciellt mycket, det sägs ju att vi
håller på att få en för varm atmosfär?

Svar till instuderingsfrågor 1 - 8

1.) 1013 hPa

2.) 0

3.) Det är en tryckmätare

4.) Boyles lag

5.) Tillståndslagen för ideala gaser

6.) Kväve, syre, koldioxid, metan, vattenånga, argon, ozon

7.) Man måste ha tillgång till många olika meteorologiska mätvärden från många olika platser. Dessutom
måste beräkningen göras i många små steg.

8.) Det geografiska området delas upp i ett rutnät. Meteorologiska data från varje ruta matas in i datorn
som sedan bearbetar data med hjälp av fysikaliska ekvationer och sedan ger datorn en prognos för hur
detta påverkar intilliggande rutors väderdata under nästa (t.ex.) minut. Nu har datorn nya väderdata för
varje ruta i systemet. Sen görs proceduren om. Meteorologiska data från varje ruta matas in i datorn som
sedan bearbetar data med hjälp av fysikaliska ekvationer och sedan ger datorn en prognos för hur detta
påverkar intilliggande rutors väderdata under nästa minut. På detta sätt kan man fortsätta med att göra
en väderprognos för de kommande 10 dagarna.

Svar till instuderingsfrågor 9 - 13

9.) Man gör flera olika datorsimuleringar med olika begynnelsevärden. Om skillnaderna i inmatade
begynnelsevärden är små och prognoserna ändå blir väldigt olika, då är inte prognosen speciellt pålitlig.
Och tvärtom förstås. Om skillnaderna i inmatade begynnelsevärden är stora och prognoserna ändå blir
ganska lika, då är prognosen tillförlitlig.

10.) Termodynamikens andra huvudsats, Newtons första lag.

11.) Jordaxelns lutning i förhållande till planet vinkelrät mot jordens omloppsbana är förklaringen till
varför årstiderna uppkommer.

12.) Vatten har extremt hög värmekapacitet och kan därför lagra stora mängder energi. Det har
betydelse för hur kraftiga orkaner byggs upp. Orkanen får energi från varmt vatten. Det har betydelse för
hur mycket värme som vindar över havet upptar och därmed hur luften stiger och molnen bildas. För
klimatet har hav och vatten stor betydelse som värmereservoar när årstiderna växlar.

13.) Solstrålningen kan förenklat sägas innehålla långvågig strålning och kortvågig strålning. Solens
infallande långvågiga strålning absorberas till stor del i atmosfären men inte den kortvågiga strålningen.
Men när den kortvågiga strålningen träffar marken och värmer den så orsakar den mycket långvågig
strålning. När denna strålning från marken träffar atmosfärens gaser så absorberas denna värmestrålning
så att atmosfären blir varmare. Det är alltså markstrålningen i kombination med mycket gaser i
atmosfären som är orsaken till atmosfären blir varmare och varmare.

Sammanfattning
I den här modulen har du jobbat med:

*) Ellära

*) Termofysik

*) Klimat och väder

Det du nu skall göra är att repetera hela kapitlet (lös gärna fler övningar i din lärobok). Därefter gör du
TDS 3:2. Eventuellt kan det sedan behövas ytterligare lite repetition. När du tycker att du behärskar
samtliga moment i denna modul gör du Uppdrag 3. Detta skickar du in till din webblärare via datorn, och
denne rättar och kommenterar det. Glöm inte att bifoga de två laborationsrapporterna. Du hittar
laborationerna på nästa sida.

Därefter väntar den sista modulen, modul 4, som kommer att behandla begreppen atomfysik, kärnfysik
och relativitetsteori.
Uppdrag, bedömning och kunskapskrav
Uppdragen är en viktig del i kunskapsinhämtningen. Det är därför viktigt att du är noggrann när du
arbetar med uppdragen, och att du är säker på att du förstår och behärskar det du skickar in. Vid den
Avslutande examinationen kommer du att få visa just detta.

Hur mycket arbete du ska lägga ner på svaren styrs bl a av din betygs- ambition, dina förkunskaper, hur
snabbt du läser kursen etc.

Jobba för att hålla studieplanen och de inlämningsdatum som finns i Novo. Risken är annars att du får för
mycket att göra i slutet av kursen. Kom också ihåg att betyget sätts efter den Avslutande examinationen.
Det är meningen att du skall utvecklas och lära dig under kursens gång.

Uppgifterna i uppdragen är uppdelade i olika betygsnivåer (E, C och A). Efter varje fråga anges vilken nivå
den ligger på.

Läs om bedömning och betyg i det svarta fältet. Vi som är lärare i kursen vill att du skall nå ditt mål. Vi
skall också säkerställa att du når Skolverkets uppsatta kunskaps-krav. Vill du repetera kunskapskraven för
kursen? Tryck då på ikonen så kommer du direkt till Skolverkets hemsida.
https://www.skolverket.se/undervisning/vuxenutbildningen/komvux-gymnasial/laroplan-for-vux-och-
amnesplaner-for-komvux-gymnasial/amne?url=1530314731%2Fsyllabuscw%2Fjsp%2Fsubject.htm
%3FsubjectCode%3DFYS%26courseCode%3DFYSFYS01a%26lang%3Dsv%26tos%3Dgy%26webtos
%3Dvuxgy%26p%3Dp HYPERLINK "https://www.skolverket.se/undervisning/vuxenutbildningen/komvux-
gymnasial/laroplan-for-vux-och-amnesplaner-for-komvux-gymnasial/amne?
url=1530314731%2Fsyllabuscw%2Fjsp%2Fsubject.htm%3FsubjectCode%3DFYS%26courseCode
%3DFYSFYS01a%26lang%3Dsv%26tos%3Dgy%26webtos%3Dvuxgy%26p
%3Dp&sv.url=12.b173ee8160557dd0b8100d#anchor_FYSFYS01a"& HYPERLINK
"https://www.skolverket.se/undervisning/vuxenutbildningen/komvux-gymnasial/laroplan-for-vux-och-
amnesplaner-for-komvux-gymnasial/amne?url=1530314731%2Fsyllabuscw%2Fjsp%2Fsubject.htm
%3FsubjectCode%3DFYS%26courseCode%3DFYSFYS01a%26lang%3Dsv%26tos%3Dgy%26webtos
%3Dvuxgy%26p
%3Dp&sv.url=12.b173ee8160557dd0b8100d#anchor_FYSFYS01a"sv.url=12.b173ee8160557dd0b8100d#
anchor_FYSFYS01a

Dina resultat under kursens gång samlas i en matris. Matrisen visar i vilken utsträckning du uppnått de
olika kunskapskraven i kursen. Matrisen, tillsammans med lärarens kommentarer, vägleder dig i vad du
behöver träna mer på för att nå ditt betygsmål. Efter den avslutande examinationen kommer matrisen
justeras en sista gång och därmed utgöra en del av betygsmotiveringen. Kom ihåg att nivån du visat
under kursens gång också behöver visas under den avslutande examinationen.

Studieguide 4

En översikt
I din sista modul kommer du att studera den moderna fysikens utveckling. Du kommer att få lära dig
mycket om materiens minsta byggstenar. Ett stort avsnitt behandlar kärnfysik, alltså vad atomkärnan
består av och den beter sig. Till sist tar vi en titt på relativitetsteori och standardmodell - alltså företeelser
som ligger utanför den klassiska fysiken!

Klicka nu på äpplet för att få en konkret målbeskrivning!

Målbeskrivning
Här får du en målbeskrivning, så att du kan få en uppfattning om vad ditt studium av den fjärde och sista
modulen ska leda fram till.

När du har gått igenom de tre sista kapitlen, Den moderna fysikens utveckling, Kärnfysik och
Relativitetsteori och standardmodell, ska du behärska:

• atomens historia

• kvanthypotesen

• Bohrs atommodell

• röntgenstrålning

• radioaktivitet

• atomkärnan

• kärnreaktioner

• halveringstid

• fission och fusion

• biologiska verkningar

• relativitetsteorin

• standardmodellen

Studietips
Arbeta med studieguiderna här i Novo parallellt med att du läser och löser uppgifter i din lärobok.

Var aktiv, titta på webblektioner och gör interaktiva övningar här i Novo, gör Testa-dig-själv och andra
övningar.

Arbeta aktivt med ditt betygsmål. Uppgifterna i upp-dragen här i Novo är nivå-uppdelade (E-, C- och A-
nivå). Koncentrera dig på den nivå som är ditt mål!

Utnyttja fysikhjalp@hermods.se
Att läsa i din lärobok
Relativitet, s 281-292

Materia och naturens krafter, s 293-320

Strålning från atomer och rymden, s 321-348

Kärnenergi, s 349-358

Strålning på gott och ont, s 359-376

Inledning
Denna studieguide omfattar kapitel 13 - 16 i läroboken.

I denna modul kommer du att få stifta bekantskap med följande begrepp:

Kvanthypotesen

Bohrs atommodell

Röntgenstrålning

Radioaktivitet

Atomkärnan

Kärnreaktioner

Halveringstid

Fission och fusion

Relativitetsteorin

Standardmodellen

När modulen är klar skall du sända in ditt sista uppdrag. Laboration nr 5 skall bifogas Uppdrag 4.
Instruktioner till denna kommer längre fram i denna modul.

Materiens inre
Vi börjar med att orientera oss lite om hur kunskapens om atomen och materiens uppbyggnad vuxit
fram. Detta är beskrivet i följande text fram till avsnittet om kärnfysik. I läroboken är det främst sidorna
293 - 317 som belyser detta. Genom att läsa både boken och webbtexten får du en nyanserad
sammanvägd bild.

Den moderna atomteorins utveckling går tillbaka till 1600-talet. Det dröjde dock ända till början av 1900-
talet innan en hållbar atommodell hade vuxit fram. Den historiska utvecklingen kan sammanfattas i
följande punkter:
1600-talet:

Robert Boyle formulerar tankar om att det finns grundämnen som är mer elementära än andra

ämnen.

ca 1800:

John Dalton antar att varje grundämne är uppbyggt av atomer som är karakteristiska för varje

grundämne.

1869:

Dmitrij Mendelejev ställer upp det periodiska systemet och tilldelar varje atom ett

atomnummer, Z.

1895:

Wilhelm Röntgen upptäcker röntgenstrålningen.

1896:

Henri Bequerel och Marie Curie upptäcker radioaktiv strålning.

Atomen
1897:

Joseph John Thomson upptäcker elektronen. Han presenterar också en atommodell där han tänker sig
elektronerna som små negativa partiklar inbäddade i positiv materia, som russin i en bulle.

1911:

Ernest Rutherford gör tillsammans med Hans Geiger och Ernest Marsden ett avgörande experiment som
leder till att atomkärnan upptäcks. Atomens massa är koncentrerad till en mycket liten volym i atomens
centrum.
1913:

Niels Bohr presenterar en helt ny teori för atomens uppbyggnad.

Klicka på bilden här bredvid för att använda det interaktiva periodiska systemet när du behöver det!

http://www.ptable.com/?lang=sv

Rutherfords atommodell
Rutherfords arbete under 1900-talets första år ledde till att en helt ny atommodell presenterades 1911.
Hans klassiska spridningsförsök, där alfapartiklar (He-kärnor) med hög energi skickades mot en tunn
guldfolie, gav resultat som visade att Thomsons "russinbullemodell" inte var realistisk.

Enligt Rutherfords modell består en atom av en liten kärna och elektroner som kretsar runt den. Kärnan
upptar bara en liten del av atomens volym, men större delen av dess massa. Kärnans radie är ca 10-15 m
och atomens radie ca 10-10 m.

Atomkärnan är positivt laddad. En atom med atomnummer Z har laddningen +Ze i kärnan och -Ze hos de
Z elektroner som kretsar runt kärnan (e är elementarladdningen).

Klicka på vänstra bilden för att utföra experimentet själv eller på den högra för att se en animation!

https://phet.colorado.edu/en/simulation/rutherford-scattering

https://www.youtube.com/watch?v=5pZj0u_XMbc HYPERLINK "https://www.youtube.com/watch?


v=5pZj0u_XMbc&feature=youtu.be"& HYPERLINK "https://www.youtube.com/watch?
v=5pZj0u_XMbc&feature=youtu.be"feature=youtu.be

Materiens inre: atomen


● Atomen består mest av tomrum

● Atommassan och dess positiva laddning måste vara koncentrerad till en ytterst liten kärna omgiven av
elektroner

● Modellen kallas även planetmodellen och gäller i huvudsak än idag

● Rutherford mätte kärnans diameter till 10 −15 m och atomens diameter till 10−10 m

● Atomer kan vara sammansatta till molekyler

Atomerna är byggstenarna i all materia i universum, men de är inte de minsta partiklarna som finns.
Atomerna är uppbyggda av mindre partiklar, protoner, neutroner och elektroner.
En atom består av positivt laddande protoner och neutroner utan någon laddning alls. Protonerna och
neutronerna bygger upp atomens kärna. Kring kärnan, på ganska stort avstånd i atomens mått mätt,
finns de negativt laddade elektronerna. Elektronerna finns på olika skal, banor eller energinivåer.
Elektronerna är just elektroner, vi har inte lyckats slå sönder dem för att veta om de består av något
annat. Protonerna och neutronerna däremot består av kvarkar. Det finns sex olika typer av kvarkar och
de förekommer alltid i par. Vi har aldrig detekterat en ensam kvark. Protonen består av två uppkvarkar
och en nerkvark, neutronen av två nerkvarkar och en uppkvark. Vad kvarkarna består av är det ingen
som vet i nuläget.

Atomen, forts
Den enklaste atomen är väteatomen. Den består av en proton och en elektron. Denna elektron finns
utanför kärnan och har en viss energi. Att rita energi rent allmänt är omöjligt, men "skalmodellen" är
mycket användbar. Man tänker sig då att elektronen kretsar i en bana runt kärnan. Om man ger
elektronen ett energitillskott på något sätt, t.ex. i ett rör med högspänning, kan elektronen hoppa ut och
lägga sig i en bana. Elektronerna kan bara finnas i banor på vissa bestämda avstånd från atomkärnan, så
kallade energinivåer. Varje atomslag har sina egna energinivåer. Elektronen kan finnas på olika
energinivåer men aldrig mellan dem. Du kan stå på golvet eller på stolen eller på bordet. Du kan inte stå i
luften mellan bord och stol. Detta är grunden i kvantteorin.

Anta nu att det kommer en alfapartikel, en annan elektron eller gamma strålning (ljus), och krockar med
en elektron som finns på ett nedre skal. Om energin passar så ger strålningen sin energi till elektronen
som plötsligt inte får befinna sig på det skalet för energin passar inte för skalet. Då måste elektronen
hoppa upp ett eller flera skal. Men tänk om det skalet är redan fyllt!

Klicka på vänstra bilden för att bygga en atom och på den högra för att se hur stor en atom är!

http://phet.colorado.edu/en/simulation/build-an-atom

https://www.youtube.com/watch?v=yQP4UJhNn0I HYPERLINK "https://www.youtube.com/watch?


v=yQP4UJhNn0I&feature=youtu.be"& HYPERLINK "https://www.youtube.com/watch?
v=yQP4UJhNn0I&feature=youtu.be"feature=youtu.be

Mer om atomen
Elektronen måste hoppa ändå. Även om skalet inte är fyllt så finns det ett ”håll” nu, där elektronen var
innan. Atomen ”mår inte bra” såhär och skickar tillbaka sin elektron omedelbart. Men nu finns plötsligt
ett överskott av energi som elektronen måste bli av med. Den energin skickar atomen ut som ljus. Ljuset
kan vara synligt eller osynligt. Det är så ljuset skapas.

Robert Brown var den förste som beskrev rörelserna av atomerna. Han undersökte pollenkorn i vatten
med mikroskop och såg att pollenkornen rörde på sig. Det var Albert Einstein som förklarade att rörelsen
berodde på att osynliga partiklar, dvs. atomerna eller molekylerna knuffade pollenkornen. Det var
bevisat att atomen verkligen fanns, inte bara i teorin. John Dalton var först med att ge atomen en
vetenskaplig betydelse. Niels Bohr skapade 1913 den atommodell som kallas för Bohr modellen och
används fortfarande. Niels Bohr fick Nobelpriset i fysik 1922.
Kvanthypotesen
Under 1800-talet undersöktes spektra från olika typer av ljuskällor. Man upptäckte då följande fenomen,
som man ännu inte kunde förklara:

*) Ett ämne i gasform ger upphov till ett linjespektrum som är karakteristiskt för det ämnet. Varje
grundämne har sitt specifika linjespektrum som fungerar som ett "fingeravtryck" för ämnet. Denna typ
av spektrum kallas emissionsspektrum.

*) Spektrum från solen är kontinuerligt, men innehåller karakteristiska svarta linjer. Dessa linjer kallas
Fraunhofer linjer efter den fysiker som bestämde våglängderna för dem. Denna typ av spektrum kallas
absorptionsspektrum.

*) Samma typ av absorptionsspektrum erhålls när vitt ljus får passera genom en gas av ett bestämt
ämne. De svarta linjerna överensstämmer med emissionslinjerna i ämnets emissionsspektrum.

Emissionsspektrum från vätgas innehåller 3 serier av linjer:

1.) Lymanserien som sänder ut ljus i det ultravioletta området.

2.) Balmerserien som sänder ut ljus i det synliga området.

3.) Paschenserien som sänder ut ljus i det infraröda området.

Den svenske fysikern Johannes (Janne) Rydberg formulerade ett samband, som med stor noggrannhet
bestämmer våglängderna i vätespektrum. Detta samband brukar kallas Rydbergs formel. Linjerna i
Balmerserien beräknas på följande sätt:

R kallas Rydbergkonstanten för väte

Bohrs atommodell
Rutherfords atommodell kunde inte förklaras med hjälp av den klassiska fysiken. Enligt klassisk teori
skulle elektronerna "falla" in mot atomkärnan medan de sände ut kontinuerlig "bromsstrålning". Ett
annat problem var att förklara mekanismerna bakom emissions- och absorptionsspektra. Niels Bohr
lyckades kombinera känd kunskap med nya antaganden för att skapa en helt ny atommodell. Han utgick
från Rutherfords atommodell, kvanthypotesen och Einsteins teori om fotoner. Detta kombinerade han
med egna antaganden och kunde så småningom presentera en modell för väteatomen. Du kommer att
läsa mer om Bohrs atommodell i kursen fysik 2, men i texten som följer ger vi en liten kort
sammanfattning för att ge en helhetsbild.

Bohr gjorde ett antal antaganden, eller postulat, som grund för en ny atommodell. De kan sammanfattas
i följande två punkter:

1.) Elektronerna rör sig i bestämda cirkulära banor runt atomkärnan utan att sända ut strålning. I varje
sådan bana (tillstånd) har elektronen en bestämd energi.

2.) När en elektron övergår från en bana med energin E N till en annan bana med energin En, sänder
atomen ut strålning i form av en foton med energin hf = E N - En, där f är strålningens frekvens och h är
Plancks konstant.

I Bohrs modell för väteatomen kan energinivåerna beräknas med hjälp av följande samband:

Bohrs modell överensstämmer med Rydbergs formel för väteatomen. Observera att energinivåerna är
negativa. Det betyder att atomen joniseras när E = 0 J. Den lägsta nivån (n = 1) kallas atomens
grundtillstånd. De högre nivåerna (n > 1) kallas exciterade tillstånd.

Bohrs postulat
Bohrs atommodell förklarar varför vi får linjespektra från atomer. Den beskriver också väteatomens
energinivåer korrekt. Atomer med flera elektroner har en mer komplex uppbyggnad. Där är Bohrs modell
otillräcklig. "Vätelika" atomer, t.ex. Li, Na och K, kan delvis beskrivas på ett liknande sätt. Det ska vi dock
inte gå in på i den här kursen.

Niels Bohr (1885-1962)

Dansk fysiker som fick Nobelpriset 1922. Bohrs två postulat för en stabil atom kom 1913.

1.) Elektronerna rör sig endast i vissa tillåtna cirkulära banor runt atomkärnan utan att sända ut
strålning.

2.) Atomen strålar endast när en elektron övergår från en bana till en annan och då med en frekvens som
motsvarar energiskillnaden mellan banorna.

Excitation av atomer
Bohrs atommodell förklarar spektrallinjer med att den exciterade atomens elektroner faller tillbaka till
lägre energinivåer, elektronbanor. Energiöverskottet sänds ut som fotoner (linjer).

Atomer exiteras främst på två sätt:

1.) Via rörelseenergi från en kolliderande elektron. För att övergången ska ske måste elektronens
rörelseenergi vara minst energiskillnaden till nästa energinivå. Med andra ord, via
partikelbombardemang. Detta innebär att man låter en partikel krocka med atomen så att den på så sätt
får energi. Här måste inte den krockande partikeln ha samma energi som skillnaden mellan
energinivåerna då partikeln kan åka iväg med överskottsenergin.

2.) Via absorption av en foton vars energi är exakt energiskillnaden till en högre nivå. Detta kan man
exempelsvis göra genom att tillföra värme och på så sätt inducera en excitation av atomen.

Enligt Bohrs modell ligger alltså elektronerna i olika energinivåer(elektronbanor). När atomen tillförs
energi (exciteras) så kommer en atom att kunna nå en högre energinivå. Den kommer sedan förr eller
senare att falla tillbaka spontant(de exciteras) till sin ursprungliga energinivå. Då strålar atomen ut en
foton motsvarande den energi som skiljer mellan de två energinivåerna.

Energienheten joule är ofta opraktisk att använda i dessa sammanhang. I stället används enheten 1
elektronvolt, Sambandet mellan enheterna ges av:

1 eV = 1,602 · 10-19 J

Enheten elektronvolt
Energienheten elektronvolt (lätt att tro att det handlar om spänning!):

Elektronvolt definieras som den energi elektronen får om den accelereras av en spänning på 1 volt.

Klicka på filmerna nedan för att se två genomgångar!

https://www.youtube.com/watch?v=_GsOUFIWDO4 HYPERLINK "https://www.youtube.com/watch?


v=_GsOUFIWDO4&feature=youtu.be"& HYPERLINK "https://www.youtube.com/watch?
v=_GsOUFIWDO4&feature=youtu.be"feature=youtu.be

https://www.youtube.com/watch?v=Sk8fgh9VX9k HYPERLINK "https://www.youtube.com/watch?


v=Sk8fgh9VX9k&feature=youtu.be"& HYPERLINK "https://www.youtube.com/watch?
v=Sk8fgh9VX9k&feature=youtu.be"feature=youtu.be

Röntgenstrålning
Även röntgenstrålning är ett begrepp som tas upp mer i följande fysikkurs, men vi avslutar med några
rader för att ge en grundläggande bild för din fortsatta förståelse av bakgrunden till röntgenstrålningen.

Den tillämpning av röntgenstrålning som de flesta har kommit i kontakt med är avbildning i medicinska
sammanhang. Då utnyttjar man röntgenstrålningens genomträngningsförmåga för att kunna "se" in i
människokroppen.

I det här avsnittet ska vi titta på hur röntgenstrålningens fysikaliska egenskaper och hur röntgenstrålning
kan produceras i ett röntgenrör.

Röntgenstrålningen upptäcktes 1895 av den tyske fysikern Wilhelm von Röntgen. Röntgenstrålning är
elektromagnetisk strålning med hög energi och produceras med hjälp av ett röntgenrör. Elektroner
accelereras med hjälp av en högspänning. När de träffar anoden bromsas de in och sänder ut en eller
flera fotoner. Om spänningen över röntgenröret är U blir röntgenfotonernas kinetiska energi högst lika
med:

Ek = e · U

Röntgenstrålning, forts
Röntgenstrålar har våglängder mellan omkring 5 pm och 10 nm (går alltså över på gammastrålningen).
Röntgenstrålar med våglängder under 0,1 nm kallas hårda, medan strålar med våglängder över 0,1 nm
kallas mjuka.

Röntgenstrålning som består av flera olika våglängder kallas vit, medan strålning med (i huvudsak) en
våglängd kallas monokromatisk. Röntgenstrålning kommer bland annat från elektroner som ökar sin
lägesenergi i atomer för att sedan återställa den. Då frigörs strålning. Är energidifferensen låg uppstår
synligt ljus, men om den är högre uppstår röntgenstrålning. Röntgenstrålning används idag bland annat
inom sjukvården för att undersöka patienter. Röntgenstrålar skickas då mot patienten och penetrerar
olika vävnadstyper olika bra. Skelett, exempelvis, kastar en skugga på den uppfångande filmen bakom
patienten, vilket resulterar i en bild av patientens skelett.

Så kallad radioterapi, där röntgenstrålar skickas mot tumörer används också som behandling mot cancer.
Även inom bland annat flygsäkerhet används röntgen för att skanna resväskor efter metallföremål.
Väskorna får då åka genom en ”tunnel” som skickar röntgenstrålar genom dem. Eftersom strålarna inte
går genom metall kommer metallföremål att ge en skugga på den resulterande bilden.

Röntgenspektrum
Röntgenstrålning skapas genom att beskjuta en metallyta med elektroner med hög fart, dvs. hör energi,
ca 50 eV. Röntgenstrålning är elektromagnetisk strålning (ljus) med kort våglängd, 0,001 − 50
nanometer. Röntgenstrålningen har en väldigt typiskt kurva när det gäller intensitet på olika våglängder.

Som ni ser har vi först ett kontinuerligt spektrum för att sedan få två skarpa toppar, eller s k spektrallinjer.
Topparna visar den karakteristiska strålningen som beror på och är unik för materialet i metallplattan,
dvs. anoden. Den första delen av kurvan visar bromsstrålningen.

Ett röntgenspektrum uppkommer genom att man låter en elektron från något av de inre skalen exciteras.
Då kommer en annan elektron och tar den lediga platsen. Detta gör att det skickas ut en röntgenfoton.

Vid inbromsningen av elektronerna skapas två typer av strålning. Bromsstrålning, när laddningar som
accelereras sänder ut kontinuerlig strålning, och karakteristisk

strålning, när elektronerna exciterar metallatomerna genom att slå ut elektroner från de innersta skalen.
När dessa skal besätts med fria elektroner, skapas fotoner med hög energi. Metallen som används består
av tunga atomer t.ex. wolfram eller molybden som har många elektroner och tål dessutom höga
temperaturer.

Lite historia om röntgen


År 1895 upptäcker Wilhelm Conrad Röntgen (1845-1923) röntgenstrålningen som var då okänd och
kallades x-ray.

Han fick det första Nobelpriset i fysik 1901. År 1912 upptäckte Max von Laue (1879-1960) att röntgen är
elektromagnetisk strålning genom diffraktion i kristaller. Han fick Nobelpriset 1914.

Exempel:

Elektroner i ett röntgenrör accelereras med spänningen 40 kV. Bestäm röntgenstrålningens våglängd.
Lösning:

Radioaktiviteten upptäcks

År 1896 upptäcker Henri Becquerel att uransalt svärtar en fotografisk plåt. Marie Curie inför begreppet
radioaktivitet och upptäcker att strålningen är joniserande. Makarna Curie och Becquerel tilldelas
Nobelpriset i fysik 1903. Ett radioaktivt ämne är ett ämne som avger strålning från atomkärnan av sig
själv. Atomkärnan är helt enkelt för stor och den starka kärnkraften kan inte riktigt hålla ihop den. Kärnan
gör sig av med överskottsenergin genom att falla sönder. Vi säger att ämnet är radioaktivt. Lite felaktigt
säger man ofta "radioaktiv strålning". Det är inte strålningen som är radioaktiv, utan själva ämnet.

Sönderfall är alltså när en atomkärna går sönder och avger strålning. Ernest Rutherford, från Nya
Zeeland, upptäcker tre typer av radioaktivitet som han kallar för alfa(α)-, beta(β)- och gamma(γ)strålning.

Sammanfattning, avsnitt 10
*) Gå igenom sammanfattningen på sid 317 i din lärobok.

*) Titta på exempel 1 sid. 299.

*) Lös övningsuppgifter 13.1, 13.3, 13.4 och 13.7. (Du får söka uppgifter om Jordens radie i en
formelsamling.)

*) De fyra krafterna sägs förmedlas av partiklar. Läs om dessa förmedlarpartiklar och en


förklaringsmodell hur de fungerar på sid 314 i läroboken.

Kärnfysik
I detta avsnitt kommer du att studera:

● Atomkärnan

● Krafter och energi inuti atomkärnan

● Enkla kärnreaktioner

● Mer om radioaktivitet

● Halveringstid

● Fission av tunga kärnor

● Fusion av lätta kärnor

● Biologiska verkningar
Radioaktivitet har du säkert redan hört talas om. I massmedia diskuteras regelbundet både kärnkraftens
avfallshantering och riskerna för kärnvapenspridning. Radioaktivitet beror på processer som sker inne i
atomkärnan. I kapitlet Radioaktivitet ska vi gå igenom grunderna inom kärnfysiken. Vi ska också studera
ett antal tillämpningar, t ex inom medicin och energiförsörjning.

Målbeskrivning:

När momentet är avslutat skall du:

● ha grundläggande kunskaper om atomkärnans uppbyggnad, sambandet mellan massa och energi och
förstå principerna för kärnreaktioner

● ha grundläggande kunskaper om radioaktivitet och olika typer av strålning

● känna till strålningens biologiska verkan och hur den mäts

● ha grundläggande kunskaper om fission och fusion

● kunna ge exempel på kärnfysikaliska tillämpningar i vår omvärld inom skiftande områden

Atomkärnan
Läs texten i läroboken på s 322-325. Atomkärnan upptäcktes redan 1911. Då visste man också att den var
positivt laddad. Det dröjde dock ända till 1932 innan neutronen upptäcktes, och man kunde fastställa att
atomkärnan är uppbyggd av protoner och neutroner. Då visste man redan att samma grundämne kan ha
olika massor. Efter neutronens upptäckt kunde man visa att dessa olika isotoper av ett och samma
grundämne har samma antal protoner, men olika antal neutroner i kärnan.

En isotop beskrivs med hjälp av masstalet (A), atomnumret (Z) och antalet neutroner (N). Följande
samband gäller: A = Z + N. Masstalet anger antalet nukleoner i kärnan.

En bestämd atomkärna kan anges på följande sätt:

X är ämnets kemiska beteckning.

Atommassan anges ofta i atommassenheten 1u. Om man tittar i en formelsamling finns storleken på 1u
angiven: 1u = 1,66 ·10-27 kg. Du har den också på s 322. På s 324 anges protonens, neutronens och
elektronens massor. Lös uppgift 14.2-14.4 på s 346 i läroboken.

Inne i atomkärnan

Atomkärnan består av protoner och neutroner. Protonen har positiv laddning och neutronen är neutral.

I avsnittet om statisk elektricitet såg vi hur saker med lika laddning stöter bort varandra. Hur kan då
kärnan hålla ihop? Jo, de positiva protonerna hålls ihop av en kraft som bara verkar på oerhört nära håll.
Det är den starka kärnkraften. För det mesta lyckas den starka kärnkraften hålla ihop kärnan.

Den enklaste kärnan av alla är vätets. Den har bara en proton. Nästa grundämne är helium som har två
protoner i kärnan, nästa är litium som har tre, osv. Den tyngsta kärnan som finns i naturen är urankärnan
med 92 protoner. I laboratorier har man fått fram kärnor med en bra bit över hundra protoner.
Grundämnena ordnas i periodiska systemet efter antalet protoner i kärnan. Antalet protoner är samma
tal som atomnumret.

Atomens delar
Protonen upptäcks

Rutherford upptäcker tillsammans med Hans Geiger och Ernest Marsden atomkärnan och dess positiva
laddning med sitt guldfolieexperiment år 1910. Han fann att kärnan kan uppdelas i den positiva
vätekärnan som han kallade proton.

Neutronen upptäcks

Man visste att kärnan innehåller mer massa än bara positiva protoner. Idén var att några protoner
neutraliseras av elektroner. W Bothe och H Becker besköt år 1930 beryllium med alfapartiklar.
Experimentet gav en genomträngande neutral strålning med högre energi än gammastrålning. James
Chadwick (1891-1974) antar år 1932 att strålningen består av neutrala partiklar, neutroner. Chadwick
fick Nobelpriset 1935.

Atomens delar och massor

Atomen består av följande tre elementarpartiklar med massorna me , mp och mn:

Kärnpartiklarna protoner och neutroner kallas även nukleoner.

Atomen (nukliden, grundämnet) X med A stycken nukleoner och Z stycken protoner betecknas:

Z anger atomnumret, A anger masstalet (protoner och neutroner tillsammans). Antalet neutroner N = A
- Z.

Titta nu på filmen!

https://www.youtube.com/watch?v=kBgIMRV895w HYPERLINK "https://www.youtube.com/watch?


v=kBgIMRV895w&feature=youtu.be"& HYPERLINK "https://www.youtube.com/watch?
v=kBgIMRV895w&feature=youtu.be"feature=youtu.be

Isotoper
Det finns flera isotoper av ett och samma grundämne. Alla har samma kemiska egenskaper som ämnet,
eftersom de yttersta elektronerna som bestämmer dessa egenskaper är samma. Antalet protoner i
kärnar är också samma. Det som skiljer isotoperna från varandra är att de har olika antal neutroner.
Atomkärnan påverkar inte de kemiska egenskaperna hos ett ämne. Det finns isotoptabeller som
innehåller stabila isotoper och fler än 2000 ostabila isotoper.

T ex har grundämnet kisel tre isotoper:

Grekiska: isos topos = samma plats, dvs. samma plats i periodiska systemet.

Det finns tre olika isotoper av väte med ingen, en eller två neutroner i kärnan.

Massdefekt och bindningsenergi


Gör övningsuppgifter 14.5 och 14.6 i läroboken. Använd en tabellsamling, eller en nuklidkarta
(http://atom.kaeri.re.kr/) för att slå upp nuklidmassan. Klicka på kartan och sök upp den nuklid du letar
efter eller skriv in t.ex. Si-28 i sökfältet. Massan av fria protoner och neutroner är större än om de är
bundna i en atomkärna. Varför?

Jo, energi måste tillföras för att bryta loss dem från kärnan. Enligt relativitetsteorins formel E = m · c2 kan
massa omvandlas till energi. Skillnaden i massa (Δm) kallas för massdefekt och motsvarar
bindningsenergin som krävs för att bryta isär nukleonerna. Omvänt, avges energi om fria neutroner och
protoner slås samman till en atomkärna.

Exempel:

Nuklidmassor
Universella massenheten u

Inom kemin och kärnfysiken anges ofta massa med den universella massenheten, u.

● Elektronen väger 0,0005486 u

● Protonen väger 1,0072765 u

● Neutronen väger 1,0086649 u

Nuklidmassa och atomvikt


Masstalet är antalet protoner och neutroner

Nuklidmassan är atomens (isotopens) massa inklusive elektroner

Atomvikten (atommassan) är den genomsnittliga massan av grundämnets alla isotoper, inklusive


elektroner.

Exempel:

Atomvikten: 28 · 0,9223 + 29 · 0,0467 + 30 · 0,031 = 28,1087 u

Massan per nukleon

När masstalet ökar minskar massan per nukleon - fram till järn. Låg massa per nukleon innebär hög
bindningsenergi. Atomer med masstalet knappt 60 (Järn och nickel) har lägst massa per nukleon och
högst bindningsenergi. Lätta atomer som väte avger energi om de slås samman (fusion) till tyngre
atomer, t.ex. helium. Tunga atomer som uran-235 avger energi om de klyvs (fission) till lättare atomer.

Enkla kärnreaktioner
En kärnreaktion kan ske om en kärna beskjuts med en partikel, t ex en proton eller en neutron. Den
ursprungliga kärnan omvandlas så att nya ämnen och partiklar bildas. En reaktion kan ske om följande
villkor är uppfyllda:

● Laddningen bevaras

Den sammanlagda laddningen hos de inblandade partiklarna är lika stor före och efter reaktionen. Vid
reaktionen här finns det sammanlagt 6 protoner både före och efter reaktionen.

● Nukleontalet bevaras

Summan av antalet neutroner och protoner, det vill säga summan av masstalen, är lika stor före och efter
reaktionen. Vid reaktionen här är summan av masstalen 4 + 14 = 18 före reaktionen och 17 + 1 = 18
efter reaktionen.

● Den totala energin bevaras


Enligt Albert Einsteins relativitetsteori är massa ekvivalent med energi enligt sambandet E0 = mc2, där c
är ljushastigheten i vakuum. Med hjälp av detta samband kan massenergin för de inblandade partiklarna
beräknas. Partiklarna kan också ha kinetisk energi, Ek, både före och efter reaktionen. Slutligen kan en
kärna befinna sig i ett exciterat tillstånd efter reaktionen, vilket leder till att gammastrålning med
bestämd energi, Eγ, sänds ut. Summan av de tre energislagen är lika stor före och efter reaktionen, det
vill säga E = E0 + Ek + Eγ är konstant.

Enkla kärnreaktioner, forts


Massan hos en atomkärna, som vi har sett tidigare, är alltid mindre än summan av nukleonernas massor.
Massan per nukleon har beräknats för olika atomkärnor i diagrammet på s 325 i läroboken. Enligt
diagrammet har 56Fe lägst massa per nukleon. (Egentligen är det Ni om vi räknar efter noga) Det betyder
att det kan frigöras energi både när tunga kärnor delar sig och när lätta kärnor slås ihop till tyngre.

Den första typen av reaktion kallas fission och tillämpas i kärnkraftverk. Den andra typen av reaktion
kallas fusion. Det forskas en del för att kunna utnyttja denna energiform i kraftverk, men det är långt kvar
innan alla tekniska problem är lösta.

Solens energi alstras med hjälp av fusionsenergi, där lätta väte och heliumkärnor slås ihop till tyngre
kärnor.

Studera Exempel 1 på s 329 i läroboken för att se hur den frigjorda energin vid en kärnreaktion kan
beräknas.

Alfastrålning
Läs texten i läroboken sid 326 - 337.

Alfa-, beta- och gammastrålningens egenskaper sammanfattas nedan:

α-strålning:

Vid α-strålning sänds en alfapartikel dvs. två protoner och två neutroner som sitter ihop. Man säger att
det sänds en He-kärna ut. Det betyder att masstalet hos dotterkärnan minskar med 4 och atomnumret
minskar med 2.

Exempel:

Det är oftast tunga atomkärnor som spontant sönderfaller med α-sönderfall. Den utsända α-partikeln får
en bestämd kinetisk energi. Dotterkärnan är ofta exciterad efter sönderfallet och sänder ut γ-strålning.

Rutherford visade år 1903 att α-strålar är positiva och år 1908 att de är heliumkärnor (He 2+) med hög
hastighet och energi. Energin är några megaelektronvolt. α−strålning har kort räckvidd och stoppas av
papper och hud. Den är dock skadlig (joniserande) vid inandning och förtäring. α-strålning består av α-
partiklar som är två protoner och två neutroner som sitter jättehårt ihop. Denna typ av strålning kan inte
röra sig så långt, inte mer än ett par cm i luft och den kan knappt gå genom papper. Den atomkärna som
skickar ut α-strålning tappar två protoner och två neutroner och atomnumret minskar med 2 samt
masstalet minskar med 4.

Alfastrålning, forts
Exempelvis sönderfaller uran-238 till torium-234 vid alfasönderfall:

Energin fås av massdefekten och avges i form av rörelseenergi hos främst alfapartikeln. Atomkärnor i
naturen som sänder ut alfastrålning har alla masstal över 209. Den starka kärnkraften räcker inte för att
hålla samman dem. Radium har atomnummer 88. Kärnan i isotopen radium-226 har 88 protoner och 138
neutroner. Radium ger alfastrålning och det betyder att det är 2 protoner och 2 neutroner, alltså en
helium kärna, som skickas ut. Efter sönderfallet finns det kvar 86 protoner och 136 neutroner. För att på
snabbaste och enklaste sätt at reda på vilket nytt ämne som bildats skriver vi ut atomnummer och
masstal med siffror och kontrollerar att talen stämmer. Det är ämnet som har atomnummer 86. Det är
radon.

Klicka på bilden för att utforska alfasönderfall!

http://phet.colorado.edu/en/simulation/alpha-decay

Betastrålning
β--sönderfall:

Vid β--sönderfall omvandlas en neutron till en proton, en elektron och en antineutrino. Denna
grundläggande reaktion kan skrivas:

Vid β--sönderfall ökar atomnumret med 1 medan masstalet är oförändrat.

Exempel:

β+-sönderfall
Vid β+-sönderfall omvandlas en proton till en neutron, en positron och en neutrino. Denna
grundläggande reaktion kan skrivas:

Vid β+-sönderfall minskar atomnumret med 1 medan masstalet är oförändrat.

Betastrålning, forts
Klicka på bilden för att utforska betasönderfall!

http://phet.colorado.edu/en/simulation/beta-decay

β-strålning består av snabba elektroner eller positroner (positiva elektroner). β-strålning består av β-
partiklar som har bildats när en neutron i kärnan har omvandlats och blivit en elektron och en proton.
Det är den svaga kärnkraften som styr denna omvandling. Denna typ av strålning kan röra sig lite längre,
ett par dm i luft och den kan gå genom 3 mm tjock aluminiumfolie. Vi säger att strålingen har kort
räckvidd, några mm, tränger igenom papper och hud men skadar inte djupare. Den är dock skadlig vid
inandning och förtäring. β-strålning stoppas av glas och metall. Den atomkärna som skickar ut β-strålning
tappar alltså en neutron i kärnan men får istället en extra proton. Detta medför att atomnumret ökar
med 1 men masstalet förändras inte (den väger alltså ungefär lika mycket som tidigare).

Den fria neutronens halveringstid är 10,6 minuter. Neutroner i atomkärnor med överskott på neutroner
kan sönderfalla till protoner, t.ex i

som finns naturligt i människokroppen.

Energin fås av massdefekten och avges i form av rörelseenergi hos främst betapartikeln. En positron och
en elektron förintar varandra och sänder ut två fotoner, dvs bevarar rörelsemängden.

Gammastrålning
Vid γ-strålning frigörs fotoner med hög energi. γ-strålning frigörs ofta vid både α- och β-sönderfall,
eftersom dotterkärnan befinner sig i ett exciterat tillstånd efter sönderfallet. Kärnan kan övergå till
grundtillståndet genom att sända ut γ-strålning med bestämd energi. Vid γ-sönderfall ändras inte
kärnans sammansättning. Masstal och atomnummer är oförändrat.

Vid radioaktiva sönderfall sker ofta en hel serie sönderfall innan en stabil kärna erhålls. Rutherford visade
år 1914 att gammastrålning är elektromagnetisk, och består av fotoner med mycket hög energi (100 keV
- 10 MeV). Strålningen har lång räckvidd, är starkare än röntgen och är joniserande. Den tränger igenom
vävnad och delvis även metall. Den stoppas dock av tungmetaller som t.ex bly (Pb).

Vid både alfa- och betasönderfall bildas gammastrålning samtidigt. Vid alfa-och betasönderfall hamnar
kärnan ibland i ett exciterat tillstånd. Atomkärnan sänder ut elektromagnetisk strålning, gammafotoner,
vid återgången till grundtillståndet. Gammafotonerna har hör energi, några MeV.
Gammastrålningen har mycket god genomträngningsförmåga. Det behövs flera cm bly för att sakta ner
den. Det kan aldrig riktig stoppas. Gammastrålningens stora genomträngningsförmåga och skadeverkan
på celler utnyttjas för att döda cancerceller i djupt liggande tumörer.

Alfastrålningen stoppas lätt men orsakar störst skada om den når fram. Betastrålningen är lite svårare att
stoppa men ställer till mindre skada. Gammastrålningen är väldigt svår att stoppa men skadar minst, om
energin inte är för stor.

Max Planck
Max Planck formulerade kvanthypotesen som innebär att en atom bara kan sända ut eller taga emot
energi i bestämda energikvanta. Dessa kvanta har en energi som är proportionell mot frekvensen på den
strålning (det ljus) som atomen sänder ut eller tar emot:

E = h · f, där f är strålningens frekvens och h är en proportionalitetskonstant.

Proportionalitetskonstanten h har fått namnet Plancks konstant. Den har värdet 6,63 · 10 -34 Js.

Max Planck (1858-1947)

Planck var en tysk fysiker och professor vid Universiteten i Kiel, Berlin och Wien. Han utforskade och
förklarade värmestrålningen. Han antog att strålningen utgörs av kvantiserad energi det vi kallar fotoner.
Utgångspunkten i hans teori är att processerna inte är kontinuerliga, utan sker i bestämda steg med en
viss sannolikhet. Planck anses som grundare av kvantfysiken och fick Nobelpriset i fysik 1918 för
kvantteorin. Utan kvantfysik hade saker som mikroelektronik inte funnits idag.

Albert Einstein tänkte sig dessa energikvanta som "ljuspartiklar", som han kallade fotoner. Han tänkte sig
all elektromagnetisk strålning som en ström av partiklar, fotoner, där varje foton har energin:

Einstein förklarar fotoeffekten


Albert Einstein använder Plancks kvant- och fotonhypotes för att förklara fotoeffekten. Han antar:

1.) att elektroner upptar fotonenergi i odelbara kvanta

2.) att fotonen endast kan avge hela sin energi till en elektron

3.) att en del av fotonenergin går åt till att frigöra elektronen från ytan, det s k utträdesarbetet

4.) att resten av energin ger den till den frigjorda fotoelektronen som får en rörelseenergi

Dessa hypoteser sammanfattade han i Einsteins fotoelektriska ekvation. Mer om det i fysik 2. Han
presenterade sin fotoelektriska ekvation år 1905 och fick Nobelpriset för detta år 1921. Vi återkommer
till den fotoelektriska effekten längre fram i kapitlet. Vi kommer också att utnyttja kvant hypotesen för att
förklara uppkomsten av emissions- och absorptionsspektra.
Den danska forskaren Niels Bohrs teorier och den tyske Albert Einsteins forskade om kvantmekaniken på
olika sätt. Bohrs kvantmekanik går ut på att inget i naturen går att förutse. Einstein menar istället att allt
är rationellt och det alltid fanns en anledning till att naturen beter sig på ett visst sätt ända in på
atomnivå. Bohr menade till slut att det fanns två kvantmekaniska system den förutsägbara samt den
oförutsägbara. De bråkade om kvantteorin i hela sina liv.

Fotonens och energins kvantisering markerar en brytning runt år 1900 mellan klassisk fysik och modern
fysik. År 1926 presenterade Erwin Schrödinger (1887-1961) kvantmekaniken där en vågfunktion
beskriver små partiklars uppförande. Vågfunktionen (Ψ) anger bl a sannolikheten att finna partikeln i ett
visst område.

Neutrino och energi


Vid ett visst alfasönderfall får alla alfapartiklar samma energi. Betapartiklarnas energi varierar dock
mellan 0 och ett maxvärde, motsvarande massdefekten. Förklaringen är att vid betastrålning emitteras
en neutral neutrino (ν). Neutrinon introducerades 1930 av Wolfgang Pauli och observerades först 1956.
Neutrinon väger max 10eV. Jämför med elektronen som väger 511keV. Ett exempel på betasönderfall:

Lös uppgifterna 14.7, 14.9 och 14.10 i läroboken sid 346.

Halveringstid
Radioaktiviteten kan förklaras med hjälp av modeller som påminner om atomfysikens modeller. Enligt
Bohrs atommodell befinner sig elektronerna i olika tillstånd. Elektronen sänder ut strålning när den går
från ett tillstånd med högre energi till ett tillstånd med lägre energi. Atomkärnan kan beskrivas på ett
liknande sätt. En kärna som befinner sig i ett tillstånd med högre energi kan övergå till ett tillstånd med
lägre genom att sända ut radioaktiv strålning med bestämd energi. Läs läroboken sid 338 - 344.

För att beskriva hur snabbt en viss typ av atomkärnor sönderfaller används begreppet halveringstid.
Halveringstiden är den tid det tar för hälften av kärnorna i ett ämne att sönderfalla.

Sönderfallslagen kan användas för att beräkna hur många kärnor av en viss typ som finns kvar efter tiden
t. Den formuleras i läroboken på följande sätt, där N0 är antalet kärnor från början, N är antalet kärnor
vid tiden t och T är halveringstiden:

Ett vanligare sätt att formulera sönderfallslagen är följande:

där λ är sönderfallskonstanten. Sönderfallskonstanten är ett mått på sannolikheten för att en kärna ska
sönderfalla.
Halveringstid, forts
Sönderfallskonstanten är som sagt ett mått på sannolikheten för att en kärna ska sönderfalla. Vi kan få
ett samband mellan halveringstiden och sönderfallskonstanten med hjälp av följande resonemang:

Insatt i sönderfallslagen får vi:

Om vi använder logaritmen på båda sidor får vi:

Aktivitet
Aktiviteten för ett radioaktivt ämne definieras som antal sönderfall per tidsenhet. SI-enheten är
bequerel: 1 Bq = 1 s-1

Aktiviteten ändras med tiden på följande sätt:

där A0 är aktiviteten från början, A är aktiviteten vid tiden t och T är halveringstiden.

Aktiviteten minskar med tiden på samma sätt som antalet kärnor gör. Aktiviteten är ett mått på
sönderfallshastigheten för det aktuella ämnet.

Om antalet kärnor är N och sönderfallskonstanten är λ, kan aktiviteten beräknas som:

Detta uttryck kan också skrivas:

där A0 är aktiviteten från början. Lös nu några uppgifter i boken, t.ex. 14.15, 14.16, 14.17 på s 347.

OBS! Notera att läroboken använder R (radiation) som storhet för A (aktivitet).

Aktivitet, forts
Antalet kärnor är stor i början, och sannolikheten för dem att falla sönder också stor. I takt med att N
avtar, avtar aktiviteten.

Kurvan visar hur aktiviteten avtar med tiden. (Aktivitet betecknas också med R i viss litteratur.)

Genom experiment visar man att aktiviten avtar exponentiellt enligt figuren. Den tiden det tar för
aktiviteten att sjunka till hälften kallas för halveringstid (T 1/2).

Exempel på sönderfall
Halveringstid är den tid som det har tagit för hälften av ett föremål att avge sin strålning eller hälften av
ämnets kärnor att sönderfalla. Men eftersom atomkärnorna avger sin strålning helt slumpmässigt, kan
man inte exakt säga när det ska ske, men man kan räkna ut sannolikheten, för att det finns så många
atomer. Alla ämnen har en egen halveringstid som kan bestämmas genom experiment. Statistiskt sett
kan vi säga, att av hundra miljoner kärnor av kol-14 finns femtio miljoner kvar efter 5 600 år. Resten har
omvandlats till kväve. Man säger då att kol-14 har en halveringstid på 5 600 år. En radioaktiv isotops
halveringstid är den tid som gått när hälften av kärnorna sönderfallit. Halveringstiden är alltså ett mått på
ett radioaktivt ämnes livslängd.

Exempel på hur det fungerar:

Anta att du börjar med 100 kärnor. Efter första halveringstiden så har du 50 kärnor kvar, efter nästa
halveringstid så har du 25 kärnor kvar och efter ytterligare en halveringstid så har du 12,5 kärnor kvar.
Alltså alltid hälften av det som är kvar. Det är därför vi aldrig kan förutse när den sista kärnan faller
sönder.

Olika ämnen har olika halveringstid; några exempel är:

Krypton-94: 1,4 sekunder

Jod-131: 8 dagar

Cesium-137: 30 år

Kol-14: 5730 år

Uran-238: 4,5 miljarder år

Kol-14-metoden
Med hjälp av kol-14-metoden kan man bestämma hur länge sedan en levande organism dog. Allt levande
absorberar CO2 från luften. Av dessa molekyler är majoriteten vanligt kol-12, men en liten del är kol-14
och halten i atmosfären (eller förhållandet mellan kol-12 och kol-14) har varit ungefär densamma i
tusentals år. Man vet att kol-14 har en halveringstid på 5730 år. När en organism dör så tar den inte
längre upp någon koldioxid, det betyder att förhållandet mellan kol-12 och kol-14 kommer att minska
med hälften var 5730:e år. Det betyder att om ett träd har ett förhållande kol-14/kol-12 som är hälften av
det i levande träd så är det ungefär 5700 år sedan det dog. Metoden fungerar bra om man vill veta
åldern på föremål som är yngre än 60 000 år. I äldre objekt så är kol-14 halten så liten att det är svårt att
mäta den exakt. För att bestämma ålder på äldre objekt så kan man titta på halten uran-238 som har en
halveringstid på 4,5 miljarder år. Metoden kan användas för att bestämma ålder på stenar.

Kol har atomnummer 6 dvs 6 protoner i kärnan. Det kan finnas 6, 7 eller 8 neutroner. Det innebär att den
finns tre olika isotoper för kol. Man kan skriva:

Kol-13 är också intressant eftersom det finns i fiskar och havsdjur. Man kan t ex mäta halten av kol-13 i
ben och får reda på vad stenåldersmänniskor har ätit mycket av.

Problemlösning med Kol-14-metoden. Klicka på bilden!

https://www.youtube.com/watch?v=RUbVyES2_YY HYPERLINK "https://www.youtube.com/watch?


v=RUbVyES2_YY&feature=youtu.be"& HYPERLINK "https://www.youtube.com/watch?
v=RUbVyES2_YY&feature=youtu.be"feature=youtu.be

Laboration nr 5
I samband med denna modul skall du genomföra en laboration, och det är nu dags för denna. Det är en
hemlaboration.

Laborationsinstruktion och rapportmall hittar du genom att klicka på knapparna nedan. (Du kan också
komma åt dessa om du klickar på "Laborationer" i Introduktionsmodulen eller i Kursbiblioteket sist i
denna studieguide.)

Du skall alltså nu göra Laboration nr 5!

Laboration nr 5: Halveringstid

http://novo.hermods.se/pluginfile.php/270167/mod_page/content/43/Halveringstid.pdf

Rapport: Bifogas med Uppdrag 4

http://novo.hermods.se/pluginfile.php/254482/mod_page/content/13/1.%20Rapportmall%20med
%20checklista%2C%20rev.pdf

Kärnenergi
I följande text tar vi upp begreppet kärnenergi. Motsvarande text hittar du i läroboken på sid 349 - 374.

Den starka kärnkraften, en av de fyra naturkrafterna, håller ihop atomkärnan. Denna kraft är ca 10 40
gånger starkare än gravitationskraften! Dock verkar den över väldigt korta avstånd, ca 10 - 15 m. Om man
kan utvinna energi ur en atomkärna leder det till frigörelse av oerhörda mängder energi, vilket sker i
atom- och vätebombsexplosioner.

Det finns två tillvägagångssätt. Antingen genom s k fission, klyvning av atomkärnor, vilken sker i
atombombsexplosioner, eller sammanslagning av atomkärnor, s k fusion. Detta sker i en
vätebombsexplosion och i stjärnors inre. En stjärna (som solen) är en kontrollerad vätebomb. Om
fusionsprocesserna går för fort utvidgar sig stjärnans kärna, där processerna mestadels äger rum, och
sammanslagningarna sker i långsammare takt eftersom temperaturen då minskar. Om det sker för få
sammanslagningar drar kärnan ihop sig och fusionen går snabbare eftersom temperaturen ökar.

De flesta grundämnen vi känner till som är tyngre än helium bildas genom fusionsprocesser i stjärnors
inre, de övriga bildas genom supernovaexplosioner. Vi människor består alltså till stor del av
stjärnmaterial!!!

Elementarpartiklar
Partiklar som protoner, neutroner, elektroner, fotoner etc. går under beteckningen elementarpartiklar.
De har olika egenskaper; massor laddningar, spinn, livstider etc. En partikel har i regel en antipartikel, en
partikel med samma massa men omvänd laddning. En elektron, kan slå sig ihop med en positron,
(antielektron), då förintas de och ljus (fotoner) sänds ut. Detta kallas annihilation.

Det omvända är också möjligt, energirika fotoner kan omvandlas till partikel + antipartikel. Detta kallas
parbildning och ska diskuteras i mån av tid. Det finns ett stort antal elementar-partiklar. En del av dessa
kommer vi att stöta på och man kan notera att de ”vanligaste” partiklarna, protonen och neutronen,
tillhör gruppen baryoner medan elektroner, neutriner och myoner tillhör leptonerna.

Titta på en genomgång av elementarpartiklar!

https://www.youtube.com/watch?v=HGhhF8BXfWI HYPERLINK "https://www.youtube.com/watch?


v=HGhhF8BXfWI&feature=youtu.be"& HYPERLINK "https://www.youtube.com/watch?
v=HGhhF8BXfWI&feature=youtu.be"feature=youtu.be

Fission
I slutet av 1930-talet gjordes ett försök att tillverka nya grundämnen genom att beskjuta tunga kärnor
med neutroner. När resultatet analyserades upptäcktes det att de nya kärnorna var lättare än
ursprungskärnan. Den österrikiska fysikern Lise Meitner lyckades förklara processen teoretiskt. Fissionen,
kärnklyvningen, var upptäckt. En typisk fissionsreaktion ser ut på följande sätt:

En urankärna beskjuts med långsamma neutroner. En neutron tränger in i urankärnan och en


mellankärna bildas. Denna mellankärna är mycket instabil och klyvs i två nya kärnor. De nya kärnorna har
masstal som är ungefär A = 95 och A = 140. Antalet protoner är totalt lika med 92. Sannolikheten för att
urankärnan ska klyvas i två lika stora delar är i stort sett lika med noll. Det bildas alltid en lättare och en
tyngre kärna, men det finns många olika möjligheter. Några möjliga alternativ visas här nedan till höger.

Naturligt uran består av nukliderna:

I kärnkraftverk används uran-235 som bränsle. Halten uran-235 anrikas till ca 3%.
Fission, forts
Dotterkärnorna är mycket instabila och sönderfaller genom att sända ut β- och γ-strålning. Vid fissionen
frigörs också 2 - 3 nya neutroner. Detta gör det möjligt att få till stånd en kedjereaktion. Neutronerna
bromsas in och träffar nya urankärnor som klyvs. Då frigörs ännu fler neutroner o.s.v. Vid fissionen frigörs
också stora mängder energi.

Den första tillämpningen av fission blev tyvärr atombomberna som fälldes över Hiroshima och Nagasaki i
slutskedet av andra världskriget. I en atombomb tillåts fissionen att fortsätta så att så mycket energi som
möjligt frigörs vid explosionen.

Efter andra världskriget har fissionsprocessen också utnyttjats för energiframställning i kraftverk. I ett
kärnkraftverk kontrolleras fissionen med hjälp av styrstavar som kan absorbera neutroner. På så sätt ska
fissionen aldrig kunna ske okontrollerat. Vid svenska kärnkraftverk kontrolleras säkerheten regelbundet
av statliga myndigheter. Vid en del kärnkraftverk i andra länder är risken för olyckor betydligt större,
vilket Tjernobylolyckan 1986 visade.

Du kan läsa mer om både kärnkraft och kärnvapen i något uppslagsverk. Det finns också en del
informationsmaterial från kraftbolagen och de statliga myndigheterna. Undersök också om du har
möjlighet att göra studiebesök på ett kärnkraftverk.

Lös uppgifterna 15.1 - 15.3 på s 358 i din lärobok.

Utforska fission

Klicka på bilden för att utforska fission på egen hand!

http://phet.colorado.edu/en/simulation/nuclear-fission

Exempel på fission
Exempel på var man använder fission är atombomber där kedjereaktioner sker okontrollerat, och det är
faktiskt meningen att den ska göra massor med skada (mycket energi på en gång) samt i kärnkraftverk
där vi kontrollerar kedjereaktionen och tar ut energi på mycket längre tid. Varje klyvning frigör 2-3
neutroner som i sin tur klyver nya kärnor. Detta orsakar en kedjereaktion. Varje klyvning skapar tre nya
neutroner. Klyvningen ökar exponentiellt och blir explosionsartad (atombomb!) om halten uran-235 är
tillräckligt hög (50 %) och uranmassan kritisk (30kg).

Om halten och massan av uran-235 eller plutonium-239 är tillräckligt stor startar en okontrollerbar
kedjereaktion. I Hiroshimabomben år 1945 var halten uran-235 50% och uranmassan 60kg, stor som en
fotboll. Sprängkraften motsvarade 13000 ton trotyl, detonerade 580 meter över stadens centrum och
orsakade 78000 civilas död.

I våra kärnkraftverk får vi värmeenergi genom kärnklyvning. Det bränsle som utnyttjas i en kärnreaktor är
uran-235. När denna urankärna beskjuts med en neutron kommer den i så häftig gungning att den klyvs i
två lättare kärnor och tre neutroner.
Om man tänker sig att man väger neutronen och uran-235 kärnan och sedan väger de två
klyvningsprodukterna tillsammans med de tre neutronerna, märker man att det saknas lite massa. Det är
denna massa som omvandlats till energi enligt Einsteins berömda formel E = m · c2. Vi får ut energi
genom kärnklyvning eller fission.

E är energin i joule

m är massan i kg

c är ljusets hastighet i m/s

Uran blir plutonium

Titta på filmen ovan som beskriver en reaktor!

https://www.youtube.com/watch?v=szpnRx7U41M HYPERLINK "https://www.youtube.com/watch?


v=szpnRx7U41M&feature=youtu.be"& HYPERLINK "https://www.youtube.com/watch?
v=szpnRx7U41M&feature=youtu.be"feature=youtu.be

Fusion
Läs s 355-357 i läroboken. Vid fusion slås två lätta kärnor samman, samtidigt som energi frigörs. Solens
energi kommer från fusionsreaktioner i solens inre. Den enklaste fusions-reaktionen kan sammanfattas i
följande reaktionsformel:

Forskare har i många år försökt att tekniskt "tämja" fusionsprocessen, eftersom en fusionsreaktor skulle
lösa världens energiproblem för lång tid framåt. Tyvärr är de tekniska problemen mycket stora. Det krävs
en temperatur på miljoner grader för att fusionen ska äga rum. Energin i fusion och fission kommer från
att en del av atom-kärnans massa omvandlas till energi enligt Einsteins berömda formel E = mc 2.

I solens inre finns de flera miljoner grader som krävs för att övervinna Coulombkraften och slå ihop,
fusionera, atomkärnor. I fusionen frigörs energi enligt Einsteins ekvation. I solen omvandlas väte till
helium och varje sekund blir 4,2 miljoner ton solmassa ren energi. Solen producerar mer energi på en
sekund än hela mänskligheten på en miljon år. En del av den effekten, 3,86 · 10 26 W träffar Jorden och
möjliggör livet på vår planet. Läs gärna mer om fusionsforskning i något uppslagsverk.

Fusion, forts
Genom samma reaktion får vätebomben sin enorma energi. Temperaturen som krävs för att fusionen ska
komma igång, levereras av en uranbomb som exploderar inne i vätebomben. Den första sprängningen av
en vätebomb gjordes av USA på Bikiniatollen i Stilla havet 25 juli 1952. I resterna av bomben hittade man
ett nytt grundämne med atomnummer 99. Det gavs namnet einsteinium för att hedra Einstein.

Fördelen med fusion är att det skapas inga långlivade radioaktiva restprodukter. Det skulle vara en
perfekt och billig energikälla eftersom bränsle som är ju väte är det mest vanliga ämnet i Universum.
Fusion frigör mer energi än fission men som sagt, det är svårt att åstadkomma. I nuläget finns några
forskningsreaktorer men energin som krävs för den höga temperaturen överstiger energin som fås ut.
Försöksanläggningen JET(Joint European Torus) finns i Storbritannien och ITER (International
Thermonuclear Experimental Reactor) som ska stå klar i Frankrike.

Fusionen mellan deuterium och tritium (D-T-reaktion) är enklast och sker i en vätebomb:

Fusion i solen
Solen består av 73 % väte, 25 % helium och resten bland annat syre, järn och kol. Halten av väte minskar
och halten av helium ökar. Hans Bethe (1906-2005) förklarade 1939 solens fusionsprocess och fick
Nobelpriset 1967. Fusionen kallas proton-proton-cykeln och sker i tre steg:

Steg 1 och 2 görs två gånger innan steg 3 och frigör då totalt 26,3 MeV. Detta sker i solens centrum vid en
temperatur på 15 miljoner grader.

Titta på en genomgång av fission och fusion!

https://www.youtube.com/watch?v=yTkojROg-t8 HYPERLINK "https://www.youtube.com/watch?


v=yTkojROg-t8&feature=youtu.be"& HYPERLINK "https://www.youtube.com/watch?v=yTkojROg-
t8&feature=youtu.be"feature=youtu.be

Biologiska verkningar
Följande text tar upp begrepp från läroboken s 359-373. De biologiska verkningarna av strål-ning
studeras inom en gren av fysiken som kallas radiofysik. Strålningens omfattning och verkan kan beskrivas
med storheter som tar hänsyn till hur mycket energi som absorberas i vävnaden. Några av de
grundläggande begreppen presenteras i det här avsnittet. Läs texten så att du förstår innehållet och kan
lösa uppgifterna. Du behöver inte lära dig de formler som ingår.

Medicinska tillämpningar av kärnfysik är intressanta. Undersök om du har möjlighet att göra studiebesök
på något sjukhus med radiologisk verksamhet. Du kan också läsa mer om medicinska tillämpningar i
något uppslagsverk.

Strålningsskador

Eftersom en partikel med mycket energi kan jonisera atomer och molekyler, ställer detta till skador i
biologisk vävnad. Särskilt allvarlig är om cellens DNA skadas och skadan förs vidare genom arvsmassan.
Extra farligt är också om nervceller skadas. Kroppen kan laga för det mesta skadan men ibland kan det gå
så illa att den skadade cellen inte förstörs utan fortsätter bilda kopior av sig själv och cancer tumörer
utvecklas.
Radioaktivitet i sjukvården

Radium började tidigt användas för att bekämpa cancer. Man förstod att strålningen dödade
cancercellerna. Medicinsk utrustning måste steriliseras med radioaktiv bestrålning eftersom strålningen
dödar bakterierna effektivt. Strålningen går genom förpackningen och påverkar inte produkten på något
farligt sätt.

Stråldos och kvalitetsfaktor


Strålningsenergin som absorberas (J/kg) kallas absorberad dos och mäts i enheten gray (Gy) efter
fysikern Louis Harold Gray. Medicinsk effekt beror på strålningens kvalitetsfaktor vid likartad energi.

1) Alfastrålning är farligast eftersom energin avges på en kort bromssträcka. Kvalitetsfaktor 20.

2) Proton och neutronstrålning, kvalitetsfaktor 10.

3) Röntgen, gamma och betastrålning, kvalitetsfaktor 1 för att de joniserar minst.

Ekvivalent dos (Sievert)

Absorberad dos multiplicerad med kvalitetsfaktorn ger ekvivalent dos som mäts i enheten sievert (Sv).
Enheten är uppkallad efter svenske strålskyddsfysikern Rolf Sievert (1896-1966).

Stråldoser

● Årsdosen bör inte överstiga 5mSv/år

● 50 mSv/år tillåts för personal som arbetar i radioaktiva miljöer t.ex. sjukhus eller

kärnkraftverk

● Normal års dos är 4 mSv/år

● Radon i bostäder (45 %), i sjukvård (35 %), bakgrundsstrålning (19 %)

● Kalium och kalcium i kroppen ger 0,2 mSv/år

● Flygning över Atlanten ger 0,1 mSv

Biologiska effekter, radon


Utan behandling dör 50% vid 3 Sv helkroppsbestrålning. 5 Sv helkroppsbestrålning är dödlig dos.
Personen avlider utan behandling inom 2 veckor. Antibiotika och benmärgtransplantation minskar
dödligheten upp till 12 Sv. 100 Sv leder till nervskador och omedelbar död. Vid bestrålning av hjärntumör
används en lokal dos på 50 Sv.
Radium och radondöttrar

Radium (Ra) finns naturligt i jordskorpan och sönderfaller till den radioaktiva ädelgasen radon (Rn), som
upplöses i grundvatten och luft. Radon sönderfaller i sin tur till radondöttrar genom α−strålning, kan
hamna i lungorna och orsaka lungcancer. Uppskattningsvis dör ca 1500 människor varje år av lungcancer
orsakad av radon i inomhusluft. Radon är en radioaktiv gas som bildas i berggrunden och följer sedan
med luft och vatten in i husen.

I mitten av 1900-talet byggdes hus av blå lättbetong som avger radon. Därför förbjöds den 1974. Radon
sänder ut en alfapartikel och polonium bildas. Denna sönderfaller i sin tur och det blir en hel rad
radondöttrar. Dessa fastnar på damm i luften, vi andas in dem och så fortsätter sönderfallet. Trots att
alfastrålning har en kort räckvidd och stoppas med en bit papper kan den alltså ställa till stora skador
inne i våra lungor.

Lös uppgift 16.1-16.3 på s 375 i läroboken, och gör Laboration nr 5!

Om du vill veta mer om strålning, hittar du några filmer om detta här!

https://www.youtube.com/watch?v=nM23d_ImboM HYPERLINK "https://www.youtube.com/watch?


v=nM23d_ImboM&feature=share&list=PLDE82FF9404E57FF3&index=1"& HYPERLINK
"https://www.youtube.com/watch?
v=nM23d_ImboM&feature=share&list=PLDE82FF9404E57FF3&index=1"feature=share HYPERLINK
"https://www.youtube.com/watch?
v=nM23d_ImboM&feature=share&list=PLDE82FF9404E57FF3&index=1"& HYPERLINK
"https://www.youtube.com/watch?
v=nM23d_ImboM&feature=share&list=PLDE82FF9404E57FF3&index=1"list=PLDE82FF9404E57FF3
HYPERLINK "https://www.youtube.com/watch?
v=nM23d_ImboM&feature=share&list=PLDE82FF9404E57FF3&index=1"& HYPERLINK
"https://www.youtube.com/watch?
v=nM23d_ImboM&feature=share&list=PLDE82FF9404E57FF3&index=1"index=1

https://www.youtube.com/watch?v=nM23d_ImboM HYPERLINK "https://www.youtube.com/watch?


v=nM23d_ImboM&list=PLDE82FF9404E57FF3&feature=share&index=1"& HYPERLINK
"https://www.youtube.com/watch?
v=nM23d_ImboM&list=PLDE82FF9404E57FF3&feature=share&index=1"list=PLDE82FF9404E57FF3
HYPERLINK "https://www.youtube.com/watch?
v=nM23d_ImboM&list=PLDE82FF9404E57FF3&feature=share&index=1"& HYPERLINK
"https://www.youtube.com/watch?
v=nM23d_ImboM&list=PLDE82FF9404E57FF3&feature=share&index=1"feature=share HYPERLINK
"https://www.youtube.com/watch?
v=nM23d_ImboM&list=PLDE82FF9404E57FF3&feature=share&index=1"& HYPERLINK
"https://www.youtube.com/watch?
v=nM23d_ImboM&list=PLDE82FF9404E57FF3&feature=share&index=1"index=1

https://www.youtube.com/watch?v=kmR6pzobpJE HYPERLINK "https://www.youtube.com/watch?


v=kmR6pzobpJE&list=PLDE82FF9404E57FF3&feature=share&index=2"& HYPERLINK
"https://www.youtube.com/watch?
v=kmR6pzobpJE&list=PLDE82FF9404E57FF3&feature=share&index=2"list=PLDE82FF9404E57FF3
HYPERLINK "https://www.youtube.com/watch?
v=kmR6pzobpJE&list=PLDE82FF9404E57FF3&feature=share&index=2"& HYPERLINK
"https://www.youtube.com/watch?
v=kmR6pzobpJE&list=PLDE82FF9404E57FF3&feature=share&index=2"feature=share HYPERLINK
"https://www.youtube.com/watch?
v=kmR6pzobpJE&list=PLDE82FF9404E57FF3&feature=share&index=2"& HYPERLINK
"https://www.youtube.com/watch?
v=kmR6pzobpJE&list=PLDE82FF9404E57FF3&feature=share&index=2"index=2

https://www.youtube.com/watch?v=jfZ7XEYbwvg HYPERLINK "https://www.youtube.com/watch?


v=jfZ7XEYbwvg&list=PLDE82FF9404E57FF3&feature=share&index=3"& HYPERLINK
"https://www.youtube.com/watch?
v=jfZ7XEYbwvg&list=PLDE82FF9404E57FF3&feature=share&index=3"list=PLDE82FF9404E57FF3
HYPERLINK "https://www.youtube.com/watch?
v=jfZ7XEYbwvg&list=PLDE82FF9404E57FF3&feature=share&index=3"& HYPERLINK
"https://www.youtube.com/watch?
v=jfZ7XEYbwvg&list=PLDE82FF9404E57FF3&feature=share&index=3"feature=share HYPERLINK
"https://www.youtube.com/watch?
v=jfZ7XEYbwvg&list=PLDE82FF9404E57FF3&feature=share&index=3"& HYPERLINK
"https://www.youtube.com/watch?
v=jfZ7XEYbwvg&list=PLDE82FF9404E57FF3&feature=share&index=3"index=3

https://www.youtube.com/watch?v=RFkFTIBRfEQ HYPERLINK "https://www.youtube.com/watch?


v=RFkFTIBRfEQ&feature=share&list=PLDE82FF9404E57FF3&index=4"& HYPERLINK
"https://www.youtube.com/watch?
v=RFkFTIBRfEQ&feature=share&list=PLDE82FF9404E57FF3&index=4"feature=share HYPERLINK
"https://www.youtube.com/watch?
v=RFkFTIBRfEQ&feature=share&list=PLDE82FF9404E57FF3&index=4"& HYPERLINK
"https://www.youtube.com/watch?
v=RFkFTIBRfEQ&feature=share&list=PLDE82FF9404E57FF3&index=4"list=PLDE82FF9404E57FF3
HYPERLINK "https://www.youtube.com/watch?
v=RFkFTIBRfEQ&feature=share&list=PLDE82FF9404E57FF3&index=4"& HYPERLINK
"https://www.youtube.com/watch?
v=RFkFTIBRfEQ&feature=share&list=PLDE82FF9404E57FF3&index=4"index=4

https://www.youtube.com/watch?v=_0KL4juhAXk HYPERLINK "https://www.youtube.com/watch?


v=_0KL4juhAXk&list=PLDE82FF9404E57FF3&feature=share&index=5"& HYPERLINK
"https://www.youtube.com/watch?
v=_0KL4juhAXk&list=PLDE82FF9404E57FF3&feature=share&index=5"list=PLDE82FF9404E57FF3
HYPERLINK "https://www.youtube.com/watch?
v=_0KL4juhAXk&list=PLDE82FF9404E57FF3&feature=share&index=5"& HYPERLINK
"https://www.youtube.com/watch?
v=_0KL4juhAXk&list=PLDE82FF9404E57FF3&feature=share&index=5"feature=share HYPERLINK
"https://www.youtube.com/watch?
v=_0KL4juhAXk&list=PLDE82FF9404E57FF3&feature=share&index=5"& HYPERLINK
"https://www.youtube.com/watch?
v=_0KL4juhAXk&list=PLDE82FF9404E57FF3&feature=share&index=5"index=5
https://www.youtube.com/watch?v=0iPHt4X57ks HYPERLINK "https://www.youtube.com/watch?
v=0iPHt4X57ks&feature=share&list=PLDE82FF9404E57FF3&index=6"& HYPERLINK
"https://www.youtube.com/watch?
v=0iPHt4X57ks&feature=share&list=PLDE82FF9404E57FF3&index=6"feature=share HYPERLINK
"https://www.youtube.com/watch?
v=0iPHt4X57ks&feature=share&list=PLDE82FF9404E57FF3&index=6"& HYPERLINK
"https://www.youtube.com/watch?
v=0iPHt4X57ks&feature=share&list=PLDE82FF9404E57FF3&index=6"list=PLDE82FF9404E57FF3
HYPERLINK "https://www.youtube.com/watch?
v=0iPHt4X57ks&feature=share&list=PLDE82FF9404E57FF3&index=6"& HYPERLINK
"https://www.youtube.com/watch?
v=0iPHt4X57ks&feature=share&list=PLDE82FF9404E57FF3&index=6"index=6

Relativitetsteori och standardmodell


Detta sista avsnitt är av mer orienterande art. Vi skriver inte så mycket om det här i Novo, utan
uppmanar läsaren att läsa igenom lärobokens kapitel 12, sid 281 - 291. Det är en fascinerande läsning! I
slutet av kapitlet finns några räkneuppgifter - dessa skall du givetvis lösa. Även i det sista uppdraget
dyker någon sådan upp.

Tidsresor är en standardingrediens i sciencefiction-berättelser, men är det verkligen möjligt? Det visar sig
att naturen tillåter på ett sätt att böja tid, en spännande möjlighet som föreslagits av Albert Einstein när
han upptäckte speciella relativitetsteorin över hundra år sedan.

Relativ rörelse enligt Newton

Ett tåg rör sig med farten v1 = 25 m/s åt höger relativt marken. En människa går åt höger med farten v2 =
2 m/s relativt tåget. Människan rör sig relativt marken med farten v1 + v2 = (25 + 2) m/s = 27 m/s. Ganska
självklart!

Michelson-Morleys experiment 1887

Fysikerna A.A. Michelson (1852-1931) och E.W. Morley (1838- 1923) mätte år 1887 ljushastigheten för
ljus från stjärnor framför respektive bakom Jorden i sin bana runt solen. Jordens hastighet i banan runt
solen är 30 km/s. Mätningen visade, att ljushastigheten blev samma i båda fallen, ca 300 000 km/s.

Albert Einstein
Albert Einstein (1879-1955)

Inspirerad av Michelson och Morleys experiment samt av bl.a. Henrik Lorenz, formulerar Einstein 1905
den speciella relativitetsteorin. Teorin gäller för föremål som rör sig med konstant hastighet och bygger
på två antaganden:

1.) Befinner man sig i ett system som rör sig relativt ett annat system, kan man inte avgöra vilket av
systemen som rör sig. (Enkelt att förstå, tänk på tåget.)

2.) Ljushastigheten uppmäts alltid till c = 299 792 458 m/s i vakuum, oavsett observatörens hastighet.
(Svårt att förstå, tänk på ljuset från bilens strålkastare.)

En konsekvens av att ljushastigheten är konstant, oavsett referenssystem är:

Två händelser som är samtidiga i ett system är inte samtidiga i ett annat system som rör sig relativt det
första.

Om du vill veta mer om strålning, hittar du några filmer om detta här!

Klicka på bilderna nedan för att få en annorlunda förklaring på saker och ting!

https://www.youtube.com/watch?v=wteiuxyqtoM HYPERLINK "https://www.youtube.com/watch?


v=wteiuxyqtoM&feature=youtu.be"& HYPERLINK "https://www.youtube.com/watch?
v=wteiuxyqtoM&feature=youtu.be"feature=youtu.be

https://www.youtube.com/watch?v=KYWM2oZgi4E HYPERLINK "https://www.youtube.com/watch?


v=KYWM2oZgi4E&feature=youtu.be"& HYPERLINK "https://www.youtube.com/watch?
v=KYWM2oZgi4E&feature=youtu.be"feature=youtu.be

https://www.youtube.com/watch?v=trNYWekoGE0 HYPERLINK "https://www.youtube.com/watch?


v=trNYWekoGE0&feature=youtu.be"& HYPERLINK "https://www.youtube.com/watch?
v=trNYWekoGE0&feature=youtu.be"feature=youtu.be

https://www.youtube.com/watch?v=bG7c-e_0Nf4 HYPERLINK "https://www.youtube.com/watch?


v=bG7c-e_0Nf4&feature=youtu.be"& HYPERLINK "https://www.youtube.com/watch?v=bG7c-
e_0Nf4&feature=youtu.be"feature=youtu.be

https://www.youtube.com/watch?v=C2VMO7pcWhg HYPERLINK "https://www.youtube.com/watch?


v=C2VMO7pcWhg&feature=youtu.be"& HYPERLINK "https://www.youtube.com/watch?
v=C2VMO7pcWhg&feature=youtu.be"feature=youtu.be

https://www.youtube.com/watch?v=j72bPmXsyvk HYPERLINK "https://www.youtube.com/watch?


v=j72bPmXsyvk&feature=youtu.be"& HYPERLINK "https://www.youtube.com/watch?
v=j72bPmXsyvk&feature=youtu.be"feature=youtu.be

Tidsdilatation
Tiden för en händelse upplevs olika lång för observatörer som rör sig med olika hastighet.

Ska tidsdilatationen vara märkbar måste hastigheten vara stor, nära ljushastigheten. Ljuset från en lampa
som tänds i ett rymdskepp som flyger med hög hastighet, träffar bak- och framväggen samtidigt enligt en
medresenär B. Observatören A på marken upplever däremot att ljuset träffar bakväggen före
framväggen.

Om vi skickar en ljuspuls från golvet till taket i ett framrusande rymdskepp och mäter tiderna då får vi
följande. Medresenären B mäter tiden till �𝑡 och observatören på marken mäter tiden till t0. Detta
kallas för tidsdilatation och följande samband gäller om vi använder Pythagoras sats.
Betraktaren A på marken upplever att ljuspulsen går en längre väg än vad observatören på marken
upplever. Eftersom ljushastigheten är lika för båda, måste tiden de mäter vara olika. Enligt härledningen
med hjälp av Pythagoras sats upplever betraktaren A att tiden går saktare i rymdskeppet som han
observerar, dvs. t är mindre än t0. Å andra sidan upplever resenären B i skeppet att tiden går saktare för
A eftersom A rör sig också relativt B.

Tvillingparadoxen m.m.
Om tvillingen B reser iväg med mycket hög hastighet från sin tvilling A, kommer B, (enligt A) att vara
yngre än A vid återkomsten. Å andra sidan upplever B att A är lika mycket yngre än B eftersom A har rört
sig lika fort relativt B. Detta kallas för tvillingparadoxen.

I verkligheten upplever båda att B är yngre än A eftersom B har påverkats av accelerationen under start
och inbromsning. Problemet kan lösas med allmänna relativitetsteorin.

Vid vilka farter märks tidsdilatationen?

Diagrammet visar att tidsdilatationen är märkbar först vid mycket höga hastigheteter. Den är ungefär 1 %
vid 14 % av ljushastigheten, dvs. 42 000 km/s. Som jämförelse är jordens banhastighet runt solen 30
km/s, vilket också är fartrekordet för rymdsonden Voyager.

Titta på två filmer om tidsdilatation!

https://www.youtube.com/watch?v=HHRK6ojWdtU HYPERLINK "https://www.youtube.com/watch?


v=HHRK6ojWdtU&feature=youtu.be"& HYPERLINK "https://www.youtube.com/watch?
v=HHRK6ojWdtU&feature=youtu.be"feature=youtu.be

https://www.youtube.com/watch?v=sdtlAX3cX9Q HYPERLINK "https://www.youtube.com/watch?


v=sdtlAX3cX9Q&feature=share&list=PLkscb2tJGJjJBi7-4DFVHxQv_4sM8Z9oB&index=2"& HYPERLINK
"https://www.youtube.com/watch?v=sdtlAX3cX9Q&feature=share&list=PLkscb2tJGJjJBi7-
4DFVHxQv_4sM8Z9oB&index=2"feature=share HYPERLINK "https://www.youtube.com/watch?
v=sdtlAX3cX9Q&feature=share&list=PLkscb2tJGJjJBi7-4DFVHxQv_4sM8Z9oB&index=2"& HYPERLINK
"https://www.youtube.com/watch?v=sdtlAX3cX9Q&feature=share&list=PLkscb2tJGJjJBi7-
4DFVHxQv_4sM8Z9oB&index=2"list=PLkscb2tJGJjJBi7-4DFVHxQv_4sM8Z9oB HYPERLINK
"https://www.youtube.com/watch?v=sdtlAX3cX9Q&feature=share&list=PLkscb2tJGJjJBi7-
4DFVHxQv_4sM8Z9oB&index=2"& HYPERLINK "https://www.youtube.com/watch?
v=sdtlAX3cX9Q&feature=share&list=PLkscb2tJGJjJBi7-4DFVHxQv_4sM8Z9oB&index=2"index=2

Längdkontraktion
Avståndet i rörelseriktningen förkortas vid höga hastigheter. Detta kallas för längdkontraktion. Förklaring:
B passerar en sträcka mellan två punkter med farten v�𝑣på tiden t0 enligt sin klocka. Sträckan som
uppmäts av B är alltså l = v · t0. A är i vila relativt de två punkterna på sträckan och upplever att passagen
tar tiden t och att sträckan är l0 = v · t. Eftersom t0 < t måste l0 > l.

Se filmen om längdkontraktion!

https://www.youtube.com/watch?v=4T9qgVbakaw HYPERLINK "https://www.youtube.com/watch?


v=4T9qgVbakaw&list=PLkscb2tJGJjJBi7-4DFVHxQv_4sM8Z9oB&feature=share&index=3"& HYPERLINK
"https://www.youtube.com/watch?v=4T9qgVbakaw&list=PLkscb2tJGJjJBi7-
4DFVHxQv_4sM8Z9oB&feature=share&index=3"list=PLkscb2tJGJjJBi7-4DFVHxQv_4sM8Z9oB HYPERLINK
"https://www.youtube.com/watch?v=4T9qgVbakaw&list=PLkscb2tJGJjJBi7-
4DFVHxQv_4sM8Z9oB&feature=share&index=3"& HYPERLINK "https://www.youtube.com/watch?
v=4T9qgVbakaw&list=PLkscb2tJGJjJBi7-4DFVHxQv_4sM8Z9oB&feature=share&index=3"feature=share
HYPERLINK "https://www.youtube.com/watch?v=4T9qgVbakaw&list=PLkscb2tJGJjJBi7-
4DFVHxQv_4sM8Z9oB&feature=share&index=3"& HYPERLINK "https://www.youtube.com/watch?
v=4T9qgVbakaw&list=PLkscb2tJGJjJBi7-4DFVHxQv_4sM8Z9oB&feature=share&index=3"index=3

Massa och hastighet

Massa och energi


Följande formel är relativitetsteorins mest kända formel.

E = m · c2

Formeln (Einsteins berömda) säger att massa kan omvandlas till energi och tvärtom. Detta händer
fullständigt om materia och antimateria kolliderar med varandra, men det är ganska ovanligt. Vid
kärnreaktioner i t.ex. kärnreaktorer omvandlas en bråkdel av massan till rörelseenergi hos protoner och
neutroner.

Klicka på filmerna nedan för att se en förklaring och ett exempel på problemlösning!

https://www.youtube.com/watch?v=fOLfqkB2zMU HYPERLINK "https://www.youtube.com/watch?


v=fOLfqkB2zMU&list=PLkscb2tJGJjJBi7-4DFVHxQv_4sM8Z9oB&feature=share&index=4"& HYPERLINK
"https://www.youtube.com/watch?v=fOLfqkB2zMU&list=PLkscb2tJGJjJBi7-
4DFVHxQv_4sM8Z9oB&feature=share&index=4"list=PLkscb2tJGJjJBi7-4DFVHxQv_4sM8Z9oB HYPERLINK
"https://www.youtube.com/watch?v=fOLfqkB2zMU&list=PLkscb2tJGJjJBi7-
4DFVHxQv_4sM8Z9oB&feature=share&index=4"& HYPERLINK "https://www.youtube.com/watch?
v=fOLfqkB2zMU&list=PLkscb2tJGJjJBi7-4DFVHxQv_4sM8Z9oB&feature=share&index=4"feature=share
HYPERLINK "https://www.youtube.com/watch?v=fOLfqkB2zMU&list=PLkscb2tJGJjJBi7-
4DFVHxQv_4sM8Z9oB&feature=share&index=4"& HYPERLINK "https://www.youtube.com/watch?
v=fOLfqkB2zMU&list=PLkscb2tJGJjJBi7-4DFVHxQv_4sM8Z9oB&feature=share&index=4"index=4
https://www.youtube.com/watch?v=GZYMeDbh1QU HYPERLINK "https://www.youtube.com/watch?
v=GZYMeDbh1QU&feature=youtu.be"& HYPERLINK "https://www.youtube.com/watch?
v=GZYMeDbh1QU&feature=youtu.be"feature=youtu.be

Rörelse och hastighet

Mer om Einstein och relativitetsteori


År 1915 presenterade Einstein den allmänna relativitetsteorin (som en generalisering av Newtons
gravitationslag) som beskriver en accelererad rörelse. Teorin har revolutionerat fysiken och visar
följande:

*) Acceleration och gravitation är samma sak

*) Gravitationen böjer ljus

Han anses av många vara 1900-talets absolut främste vetenskapsman och blev världsberömd när han
lade fram sin speciella och sin allmänna relativitetsteori. Einstein baserade sina teorier på ljusets
konstanta hastighet, och med dem motsatte han sig inte bara det vårt sunda förnuft säger, utan även
många av de tidigare vetenskapliga antagandena. Vissa av dessa teorier var så komplexa att det skulle
dröja många år efter hans död innan man kunde bevisa dem genom experiment.

Sammanfattning
I den här modulen har du jobbat med:

*) Den moderna fysikens utveckling

*) Kärnfysik

*) Relativitetsteori och standardmodell

Det du nu skall göra är att repetera hela kapitlet (lös gärna fler övningar i din lärobok). Därefter gör du
TDS 4:1. Eventuellt kan det sedan behövas ytterligare lite repetition. När du tycker att du behärskar
samtliga moment i denna modul gör du Uppdrag 4. Detta skickar du in till din webblärare via datorn, och
denne rättar och kommenterar det. Glöm inte att bifoga laborationsrapporten. Du hittar laborationen här
nedan.

Det som därefter återstår är den Avslutande examinationen, och därefter är du klar med din kurs.

Hoppas att du har tyckt om kursen. Tack för den här tiden - och lycka till i framtiden!

Uppdrag, bedömning och kunskapskrav


Uppdragen är en viktig del i kunskapsinhämtningen. Det är därför viktigt att du är noggrann när du
arbetar med uppdragen, och att du är säker på att du förstår och behärskar det du skickar in. Vid den
Avslutande examinationen kommer du att få visa just detta.

Hur mycket arbete du ska lägga ner på svaren styrs bl a av din betygs- ambition, dina förkunskaper, hur
snabbt du läser kursen etc.

Jobba för att hålla studieplanen och de inlämningsdatum som finns i Novo. Risken är annars att du får för
mycket att göra i slutet av kursen. Kom också ihåg att betyget sätts efter den Avslutande examinationen.
Det är meningen att du skall utvecklas och lära dig under kursens gång.

Uppgifterna i uppdragen är uppdelade i olika betygsnivåer (E, C och A). Efter varje fråga anges vilken nivå
den ligger på.

Läs om bedömning och betyg i det svarta fältet. Vi som är lärare i kursen vill att du skall nå ditt mål. Vi
skall också säkerställa att du når Skolverkets uppsatta kunskaps-krav. Vill du repetera kunskapskraven för
kursen? Tryck då på ikonen så kommer du direkt till Skolverkets hemsida.

https://www.skolverket.se/undervisning/vuxenutbildningen/komvux-gymnasial/laroplan-for-vux-och-
amnesplaner-for-komvux-gymnasial/amne?url=1530314731%2Fsyllabuscw%2Fjsp%2Fsubject.htm
%3FsubjectCode%3DFYS%26courseCode%3DFYSFYS01a%26lang%3Dsv%26tos%3Dgy%26webtos
%3Dvuxgy%26p%3Dp HYPERLINK "https://www.skolverket.se/undervisning/vuxenutbildningen/komvux-
gymnasial/laroplan-for-vux-och-amnesplaner-for-komvux-gymnasial/amne?
url=1530314731%2Fsyllabuscw%2Fjsp%2Fsubject.htm%3FsubjectCode%3DFYS%26courseCode
%3DFYSFYS01a%26lang%3Dsv%26tos%3Dgy%26webtos%3Dvuxgy%26p
%3Dp&sv.url=12.b173ee8160557dd0b8100d#anchor_FYSFYS01a"& HYPERLINK
"https://www.skolverket.se/undervisning/vuxenutbildningen/komvux-gymnasial/laroplan-for-vux-och-
amnesplaner-for-komvux-gymnasial/amne?url=1530314731%2Fsyllabuscw%2Fjsp%2Fsubject.htm
%3FsubjectCode%3DFYS%26courseCode%3DFYSFYS01a%26lang%3Dsv%26tos%3Dgy%26webtos
%3Dvuxgy%26p
%3Dp&sv.url=12.b173ee8160557dd0b8100d#anchor_FYSFYS01a"sv.url=12.b173ee8160557dd0b8100d#
anchor_FYSFYS01a

Dina resultat under kursens gång samlas i en matris. Matrisen visar i vilken utsträckning du uppnått de
olika kunskapskraven i kursen. Matrisen, tillsammans med lärarens kommentarer, vägleder dig i vad du
behöver träna mer på för att nå ditt betygsmål. Efter den avslutande examinationen kommer matrisen
justeras en sista gång och därmed utgöra en del av betygsmotiveringen. Kom ihåg att nivån du visat
under kursens gång också behöver visas under den avslutande examinationen.

You might also like